Вы находитесь на странице: 1из 164

i-= - "-'-: ,* .

''
.

...".; :. i'j .J
i'.."j.'.,;l,-'t: ;-|

,.t i

':.::i
,t
I
I

-'n
u{ {'
tftdii
t^
! i t J iJ it
ffi&
ffire
:...:] 1j {={=:+

{.t ; i I _.r ! ..
,; 1 :, '1 f ;ri!
:', ;rr : l ':
ii;\i{...:rf,: r ? l;
_J
-:- ,,r t
..:,.

ltIJ

,''.','
, :.J'. rira*$i
! ,t iirlr\if"{,]..$t

: i,l: {
't
I,
,{
{4

{ j;t;,'l { .,,\'r;,', l.
{.
J
,{ $.rj *,
!l
i irir\i{,{"ij{,; }
:! a
.f 1,1

j:it
.i:,,

t' i-

ii
Gontents
Preface
Dedication
Acknowledgements vi
Abbreviations vi
vii
Ouestions 1-10
Answers'l-10 1

Ouestions 1t-20 10

Answers 11-20 19
Ouestions 21-gO 31
Answers Z1-3O 41

Ouestions 31-40 49
Answers 31-40 57
Ouestions 41-S0 72
Answers 41-50 81
Ouestions S1-60 92
Answers 51-60 101
"112
Ouestions 6l-70
Answers 61-70 121
Questions 71-gO 133
Answers 7i-90 141
Questions g1-90 151
Answers 81-gO 159
Ouestions g1-1OO 171
Answers g1-1OO \ 181
190
ouESTroNs 1-10

Case I
A.13-year-old girl presented to iccident and Emergency (A&E) with a
history of lethargy, joint pain and cough. Both of hlr parents are from
Jamaica. Her problem started 1 month ago, with an upper respiratory
tract infection (URTI). She was seen by her family docior, and a viral'
infection was diagnosed. She had a past history of rashes on her
body, but her family doctor never saw them" On examination, she has
palpable cervical lymph nodes of various sizes, a congested throat,
generalised myalgia and a swollen ankle joint on the ieft side. Her Bp
is 120175 mmHg, HR 90 b.p.nl. and RH 2ljmin" A urine test showed
protein + with red cells. Other test results are:
Hb 9'7 slot
wee 4 x 10sll with neutropenia and lymphopenia
PLT 100 x 1Oe/l
Ret 2.6%
CRP 20
'1. Which three other important investigations should be carried
out?
2. What treatments should you prescr,ik?
3. What are three possible differentia! diagnoses?
She was admitted for further investigation and on the second niglrt,
her oxygen requirement increased to 3 Unrin via a facemask to
rnaintain a Sat level above g3%. p,f l eultures were negative,
4. Which single investigation should you carry out?
l-ier eo.ndition got \ /orse, she was transferred to the paediatric
intensive care unit (PlCt,), she required ventilation, jnd othei
procedures were carried out. Her blood pressure remained high
(130/9S mmFlg) and she was treated with nifedipine.
A, kidney*US
scan was reported as normal, as was a horrnonal study. She was
treated with antibiotics for 7 days. A viral titre shows tle ror Etsv" ,q
fitlantsux te_st is very weakly positive, although she hal never
received a BCG.
5. Whicl'l three other investigations may help the diagnosis?
6. What other treatment should be added at this stag-uZ

ease 2
Ai: ir:f*nt baby.girl.aged 86 rncnths was seen in A&E with a l-ristory *f
vcmiting anci dlarrho*a for the pravious 3 days. [n the last t4 hours.
she beeanre irritoble alrd gre'w p*ie. $he ls n*t intei"ested ;n **tiri
bui"rs ct!ii clil"riiirrg fiuic*. $hs l"v*s, *i:rr-: fuii terr.c *6d l:*r r-,i: pi,*;i,r:,,ii
.r.;!iJ...,i-. ?;,a -lff. lir" l,"r*? :,. * li, .1. i .r,:.;r,;r:i;;3r ::Cnt,"A i.:";,;
t:-1 -Z
.l;,,:- ts i- i::. ri,, :ft*iiilrei":i1 .;h.; r*Cti:ti iS hea,,ilirrr pll:;i"t";1L, Tir:r
:;lfl:;"r': !,-'i:l"S fi::i;t ,,.,t1.'i, * J:*:-ipi:;1.:ri ..;:.ijiat._; X:flii iS;_i] 1-1j. *q r; i.i::. i..1ti i;
2 100 Grey Cases in Paediatrics lor MRCFCH

140 b.p.m., RR 30/min and BP90/60 mmtlg' Herliver is.3.cm below


trre rloht costai margin, the right kidney is palpable and there is no
i;ilT;;" H;i uri* itro*" prd=tein +, BBC ++ and no organisms or
ieukocytes. All cultures are negative'
1. List four immediate investigations
a ESR
b INR
c Renal US with DoPPler
d Urea and electrolYtes (UAE)
A Hepatitis B and C serologY
t Fuil blood count (FBC)
g Z4-hour urinary Protein
h VMA
i Renal bioPsY
2. What is the mosi likelY diagnosis?
a Haemolytic uraemic syndrome (HUS)
b UTI
c Gastroenteritis
d Obstructive renal failure
e Right renal artery thrombosis
f Ri6ht renal vein thrombosis
g Wilms'tumour
I AcuteglomerulonePhritis
3. Name three stePs of management'

Case 3
An 1l-vear-old bov was referred to A&E with a history of limping
i

;;rr;6 tjnt", and has since:i$:i'^:':t:;:,1:l


;;i;i;';il'i"o-niri!'.rt't9.vPql.o*:!:y-':-9f
#[';iiil;';fj;;;ii.; been comprainin;
familvlri:t?I-oi-tll^11l":::""tl
;;; i; nii knee- rhere
i,li.,'i ii'I,j;i;;h;;ird
'ignt
is no
3 years aqg ryq! an illness described as a
stiot<e, after a long time of unspecified illness' /
as normal'
Svstemic and general examinations wer€ reported
iJr a iiiglt pro-ximalweakness of the right leg'

1. What one test should You catry out?


a ESR
b CRP
Right and left knee X-raY
c
Right knee X-raY
d
Right knee US
e
f Right knee MRI
g Right knee CT
was seen
The boy presented 5 days later with more weakness and
i# ;;li "d"d " "u rs"on, *!' 9 aswhoosed
i d nryscle
i n
3"'ttl?u: :ld'.
would see him ln 2 davs' rlu
;;i;; ffi;;-th- ;;;diatrician, I
same night he came back to A&Etrith Tote Y!"-kt:: ilI"^:tPI
;il'; ;dtht ti. inrt felt'heavy" There is marked wea-kness of the
Questions 1-10 3

proximal muscle groups in the right upper and lower limbs. The
reflexes are present and the sensation is intact.
2. Which three differential djagnoses are correct?
a Guillain-Barr6syndrome
b Myeloencephalitis
c Myositis
d Spinal cord injury
e Sciatic nerve injury
f Hemiplegia secondary to intercerebral insult
g Dysraphism
hMS
i ADEM
3. What two abnormalities appear on his cranial MRI?

4. Which two investigations should be carried out?


a Nerve conduction study
bLP
c EMG
d Cranial MRI
e Muscle biopsy
fCK
g Lower limb rnuscle MRI
After 2 days in hospitalthe boy started having difficulty in
swallowing and was not able to use his right ieg.
-
5. What is the diagnosis?

Case 4
$irrce the age of 3 years, this 7-year-oid girl has presented with a
history of recurrent rnouth ulcers. she wis born by a fuli-term norn:ai
delivery TFTND) and her mother said she is a heart-hy, girl. Her mcthe!"
has ulcers in her rncuth fronr time to time but, not as many as hei"
for MRCPCbi
4 100 Grey Cases in Paediatrics

Dr0E r'se 'r!v *


::f}:T,'ff ,8;ll#fs;'ff
srel' $[s-t+:&:-**!pi;il$Ht]'f,si
flbread and a h.ign;rlo1e
norma!' Durins *"ittotiaays her ulcers
the surntner:::lT3ffit"#;
:

il;t-;o?;lrauTt is a period of 6
less frequent and she *lu J"t *tEm ojr-ce-':nlv over
one ulcer on ine
weeks' Shc is J'hianf'V-Voung. girl with
br nuii**t lip' The test results are:
ili=-|"iiJt i--1""u"*ntty
Hb 10.2 gldt
\rycc 7 x 10e/l
PLT 380 x 10eil
MCV 80 fl
MEHC 32 g/dl
LFT Normal
Stool Normal
in
should be carried out and
1. Whieh three other investigations
what order
a ESR
-U-oo"t
; GIT endoscoPY with bioPsy
I Eiio*vsealantibodies
; sioor for reducing substances
e Barium swallow
f Dental check for Candida
t T[i"tt swab
the risure berow?
z. hmPrt.liLtt""::*ifii;' are visisie in

a LYmPhocyteinfiltration
; io:taivittousatroPhY
I Pt.titt villous atroPhY
d Normalvilli
I Granutomatouschanges

3. l-.f Hl'"'ffIi,,*",, di as nosis?


Ouestions l-i0 5

Case 5
A 6-week-old infant has a history of persistent cough and wheeze for
1"9-?V., He is totally breastfedrnd feeding,has become increasingly
!.itrigytt in last 3 days. There is no history of whoop and he is afebrile.
His birthweight'ivris 3.5 rerald he now weighs 3.g kg. Flis older sister
had a febrile seizure at the age of 1 year. The family lives in a two-
bedroomed flat and his father works as a builder. There has been no
contact with anyone who has sinrilar illness and there is no history of
travel abroad.
The infant is active and has a Sat level of 92% in air.
He is using all his accessory muscles and there is no stridor.
Crackles were heard just before his cough and at the end of
inspiration.
1. Which two abnormalrtes ap.pear on the CXR?

a !1ight upper iobe collapse


*' ["{yperinflation
I r:rrc*sed [:r* n c]i ia i a i rv,rai" i h itke g
n i r-l
,I L.eit ui:g:er lt:be eo!lapse
i".arge l'leeri.
!-"i'eg|ilr'e i"ii:S --i Arl,; i:;
l.i,r*iiei: ceviated .(* rigl-r:
iI;,;i]t micidie iebe evsti* fe,:.r*n
.' 'i.rl a::,:t' i',,,,:i:i,r illli..i\rdf l-1g.9,.ii.,;r-:l;rl:,.ir j-.',:li.t;l;;:Ea,j
i]i-i:
:+,'fi*i Stfrsr test rc*,1;it* ar-r:
6 100 Grey Cases in Paediatrics for MRCPCH

Blood gas pH 7.29 : 4':--:9.!rec


ftco2 5.2 kPa e Spina brifiOa
ho, 9 kPa f FereOtrtary s|tf,rD
HCtr 17 g Dlso'nie BJncf
BE -1 : -f, 3-ee .--e- n
rcrfar.rneC
2. What is the abnormality on the arterial blood gas? a *ens.Ian @si
5. what one test can confirm the diagnosis? c iMG
a Sweat test : itr'CS
b NPA : $ ie hnm*
c Pernasalswab E 3.:rr*a Y6q;
t 3.:,.te
d Bronchoscopy and bronchial lavage X-,-a. fon:
e Chest CT ; S(- a,rofsr
t Ciliary motility study j tlrLaf -ara6p;-rg"lt

Case 6 'i"rst .r

ln a neonatal unit. there is a 13-day-old baby girl with ffoppiness,


oitticuttv feeding and ioint contractures- She was born following an "i. '!-1r4g1-3r: Sfir ip1glg,
easy pregnancy at 38 weeks' gestation by lorer sggment caesarean
ftrnr *aE iet -gmr. y
rrl:flnhcitrE'aft 3B$Bc r.
seciion d]le to i previous caesarean section. The baby eried i
i-*"Oi.t"fy but half an hour later she became ey"nosed and had -lC IAr,' rE$ r€i-r ff&= i
Ai*i.uliv inbreathing. She was admitted to 6'le s€ciai car€ baby unit i.tn-lrm6p rqr*re* -.g r

D@c mc .rnlne lEfis ltr


fSCAUt Lnd was veniilated for 8 days. Since ttren she has been fed by :
niiogastric tube (NGT) and has required oxl€pn via a facemask from Dr]lf mlr!trtm IM tf,'lrnqi
tirne io time. Her mother has had two miscsrriag€s. She has two
r.u.rlm@tlr- ir lrmre ES $ti
,mrftnri$tfiilF{ -e sieu: :iuei,
older sisters, who are heahhy, and there is no history of
conianguiniiy or such an illness in the family. fuiother sister died at e-lrm. TG lGEirmc s-n:umrq
16i ;;"";3 months, with a seve19.chgst infection, and presented in a e firG 3lltrtsr.tr/,ffi$ sflfc 1)ffi.1

*or""or"re state than she did. This sisler required ventilation for 3 mtmmfuc rnmrq ffrarSEE
*""tr, and was less alert, had ioint contractures, clenched fists, flm]fltr|tr m**trrp*lq.mrudl. rrrr U
and she tlru mummmq" &fsr tle
t ii"rii". and was grossly floppy. Treatment wre withdrawn uni

died from chest infection.


w @ryrmiltc rm@ ,r;a!r
, ffiilIG r6i|iw lttt :.|ms
On examination, this patient's occipital-frontal circumference
(OFC) is thnm,tuuu'md
on tf," grO centile, anil she has short legs with cofltraction at the hips imllllre.Id fiilrirmD" dltlrrullr
fn"e joints. She has low-set ears, a large_footanelle, prominent ffiG.trur.lr0irrryr
"na f irtrie and is less hairy than her sister. She is quite floppy, alert nnitl*f,@milrt
it,n" i" awake, has no tongue fasciculation, and the gag reflex is
"fuf
33
pr*"nt. There'is some griviry^movement in her limbs and reflexes 'qtth
ffi@
gur
5r" pi"i"nt but not Uris[. The CK is 40 mmoUl (normalrange: 4240 T,r TN
**bVt) and a metabolic screen is normal, which includes lumbar ry
.JTP
@*10
;;;;il;" (LP) and white cellenzymes, and there is no chromosomal -mn
abnormalitY
m ffiilr mnr
rui" m [J]tl
An MBI scan of the brain was reported as normal and limb X-rays fllh th MUI
no bony abnormality. other systemic examinations are normal. 'k !m [*.trr
"tro* flb lrgd
1. What is the most likely diagnosis?
ffi*uru
a Congenital mYotonic dYstroPhY mm
$ilm
U
&,nrir
rmnii
b SPinal muscle atroPhY
c Cbngenital mYasthenia gravis w f dmr

I
Ouestions 1-10 7

d Arthrogryposismultiplexa
e Spina bifida
f Hereditary sensory motor neuropathy type 1
g. Dysplasia punctatae congenital .

2. List three further investigations in the order that they should


be
performed
a Tensilon test
b EMG
c NCS
d Muscle biopsy
e Cranial MRI
f Bone X-ray for bone density
g Skin biopsy
What management would you recommend?

Case 7
A.13-year-old boy presented to A&E with a history of abdominar pain,
which was felt mainly on the right side of his, abdomen. He is a keen
footballer and played a game oi footbail 2 oays ago for about 2 hours.
H.is p.ain was very sharp and radiated to his gioinlNo vomiting
or
diarrhoea occurred. The surgeon diagnosedlppenciicitis and is[ed for
blood and urine tests to be carried out. ue was admitted overnight and
put on the next morning's operating list after review by a consuitant
surgeon. His urine test showed a RBC of more than 20b and no
leukocfres. He slept overnight after being given ibuprofen but by 4.00
a.m. he starled shouting.as_the pain got worse and was now spreading
to his shoulders and back. Review wis made by the surgeon, who
prescribe.d more analgesia, and a request was made torlne
boy to
remaln nil-by-mr:uth in preparation for a possible appendectomy by
the rnorning. After the ward round, an US of his kidneys and abdomen
owas organised, and it came
back as normal. The paediatric team was
asked to revievuthe case, as he starlec corrrplaining of spasnrs in his
Negs, arms, shoulder and said that they were painfJ!. He'was
very
irritable and jumpy vuhen touched or talked to" He was started on
antibiotics and acyciovir. and a bra!n cr scan was organised, w[:ie h
can'ie back as normal. even wlth {:ontrast" Oilrer.test iesulis are:
i.{ ir 13.3 g/rjE
12 x 18ell t[\ 82%, t_ 396i
i}LT tr 5C x 10s/l
! nrr
L.i-r f'l 15$il nlmolli
t" F. 6850 rnmrol/i
AIT
r50 tu/l
Atk. Ph s*CI ru/i
vGT 200 tuli
Atb 37g/at
Eiiirubin 37CI prn'roi/! (conjugat*C i57i)
3[J rnnrlhour
.RP i3 after 2 ureeks
t-
I
I

8 1OO Grey Cases in Paediatrics for MRCPCH


-5f,
was.getting more. agitated' r"* tj ria
The muscle spasms worsened and he r
helped and he improved' All his - t-?
;;;-r;;; ;n[ ct tormJinia-zote
disease' i:
viral serology whs n"g"ti'": Mycoplasma titres' LymeHe
Brucellaserology, ,nZ Gpto"pirosis
lgM antibodies' is :*
tr'" ise o16 vears and a BcG
ilily;;;;i^.-t"a-, *itn-uoJ"i"ti 't in the last 3years and no one
*{hfr t::t'E rrd:gf ilirgi,
r ,J"i "g"" He hasn'i ueen auroao the presence of RBCs' His LP
i S|rl|frr( fgTrlss
is ill at horne. Hls urinl-stiitino*. : '-:mnn lcrnt rtri
;;;;; no e'"n PCRs for bacteria and viruses' : --ffil[rt mElrcrar
"unotrn"tiiies,
and caet-utoplasmine were negative' Screening for [ ]a*mr Ii.rnSTf,fIT€
Tests for ANA
*" n"b'ti'e' level was also found to
The lead
;iphr-1*"iiirypsin t l,.r$,r: fOrcS lf,
be normal' - :JmqErnrJB ffiae,r
1. Which other three tests may help the
diagnosis? g fomrqun'rc iu
fiflfnrcrl trr ffir nlrlill
a Brain MRI
b Muscle bioPsY
c EMG hT
d NCS
n"p""t viral serology for EBV' parvovrrus "rlr trtr,rilfr ry.sremmt il,'r]r &
f" RePeat MYcoPlasma titres flnMni@llE 1g s'iii:"d{i : {rl
o DNA double strands UT(-'ml" fr8:lmgn"rf &jifii X *:
f, Utin" toxicologY r$h, 569,& -irs g|iBT ar-t =
2. Wh"t ,r" three diffLiential diagnoses? rnr,Glflxtr m;,[ TTa.':i* ]5 5 f
a Viral hePatitis i,:arge: u-ffi* tu'rs -:
b LePtosPirosis rm rlnrqrrir-uffruqBr,; ln]rm--
c Tetanus riitl -:
-ti! -
+'ll
d MeningoencePhalitis ,[$rlm
e Myositis ,ll, rt
f Drug abuse qs"
o Pancreatitis
t bv"t"ri.lupuserythematosus (SLE) lwr
Eru grum -"1*].,;'rr
'i p,q.
$fl{t ft&
Case I ffis -,I -$?h
r

a1d-Roolweight gain
An 18-month-old girlwith a history of vorniting
wetl tneiast 3 months and
" iwil'fiir Mllil'B,,' DrI*'t{s:
was seen in A&E."Sl"ie has not been very
i-n
iarteO to vomit and was not t -ffmrG TSr Ir-Ir]E,I
in the last 6 months, after a URTI' she gestation.bv-!:9-q^ ! -rnm&rtrmrrfE S"ii"[

interested in t"eoin6."ii" *"" u6* at 36 weeks'


: Smrryrr]cm t+s
*"i.
;;;;;iwJi"-nio.]ir,"r".n" no other concerns. she is hvpotontc ! qt6 a ,1u:i&r-:
and her mothercJii ir,.t hr. found it difficult to cope in the .*F
hernap.pies and the - 3'@ rr'- n'&*::r
last 3 month" *itt'in"'ir"qrln.v of,changing practitioner (GP)
i;;;;;;;;i i't'rins' Her s;neiar g "rrumrrq r*s:
"irl;i; 'h" (BS)' which was 4'5 mmol/l on three
checked her OtooJsugar t ]t[u$ r,!ilar;s ;i
There were no
occasions. rne ,eniiJcan shows nephrocalcinosis. : ill{l#Jt !flq "lirru* I-.r:l;.
in the family who has come
other illness". ,nilh"ls the first child t &,1trtrtriotttt*,;ffi.: tir
from north Africa. l, $l',TTrn,crtl!1 .fl 1

is on the 1Oth centile' having : 4l(1Tli1r5'$-li -r*


The child looks miserable; her weight
it moieratelv dehvdrated 3nd her U }.Umr,mg r.Edr.
dropped trom tn"'i6ittl5n" urine is clear and there
with no organomegaly' Her r @;."4 {":lr"'ir'Ir
abdomen is soft,
been carried out in the past' - .l,rtl/,CE iT i *'
is no metaboti" ;;;i;;-,
Other test results are:
"r"Jri".tinave
Ouestions 1-10 I

pH 7.56
EO, 4.2kPa
rc2 10 kPa
HCO3 33t
Be 1

1. What is the most likety diagnosis?


a Pyloric stenosis
b Protein cows' milk intolerance
c Lactose intolerance
d Bartter syndrome
e Cystic fibrosis (CF)
f Congenital adrenal hyperplasia
g Nephrogenic diabetes insipidus
2. Which two tests will help the diagnosis?

Case 9
A child presented with a history of recurrent wheeze and ear
infections. He is now 2 years old and his wheezing episodes are
under treatment with a regular budesonide inhaler and Bricanyl on a
PRN basis. His ears are red and his left ear has a discharge. His
mother said that this is the eighth ear infection he has hid this year.
scattered wheeze was found on examination of his chest and there is
no organomegaly. Other test results are:
Hb 13 g/ot
WCC 20 x 1Os/l
N 70%
lsG 7 s/l (6.5-16 g/l)
lsE 199/l (< 2 g/l)
Ear swab Pneumococcus
Sweat Na 15 mmolll (40-60 mmol/l)
CXR Hyperinflated with bronchial thickening
1. What other one test should be carried out?
a Urine for protein
b Lymphocyte subsets
c Skin prick test
d lgG subclass
e NBT
f Stool for elastasis
g Hearing test
h DNA linkage study for AS08
2. What are three appropriate steps in managing this child?
a Prophylacticantibiotics
b Bronchodilator
c Antihistamine nasal spray
d Grommet insertion
e Regular lg transfusion
f Advice on allergy
10 100 Grey Cases in Paediatrics for MRCPCH

o Referral to dietitian
dtrna b Pleural flukl cltt
t negutar inhaled corticosteroids for his c Lymph nod€ b{€
6. a Steroids
case 10 F
b NSAID

A 13-vear-old child with a history of fener and le*fiargy subsequently Systemic luprr crythrrrr
was no history o( travel abroad. Her
?"r"rli,"o-."rit ui"".s. rnererecently
father is a heavy r*oi"i, has had a productive.cough and is Sl-E is a muhisysterr c S
;;;; inuestigaiion. Shesaid she feets hot allthe ilme but has no Si can be presen:ec rt.-
;;ffi';; d;ioi *"iJnt. There is a tender, paintut srellins.on both n gh index of suspicrc-, s

lower limbs tf,"t 2 days ago' l-buprofen helped a.lot' but the pres€nt wilh muftionca-
"pp"""t"d stitt there. Her throat is mildlv
!*Liii"i,'*r,i.tt robii. iite bruises, ii 1:10 -- € com m, cnes.: c-E_s€^:
;;;;;;6i-"no a rvrantoux testwhh 0o0 and 1:1000 is negative.
";CA 5gn -ashes va-, -:
Thecoeliacscreen.noesnarealsonegative;theESRisonly2.She
opens her bowel -:.c€r.a o: lust ea-e-i
started to menstruate S ;onttts ago, irrEgularly. She f"-a:-.es varf frcr .e-.e
r"gJi*V and has had no previous health problem' t, :,:,i pressl re" frllaca -:
'1. What are the three most likely diagnoses? et-sc:: a!.e g:ler fea:_-r
a StrePtococcalthroatin{ection ;6r:;;5i6rn ar6| 5gi.-;r.Eg ,
b Coxsackie virus A in{eetion ::e :,4-.asis or S.E
"e-,
c Coeliac disease 'lef--36s 'css !r,,-s_3 s,i
sr6gr.5
d SLE Or.:CeS rS a i,:ie*
3l$:.aefs (€ 1f'".3rgt*l rTrS
e Crohn's disease
] -:riri6cril,pBnr€ e.-_: rl
f Ulcerative colitis
g Tuberculosis r.t--artbcd€s 3-=
6 HetPes gingivalstomathis . . .
what ]r.f. r^fu: S -Or?=€s: S,:a
2. Wf,i.n two inve"stigations should be carried out and in :laqnclsr.lg SL-E r-s :i
order? nir *rF!F!el|-L ctdc=cr =

a CXR .'F lfrEE=Sa -- - -


b UPPer and lower GIT endoscopY qscfmr'rE s"tr 3s - {t
c Throat swab rlmfi Urglfr tctliil -:'r
[ar6]ij!
d ANA SEmgrit @re$Gr," ,@!
e Gastric asPirate for cuhure
f ASO titres
ht
g MYcoPlasma titres
llll c hdfiDtrwi
bm
ANSWERS 1-10 dtre
fim
Case 7 I ! Igr-rcr.
1. a Blood film t. lffin
b ESR h
c MYcoPlasma titres fiffrrf,ru
2. a i.v. antibiotics
b lbuProfen ffri- rr-r
3. a Juvenile chronic arthritis (JCA) 5G-dDlr"mm
b MYcoPlasrna infeaion ffihmdi
c Viralin{ection ffiffirrffi
4. cxR nffihlh,r
5. a DNA double strand htrdbtnrhiir
Answers 1-10 11

b Pteuralfluid cpology
lvmph node biopsy"
6. a Steroids
b NSAID T '

Systemic lupus erythematos!,ts


ir a multisystem disease with many manifestations. patients
:|:
DLtr can De presented with many different symptoms and signs, and a
with
high index of suspicion shourd 6e reptln mino'for puG"t.
present with multiorgan involvemeni.
iho
,,I"^.:lTTonest presenting symptom is joint sweiling and tooks
iike
rasnes vary from.a butterfty rash on the faCe to purpura,
:.:1.^:,:i
unrcana or Just erythema of the hands and soles
of feet. otr,ei
Teatures vary from renar invorvement with gromeruronephritis
to high
ure. M yoca rd itis,-ca rd it s. p teu ri t eff u s o n a-n O pe ii ca rO ia
!L::_9^o::.:
erTusron are other features of presentation.
i i r

cNS invslvement includes


confusion and seizures, with focar neurologicar deficit, wnLrr
mates
the diagnosis of sLE very difficurt in some-cases. Aseptic
bone
necrosis, loss of visuar fierds, rymphadenopathy and
fingers or toes is a bizarre presentation of this <iiseasJ. f angiene- of ' tr,e
giooJ
disorders like haemolytic anaemia (DCT positive) and
thrombocytopenia with lymphopenia ca,i atso be manifested.
Autoantibodies are positive, incruding ANA and anti-doubre-strand
ur\A, wntch ts more significant than any other autoantibody
in
diagnosing sLE. The c3 and c4 revers r'r" t* and indicate renal
involvement. CorticosterqiO:.:l be given at a dose of t_i
with analgesia for pain re!ief. other d-rugs can be added, if
*glkglO
thJie is no
response, such as azathioprine, and cyclophosphamide
io,"
with proven renal involvement. Tne piojnosis is not cterr, "f,itOr",,
general children do better than adults. - Ort in

Case 2
1. c Renaland Doppler ultrasound
b INR
d U&Es
f FBC
2. f Right renal vein thrombosis
3. Hehydration
Heparin
Regular check of area and electrolytes

Right renal vein thrombosis


ln acute..renal failure, renal vein thrombosis should be ruled
out,
especially in neonates of a diabetic mother, on" *itt HiE,;
wno is
polyrhythmic and dehydrated. This is arso true for
chirdren with
cyanotic heart disease or septicaemia. The tioneys witi
ue eni"rg"o,
there will be gross haematuiia, origuria *ittr a row prateret
tount ano
12 100 Grey Cases in Paediatrics for MRCPCH

a fall in haematocrit. The renal function will be abnormal on a DMSA


ryGil vcrl qti:r\ arc r'
Srf.pEnar rith fu[ or pnr
scan and there will be no uptake of isotopes. Treatment is usually r',8 ftlt *uil rd ffi
supportive, which [pcludes peritoneal dialysis, if necessaryto correct 3 rirrr7gpod, *fnrg {A
congestive heart fdilure, electrolyte imbalance, and intractable I* rnaf to benrV it@
acid-osis in cases of bilateral involvement. Heparin can be used in
severe cases, but its use in mild cases is still controversial.
-ncredt*able end r corrr

CGr {
Case 3
' : E-occ:rp e-e
1. c Right and left knee X-raY ! U;.€€r GlTenocq
2. a Guillain-Ba116sYndrome - rg amd ffi s.,&i
c Myositis :. : Tel viltonls me
f Hemiplegia (stroke) a Lrnpttcfe lmffi
3. Perventricular densitY on left I Gr,en-serrsithe ae
4. CK
LP Ghtuii'fl.{irrc GIE'OF
lncreased white matter signals around artrerior hornes of lateral
ventricles on left and right -trurre rlc -ei\ a.Eer o
5. ADEM t-rirnE :f&;T6 rizr rrig
Demyelination -1E prGrffi6gfoa ci :tel.
Myelination in newborns is not formed until later in infancy. mffi u+,d$y dur.iirq i!-ift
nntrc e r.cr.ni-y rre otrE
Demyelinating diseases in newborns are very difficuh to diagnose until
the end of the first year of life. The MRI scan is very good for: picking up "rri.Efi nlS3-l.. E r{Eft t.
changes in whhe matter. There are few demyelinating diseases of aTU=Er-r.fle r:
childhood. A number of condhions affect the brain with demyelination, -
ICrSilng a )ffi c' wr.T'nE
including ADEM, MS, and leukodystrophy. Transrverse myelitis and Ttr6lersDtrliior. : a {ar:
Guillain-Ba116 (GB) syndrome affe€l the spinal cord. -eSng *o,n a:r--6liliadr r
rue s:ihe screer iri fitl.lE
ADEM is a version of MS in children. lt is usually preceded by a ifo -re Dresere C a -tl
viral-like illness, and symptoms can then appear as falls, hemiplegia t rctrr |lllgh susoreor :f I

and progressive limb weakness without sersory involvement. I3rrfrr--€c q .e1""tto


Cranial nerves can be involved earlier or later. The upper motor gruMil"Ee d6- [ ba ffir =r
neuron signs are present in the early stages and bulbar palsy is Aorrr+'ee dg- rttr rt
common. Children may need protection of their airways at an early
stage. Fulltesting, which includes neuroimaging and troir clrtItnra dsee b u
neurophysiology, is important as well as LP. A high dosage of i.v' uf ,mroibmsns on cpflr
steroids (methylprednisolone for 34 d) followed by oral tEq rf,fi shclft ryrryt
corticosteroids for 6 weeks, withdrawn slowly, is the treatment of nfuim ani de# srrr
choice. There is no place for lVlG or plasmapheresis. The prognosis is emtltry'@ rrrl lous et-4t11rl
good initially if there is no family history of MS. Children need to be
iottowed up regularly, and neuroimaging can be carried out every 3-4
'Ge'ErC fr;in clP amil

years if there are no symPtoms'


MS is usually a disease of young adults, and children can be affected h5
as early as 2 years of age. Presentation usually follows viral infection b Hyperi-fanm
with atlxia, but other features may appear, such as encephalopathy, c Srcreasee lrisr[E
hemiplegia, or even seizures. Diagnosis can be made by recurrence of Co'mpens^aiec "zsem
these symptoms with other features such as intraocular E \PI
ophthaimoplegia, which affects one-third of children. The symptoms
Answers 1-10 '13

appear very quickly and remain for weeks or months, and then
disappear with full or partial recovery. Children are usuaily tethargic,
and limb ataxla and brisk tendon reflexes are a common finding. MRI
is very good at picking up low Sgnals in white matter, and contrasted
CT may also be very helpful if MRI is not available. The prognosis is
unpredictable and a course of corticosteroids may help.

Aase 4
1. c Endomyseal antibodies
b Upper GIT endoscopy with biopsy
h lg and lgG subclass level
?. b Total villous atrophy
a Lymphocyteinfiltration
3. Gluten-sensitive enteropathy (coeliac disease)
Gluten-sensitive enteropathy (coeliac disease)
There are rnany causes of mouth ulcers, which include idiopathic
causes, trauma, viral infection, inflammatory bowel disease, coeliac
disease, and gastric ulcers.
The presentation of coeliac disease can be at any time of life but
most usually during introduction of solids in the first year of life.
There is usually one other member of the family affected and a
detailed history is very important. The presentation is varied, from
failure to thrive (FTT) in the early days to !.ecurrent mouth ulcers and
passing a lot of wind with anaemia. lt is usually associated with
malabsorption, in a form of anaemia and stool loss from time to time.
Testing for antigliadin antibodies as well as endomyseal antibodies is
part of the screen in children, with FTT in the first or second year of
life. The presence of a high level of endomyseal antibodies indicates
a very high suspicion of coeliac disease. The diagnosis can be
confirmed by jejunual biopsy during illness or after changing to a
gluten-free diet. lt is a life-long condition and patients should be on a
gluten-free diet, as the risk of intestinal lymphoma increases in
people who do not adhere to this. Another complication associated
with coeliac disease is occipital lobe calcification; whether this has ltr

any implications on epilepsy is still not very well recognised. A


biopsy will show complete villous atrophy with lymphocytic
infiltration and depth crypts. The other condition that may cause i
L

complete villous atrophy is tropical sprue. Partial villous atrophy is


associated w;th CMPI and giardiasis.

Gase 5
1. b Hyperinflation
c increased bronchiai airway thickening
?. Comperrsateei respinatony acidosis
3. b ndPA
14 100 Grey Cases in Paediatrics for MBCPCFI

Eronchiolitis
Bronehiolitis is one of the illnesses that affeets infants and pea*:
bet{ryeen octoberto March each year. some affected infants rr,:
need hospital adrnisslon and suppont, including exprernature '' '

with e!-D; those yeiurrger than 3 rnonths of age; and infants wi:'''
heart, l{:ng, anel CNS diseases. Eetween 1olo and ZYomay requ
adnrissrorito the F$CU but the rnajority of bronchiolitic infants :
eome to i.rospita!. l"here is no v€ccine but n'ronoclonal antibod : "

given as i.m. iniectians fer 5 months starting in September ea:-"


f-or pr*n:aiure trabies with CLO or infants with heart, h'lng and - -
diseases reduce the risk of gettlng RSV bronchiolitis as lvell as
risk of adrnission to tlie Plcl.J. supportive treatment for infants
admitted to lrospital is reqr.rired in the form of feeding, oxyger :
eare. Bronehodiiators have no rnajor role in treatment but the':
harrn in trying thern. Cross-infection is cornrnon, and hygiene :'"
as handwashing are vital to prevent this. NPA. is not needed if
condition is clinically indicated to be RSV bronchiolitis but mui rr"

done for infants with CLD or heart problems, or those with an


i m m unodeficiencY Problern.

ease 6
1. a Congenitalmyotonic dystrophy
2. b EMG
a Tensiion test
d Muscle bioPsY
3. Poor
4. Supportive
Genetic counselling
Palliative care

Myotonic dystroPhY
This is characterised by the association of myotonia with a dYs*
process of muscles with various endocrine and musculoskelet:
abnormalities. tt is transmitted as an autosornal dominant disc
and if present in the neonatal period is always transmitted fro.r
rnother but, ln the adolescent period it is usually transrnitted f r:
father. There is an expansion of a cTG triplet repeal associatec
the gene for myotol"tia protein kinase. The length oT the"repeat
incrEases in successive generations, especiaily when it is rnate
transmitted. There is a strong relation between the length of th:
repeat and ctinical severity and age o{ onset' The clinical prese-
is variable, frorn genera!ised rnyotonia to prominent weakness
Myotonia can be seen by tapping the thenar m.uscle or tongu:
thumb wiil remain opposed and the tongue will remain dimple:'
Relaxation of myotonia can be demonstrated by shaking hands
fatients. Atrophy begins on the face and" is followed by the shc
girdle and leg musclis. Smooth-muscle involvement may be,s'
iecreased ga]strointestinal motility, and constipation is a well'
Answers 1-10 15

reeogniscd feata;re. ln adult patients there may be baldness, IBDM,


cereija* ffilyopathy, testicuiar atrCIphy, and peripheral nerve
lrivolvement. lntellectuel impainrnent is very rare. The eongenital forrn
of myotonic dystrophy will us,gally start during the prslnatal peri*d
with hydramnios, SGA, hypotonia, weakness, and arthrogryposis
with joint contractures. The baby then beconnes vcntilator-dependent
and dies within the first year of life. Myotonia is never obsenned
before 3-4 years of age. Babies who survive the necnatal period are
often rnentally irnpaired, with bilateral ventricular dilatatisn and
macrocephaly.
EMG is very helpfulfor children abcve the age of 5 years, but
diagnosis can be made by clinical as well as by genetic study.
Clinicaliy, diagnosis ean be rnade by shaking the rnother's hand"
Prenatal diagnosis can be made by e hronic villi biopsy to look for the
mytonic dystrophy gene. There is no truly effective treatment, but
nifedipine ean sometimes be helpful.

Case 7
1. a Brain, MBI
b EIMG
c Urine toxicology
2. a Leptosirosis
b Myositis
c Drug abuse

Musele spasrns
Muscle pain following exercise is usually secondary ts loss of fluids
and electrolyres. Stretching may reverse the muscle spasms. Other
spasms that are not related to exercise may be due to various ca!!ses:
electrolyte imba!ance, hypothyroidism, hyperthyroidisrn and
hypoadrenalism are the main causes. lnfection such as tetanus, whieh
is very rare, ieptospirosis, and strychnine poisoning may also cause
rnuscle spasms but are rare causes. The metabolic causes are many
and this can be diagnosed by rneasuring lactate, NH., and creatinine
kinase, and any abnormalities of tl"rese should help in reaching the
diagnosis. Muscle biopsy and EMG are very important in reaehing the
final diagnosis.
Pain rnanagement secondary to muscle spasms can be very difficult
in children. Llnresponsiveness to appropriate use of analgesic agents
might be in addition to physical or psychological dinrensions of the
pain thet are not addressed by the analgesics. ln addition to
appropriate analgesic therapy, the psychological necds of the child
should be addressed directly and appropriate adjunctive physical
rnodalities employed. Benzodiazepines can be used; even if they do
not provide direct analgesic effects, they can reduce the distress
associated with acute pain states by decreasing anxiety, insomnia,
and rnuscle spasrns that can be associated with acute pain.
r 16 100 Grey Cases in Paediatrics for MRCPCH

Muscle spasms can present as seizures or a jumpy baby in the $fidass measurern€rfi sf]
neonatal or infantile period. This is called hyperkeplexii and it begins rtibody response ard a I
with generalized hypertonicity accompanied by brisk muscle stretch prophylaaic co-tri rnoxat
reflexes, intermittent clonus, and an exaggerated startl€ response and rrnall of group of ctrildrer
it is benign. Anticonvulsant drugs will not be very helpful. Spasms inrmunisation with SG srr
can be familial or sporadic. Nose tapping in infants of affected *mmunoglobulin every #
families induced a uniform reaction of facial twitching accompanied school and after sctrool h
by head extension, and a generalized flexor spasm, all of which may viral infections. Sorne rna
be a hallmark of hyperkeplexia. Small doses of benzodiazepines will most often betrleen the r
almost always show some benefiu all affected individuals becoming deficiency problern arrd u
asymptomatic by 2 years of age. There is no need for further undertaken to reassure f
investigations if a positive family history is suspected. needed.

CaseI Ctrc l0
1. d Bartter syndrome 1. a Stre$ococcd tfu
2. Serum K, Na e Crohn's cllsere
Bartter's syndrome is a familial disorder characterised by activation
b Cosactie Yinls,r
of the renin-angiotensin-aldosterone pathway in an attempt to
2- f ASO titrer
recover sodium chloride from the distal nephron. This causes an
c Throc[ sd
increase in potassium and hydrogen in urine secretion which will lead Strcmocmtffi*
to hypokalaemic alkalosis. Affected individuals usually present with
FTT, poor tone and lethargy, poor feeding, polydipsia and polyuria. lt ArionS tf- qrscs otf rrrc
may also be associated with developmentaldelay. Hypercalciuria and infucfirr. T
,ireii as virC
bone demineralisation are due to an increased production of firud ir*ettions. Ba-tu
prostaglandin. Nephrocalcinosis is a serious and early complication. group A are nnore so tha
Potassium, sodium and chloride supplementation is needed as well !c,4f,rlr the infertion- A t
as regular renal scans. Prostaglandin inhibitors such as indomethacin tc,d^.s,i€r and posteri'o,r pt
are indicated and angiotensin-converting enzyme (ACE) inhibitors i nked rn rn un oa
enrV.,'ne-l i

(captopril) will help. lf older children present with this syndrome, it is wH€h cirn be done wfrilr
usually milder and they have no renal problem. Gror.rp A strePtococcar at
Trea'ting red throats ls aii
to viral infe*ion- t-age,
Case 9 -i!
use the child
"rs
uqte{L
1. d lgG subclass be carried out Pronnfrfo
2. a Prophylacticantibiotics alter 214 hot"ln !f :he ,

d Grommet insertion haernolytic s,ESt@


h Regular inhaled corticosteroids for his asthma reccrnrnend€d to eradc:
always a Probl'ern, buc e
lmmune deficiency in children the throat swab b erEfi5
erytfiema ncdcsl$T-
Children with frequent severe bacterial or viral iltnesses and required
adrnissions should be treated with a high degree of suspicion for an
immune deficiency problem, which could be primary or seeondary.
Repeated lung pneumonia affecting only one lobe or segment rnay
indicate an anatomical problem. Multiorgan infection with different
organisms may indicate an immune deficienry problem. Cases of lgG
subclass G1-G3 deficiency usually present more frequently with a
URTI or lung infection. Many affected people live normal lives. lt is
commonly associated with lgA deficiency, and any quantitative lgG
it

ll
i

Answers 1-10 17
ti:

subclass rneasurement should be interpreted in conlunction with


antibody response and a clinical scenario.ln symptomatic children,
prophylactic co-trimoxazole may be needed over the winter. Only a
small of group of children whorhave a failed antibody response to
immunisation with lgG subclass deficiency require regular i.v.
immunoglobulin every 44 weeks. During the winter, the start of
school and after school breaks, children may be more vulnerable to
viral infections. Some may have up to 6-8 URTIs per year; this occurs
most often between the ages of 3-7 years. This is not an immune
deficiency problem and testing of the immune system should only be
undertaken to reassure parents and doctors, which usually is not
needed.

Case 10
1.a Streptococcal th roat infecti on
e Crohn's disease
b Coxsackie virus A infection
2.f ASO titres
Throat swab

Streptococcal throat infection


Among the causes of mouth ulcers are streptococcal infections as
well as viral infections. There are different groups of streptococcal
throat infections. Beta-haemolpic streptococci isthe commonest and
group A are more so than group B. A throat swab is impoftant to
confirm the infection. A throat swab that includes swabbing the
tonsillar and posterior pharynx is essential. Latex agglutination and
enzyme-linked immunoassay are rapid test's developed recently,
which can be done within 15-30 min on a throat swab to diagnose
Group A streptococcal antigen. Treatment can then be started.
Treating red throats is always a problern. Most throat redness is due
to viral infection. Large, inflamed, tonsils are indicative of antibiotic
use if the child is unwell, but if the child is well, a throat swab should
be carried out promptly and treatment can be given when it is ready,
after 24-48 hours. lf the culture is positive for group A beta-
haemolytic streptococcal infection, then penicillin V for 10 days is
recommended to eradicate the infection completely. Compl!ance is
always a problern, bL,t advice to finish the course as well as to repeat
the throat swab is preferable. The tender rash on the lower limbs is
erythema nodosum.
0{JESTI$f{S 11-20

Case tr'8

For tlre last 4 wceks, a boy a$ed t 1 years has heen seen im ar:
o.utpatient department with a history of cough and ebdmnrinai Erairr.
['{e is passing frequemt stools, sometinres cohtainlng rnucurs. l{is
appstitCI is not very good and he has lost sorne weight" $me of hls
s*hoolteaehers describes hir:rr es having loot his touih ancl becorne
very nervet}s. l-lis father died when the boy was g years old, ft:llowing
a cerebral haernorrhage. *"lis sister is IE years old and i* d*ing very
s.;eii. His rnother was admitted to hCIspital 1 year ago with a possihle
diagnosis of hneast eanccr. Later that year, a!! of hii rnothcr's results
came back as norrnal.
The r:ough is described as dry, mainiy occurring at night, and there
are no other symptorns. There is no history of travel abroad sinee his
father's death and he is a youth member oi a local football club. The
farnily lives in a three-bedroorned detaehed house and his mothcr
vuorks as a sales manager, fulttirne !n the last 6 months.

He is very anxious and is talklng with a low voiee, as he hed an


accident in his underwear today, while he was playing football. Flis
abdomen is soft, with mild tenderness in the right lliai fossa arrd his
ehest is clear.
ESR 70 mmlhour
Hb 11srut
MC\I 77fl
PLT 500 x 10e/[
I-FT Normal
1. what is the most useful bedside exarnination that can be carried
out?
a Blood pressure
b Urine dipstick
c Fundoscopy
d OFC
e F{eight and weight
f Elood glucose by multisticks test
g Rectalexamination
He was admitted overnigtrt and was coughing most cf the night. All
other observations are normal. There is i sn"ratt mark cn his bed,
whieh looks like mucus with blood.
2. What else can be done?
a Stool for culture
b Upper and lower GIT endoscopy with biopsies
c Barium swallow
d Stool for occult blood
e Perinealexamination
f AbdominalMRl
20 100 Grey Cases in Paediatrics for MRCPCH

irn fi B!€EririirL!6
g Check child-protection register fred md dl+d pl.?f- Sh
6 Arrange for forensic medical examination (FME) etp€rrsf,y**lodirc d
3. What are the three abnormalities visible on this slide? rylnrr*y-TfcCficncra
rEi gait re norrd- CT u
Ithat b the dagrEi
a ffi;rcfter*e gmm
h E1e nqdoflh
c Fs,rtfi rsl? pca
C TH rsve p&
e Ofric ranrrtirs
f Cervhd rmb
g hrtrroedr ryhd
LjmthrE poee& cr
a ldopdric
b 8€tt1rt irsr*a
: ffico@smrhil
o 8rr*heteb
c StE
f Trrmne
Utnr rcffic rlat !frr
drurtrbcrrhi op
a LymphocYteinfiltration r ftulfiegraini
b Goblet cells ! fu*rdtmd
c Non-caseatinggranulomatouschanges cffiA
d CryPt abscess d ft@mmntu-
d Ulcers r AludfrE
e Giant cells f hEc
f Transmuralinflammation 9mhrele
ffi
4. What is the most likely diagnosis?
a Crohn's disease Ifuffimrr
b Ulcerative colitis 1i utuffito
c Eosinophilic colitis f, (ffiqilrn
d Schwachman sYndrome
e Cystic fibrosis (CF)
I GIT tuberculosis bI
s Recto-anal fistula
Child sexual abuse (CSAi
f*p*rufruFm
h tdb.hdtu:
"fih&mihnqofrrc
Gase 12 ffi.hmmt
ffidUmd.I,tr
A 20-month-old child presented with a sudden onset of a dropped
;ight ;fu;, eyelid. There is no other abnormali'ty. There is a history of lpaobdmrft
re-cent hand-ioot-mouth disease, which occurred iust 1 week earlier. ffi.L-hrr
The family were camping in a forest for 2 nights, 6 weeks ago' Her ffil*dlcrrd
brother, who is 10 years old, complained of a headache 1 year ago, gifblr*m
which resolved after a few months. The mother has suffered from Cmn*Ldffir
cluster migraine but not in the last 6 years. Her father is a milkman :m !
und is dyjlexic. Her two older sisters are fii aird well and there are no
pets in the house.
w tr'mrmnll
Ouestions 11-20 21

13 a1 We.epmigtion, a drop@ right upper


eyelid was noted, with a
txed and dilated pupil. she is having difficutty wtur right eye movemenL
especially while looking down and to ttre side. There ii no iacial
asymmetry- The odrer cranial nerves a re intact. Her tone, power, reflexes
and gait are normal. cr and MRI brain scans were reported as normal.
1. What is the diagnosis?
a Myasthenia gravis
b Eye myoclonia
c Fourth nerve palsy
d Tlird nerve palsy
e Optic neuritis
f Cervicalrib
g- lntraocularophthalmoplegia
2. Listthree possible Gommon causes
a ldiopathic
b Benign increased intrrcranial pressure (BICH)
c Mycoplasma infection
d Borrelia infeAion (Lyme disease)
e SLE
f Trauma
3. what are the next three investigations and one bedside test that
should be carried out?
a Fundiexamination
b Borrelia antibodies
c ANA
d Mycoplasma titres
e Blood film
f Mantoux test
g CXR
h Blood pressure
i Visualfieldexamination
j Urine dipstick for protei4
k Check weight and height

Case 13
A14-year-old girl presented with a sudden onset of left-sided hemiplegia.
she has been unwellfor 2 da.ys, with headache, tiredness and rethargy.
There is no history of recent infection. she was ,ciagnosed as having a
heaft murmur, but no structural abnormalities havJbeen found
following ECHO and 24-hour EcG. A venous r,um was oiagnor"o.

Iheadache.
Vear ago her mother had a cardiac arrest preceded by severe
she was in a vegetative state wit'h coma anci massive brain
damage. She used to suffei from migraine tfrroujnort t,"', f if". in"
girl's test results are:
On examination Left hemiparesis with left facia[ weakness.
CBP 2a
ESR 50 mmlhour
22 100 Grey Cases in paediatrics for MRCpCH

LP Normal, including serology and oligoclonal band ' S,"cc, *,"rn irr
Clotting/FBC Normal
LFT and NHo, " *; 9r"m tesl
E rm tEE $*4r
blood gases, Ccnlm rgrrlcr
urine AAs and Skrn Em
organic acids I a'fi s ?q -r;g :i
Lactate Normal ! tr'4'tHl -,lik rr"l
(both CSF and blood) : laitm
Normal : i-rfiGmlxg:
MRI/MRA Show right ganglia infarction with evidence of right dllluq'*
posterior artery reducing cal liper ; ;*
1. List four possible causes ' lirynrnH sr.ml
2. List four important investigations I rru*. t:lLg c.nrr.l- St-r:
I &*nr:it-1131"e.!:
Case 14 : &;m*tur@nlng
I *r1g.:|,n*E
A 3-month-ord infant presented with a history of increasing -ril
of stool, up to 10 times/day for the rast 3 weeks. rne stoot is
frequency : ,*:::-Et-t? -r{
and contains mucus most of the time. on one oa.a.ion it
;;;"y * lurr
*"s-mixea - rfl[,Mlr lItsn;naf
with blood" His weight 3 weeks ago was 5.2 kg and it is nowi.d20
He is feedin-g very weil, every 4 hours on the Sreast,
kg.
r,i, *Ifl.,",.
; ir..rarlrumgle r,T
ha.d been told not to give him any dairy products foi "no
z *""[.. rt",.
this treatment, there was stiil no improvement. He has a runn\r
and there is no evidence of chesty cough or oiscorntort.
nose h,15
evidence of soreness on his bottom. HE is the onry chird
r["iJiu ,o j*rrr:;U-IrE 3@r rrc
.;ii
caucasian coupte, with no famirv history of irrnesJ. H; ;;;
oi; ;;;rg t
i"t[ -:r.e'Sj"*rris,tf" ng :*a
i,lrn*
term and had no neonatal problems. ", Tc4mT[ lllir:fiiwe r6Er -E
On examination, he looks happy and is smiling, with scratch
marks
ruwu
-h. .t ffirs r ,[
on his forehead. He has no rasrr on his body. iis auoomen is alTfirrnfl.nmTr(G[fm -Iqf,rr.}g
his chest is clear. His face resembres his mother,s, with
sJtt and fur,.r rm@mtf tG S[rT 15 S
an anii-ous
look. The test results are: i rme 1[ rqarE
-*maf,Irffi tltu ltrf,r.Ilflr: :r
Hb 12.5 gldl IrrU {'i*"rrmu{ilm.. -lru .lr.
WCCs 8.6 x 10s/l (N 50%, L3B%, E 0.4%) tlrrc $r r dl rr trm r{
PLT 160 x 10s/l € .liirnr.g.l,r.aln 1tu lror
Arb 39 s/t f lu 6ln fu Uauscr
Total protein 659/l :i':fflthrlI t f5bt
Ark. Ph re.
234 mmol/l ;ts ffidir l sra qr l@ f" -
Stool x 3 No growth after 5 days nul!ts S .nC Tm{finnru4r
CRP
X;f.:f
0.5 *,a!E EAEI1,[ &:i-,tru6,
ESR 10 mm/hour
Stool
' &Ltr r*CrqB E m 1rr
Negative
for reducing : 1rrgr
substances :3lF -rrr.
: Jtmur fiutrc
1. What other three investigations should be carried out? g agr',lElnGlrtr
a Stool pH Sr;EEre_(
flmE

b Hydrogen breath test -= E?ftrr


c l-actose quantitative in stool S &rilr.q: [ruilcriur=
d Endoscopy and biopsy ' fu,.r 665*
e Blood film q.m*m mc fi,urc
Ouestions 1t-20 23

f $teol for fat


g Sweat test
I, lg and lgG subElass
i Coeliac screen
D

j Skin test
2. What is the rnost likely diagnosis?
a l-actose milk intolerande
b Golitis
c lnfeaion
rl Food allergy
eCF
f... eongenitat enteropathy
J. What four.steps should yo, tate to manage
a Prednisolone for 3 weeks '- '---"-v' this infant?
b Azathioprine
C Hydrolysed milk products
d Lactose-fnee milk
e Creon
f Weekly weight
o Re-challenge with lactose and biopsy

Case 15
A 3-hour-old baby was found.by a midwife
the paeciatrician. The cyanosis rtrrt"J to be cyanosed; she celied
roon as the baby sraned
feeding but tlrere was no apnoea. r-ris ".
sat in air is 66% and on rOCI%
was 77%" r'{e was born at term and
a oron"t"r uoou"b;*
appoirrtment was reporteo as normar.
but siopped as soon as she tne*
Hi;;;il;;;;';;
"."n "iG iil""i ii."r.,or
ine *is'pregnant. she smokes
between 5 and 10 cioarettes/d ana
aoes n-oi use drugs. she is hepatiris
c-positive and her trio.ottrei chirdr;;;;;
iearthy.
and H,V-negative. The r,vperoxicie"iia-s'carrieo she is hepatitis B-
the sar was stiil in the hith z0;. Hi&p out on the baby but
45 mmHg from the umuiircat art".v
i. z-di+o mmug, with a mean of
*tr.,"# (uAc). purses are difficurt
to feel, even afrer resuscitation.-#;;;'i,;.ilbated
transfer to a specialist.cardiac and ventitated before
gas with a Pcv of 60 fr. The
H"'r aeidotic, on arteriar brood
""nir".
c-XR ;iil;;;;riness of the perihirar area.
There is no murmur but the iirst
was started and a cranial US was
h;;;;;r;; is faint. An i.v. antibiotic
,"poniO normal.
1. What needs to be done immediatelv? ",
a Exchange transfusion - -'---'' '
b cxR
c Arterialline
d Peripheral tine
e Start prostaglandin infusion
, Hart-correction of acidosis by
bicarbonate
g Arrange transfer to specialisi centre
h
i 3l1T l.y.. pntibiotics (cepha tosporins)
t{estr,ct the fluid to 40% of requirement
{or MRCPCH
24 100 Grey Cases in Paediatrics

Sedation with morPhine


iirriir. a"famine infusion 5 pg/kg/min
aortography to"h'elp
H"-*.. treated and 2 weeks later had an for his condition a-
"

corrective surgery
diagnosing anO pf anning
echocardiograPhY'
2. V/hat is the abnormality on this X-ray

with this problem r

3. Which three conditions are associated


a Marfan sYndrome
b Noonan sYndrome
c Turner sYndrome
d DiGeorge sYndrome
e Fetal alcshol sYndrome
f Wiliiarn sYndrome
g il*wri sYndrome

ease 16,
a history of pr:ssihle seiz"'-
A GF refcrrec! e 3-y*ar-old Eirl wiih looks frightenec
fruj"ioc '*'l.,**'u t*i*'' sh* wakes -up'man going" ]-he-
;;; ;'',ihe '

ssys, wt'rrte niarr-eo*ring an'J th* white h{.er :


tcr *opcts aid tupu'ts this manl: times'This
r,uu.r"l,ng p: -
iime th'ey are losed'
son'letifi'les open f:ut-ilott of tf'e
"lJnn e

for 3*4 nights per week


repeated up to ten ti*ut t*f''r' night
S

of four childrer
;J;';;Ji"i[" o' tr'''[ins uo*'''ti' she is one
Ouestions 1?-20 Zs

are we* and rivino iruith their mrother,


as their father Seft horne 1 year
1c:!Egi, stee"ps r-2 houns
ts compatible with her age andshe
Ji;irG;; daytime. Her devetos:menr
--,
'hyperactive girl' by her hotner.
h-as been-describeel ui,
The mother is quite anxious as the girl,s
suffers frorn epitepsy" a ge"neiJg;;;;;;,,ic on her mother,s side
uncle
abnormalities and the awate Ef C sfrowl examination reveats no
n_o abnormality, The FBC,
il,{g, Ca, Glu, and LFT tests
,f f *r*r'i. Sf," was treatecJ with
valproate up ro 49 "i*
mg/r<g/d, bui*r," i, .tirir,Lrirg'rnu'r.*"
loOjym
probtern and is now starting 6;uit";;;ff ih;-;;ij';_.
d;ri;g
1. What is the diagnosis?
a Nightmares
b Generalised seizures
c Complex partial seizures
d Sleep disorder
e Hallucination
f Behaviour problem
g Benign Rolandic epilepsy of childhood
h Landau-Kteffner syndlo;; - -
2' which three other investijations shourd
b6 carried out, and in
what order?
a Cranial MRI
b CranialCT
c Steep EEG
d Video telemetry
e Ambulatory EEG
f ECG
g Partial metabolic screen
h Urine toxicoiogy
Her asleep EEG shows some parorysmal
slow waves arising anteriorly.
3' What is the drug of choice that shourd
be added to contror her
seizures?
a Sooiurn valproate
b Carbamazepine
c Lamotrigine
d Vigabatrin
e Topiramate
f Steroids
g Tiagabine
h Levatricetarn

Case 17
An 8-year-ord girr was.admitted
to hospitar with rectar breeding. This
was described as,fresh btood,;;;;;;;;[ccasions
always after emptvinq rter-bowii:$;;;reated in the past and
her lower GIT colonsiopy. carried for constipation, and
uri fo"rfiv by an adult physician,
was normat. She is not cornplain,ng
ot ant iUO.o*in.f p.I;';;';;",
for I!'IRCPCH
26 100 Grey Cases in Paediatrics

has become weaker and weake' - --


svmDtoms. Her mother says she weight and looks pale' Her'
{a'- -

"montns. The gi'i';; l'"t some


iast b tuberculosis 7 years $o aoc 'r:
was diagnosed as navini J'r*on"vgeneral examinations show n:
treated successfully' S;;"tJ;i;
and
p"ti""'iitsion.[ there is no eviden':: "

abnormalities. The resurts are:


test^c1111.7nd
:;';;;iH;;i. ot oia scars' other
Hb 7.2 sldl
WCC 6.7 x 10s/l
PLT 450 x 10s/l
ESR 25 mmlhour
INR 1.5
PT, APfi Normal ranges
LFT, U&ES Normal ranges
Stool No growth after 5 daYs
AXR/US Normal
1:1000
ffil',IJ. t"" Negative with 1:10 000 and
1. What are the 3 most likely diagnoses?
a lntestinal PolYPs
L tnfl..*atorYbowel.disease
c ln{ectivegastroenterltls
d ConstiPation
e Mecket'sdiverticulum
i lntestinaltuberculosis
o Duodenal ulcers
f, H""*otrhoids
i Child sexual abuse
! Analfissure
z.'*n # ll"'"-t"'i"iiti u' p pe r o n th is-x-ra1fi 1]
I
a a

3. What is the diegnr:sis?


Ouestions 11-20 27

Case 18
A 6-mosrth-old infant presented with jaundice and FTT" He was bonn
at term and there were no ne6natal problems. Fle is breast-fed antJ is
now on solids. He is taking his food without difficulty. At the age of 5
months, he was seen by GPs, when a history of cough and a heart
murmLlr were diagnosed, and he was referred to hospital. Over the
iast 3 days he has become more lethargic, is not interested in his
food, and his colour is changing.
l'{is father has a valvular heart problem, but no sLlrgery was required
and there are no other siblings. There is an ejection systollc rnurmur
on the left side of the chest, mainly loud at the second intercostal
space. The liver is about 4 cm below the right costal margin, with
yellow sclera. Stool colour is normal. His urine is described as 'dark'
by his mother. His father has a long face and there are other
abnormal features on the spinal X-rays {butterfiy vertebrae}.
Hb 11"5 g/dl
WWC 'l1x 10s11 (N 657o)
Pl-T 320 x 10e/l
Eilirubin 265 mmol/l (20% conjugated)
AIk. Ph 660 ruir
ALT 12O IU/I
Alb 32 g/l
yGT 80ltJ/l
CMV lgG Posltive
Caeruloplasmine Normal
Hepatitis B,C,A serology Negative
US of abdornen Large liver vvith normal consistency and
no splenomegaly
1. What are the two most useful investigations that should be
carried out?
a ECG
b cxR
c ECHO
d Liver biopsy
e Hepeat viral serology
f Bone marrolv biopsy
g Muscle biopsy
h White cellenryme
i Urine for organic acids and GAGs
j Blood for AAs
k Urine toxicology
I Transferrin level
2. What is the diagnosis?
a Hurler syndrome
b Hunter syndrome
c Carbohydrate glycosylated protein deficiency syndrome
d Alagille syndrome
e Glycogen storage disease type 1b
28 100 Grey Cases in Paediatrics for MRCPCH

t Chronic active hePatitis


ct Pomp disease

Gase 19
A mother brought her 10-year-old daughter, who has a history c'
frequent cough and a high temperature for the last 6 days. Her c: -
has been getting worse over the last year, in spite of asthma
treatmentlnd courses of antibiotics prescribed by her GP. The c: -
is productive and has been consistent for the last 2 days.
She was borrt f ull term, with no problems in the first 2 years' Or
-

second birthJay she contracted a URTI and was treated with


antibiotics for 2 weeks, as her symptoms took a long time to cle:'
The family li..,es in a council flat and recently one child in her sc- -
was diagnosed as having pulmonary tuberculosis. The test resu :
a re:
Mantoux test Negative with 1:'10 000 and 1:1000
1-E)D t5
10 mm/hclur and repeated after one week
'1
ESH 2
mm/hour
WCC x 10s/l (L 40%, N 57%)
'10
Sweat test weight 110 bg (Na 30 mmol)
i V/hat are the abnormalities on this CXR?

a [{vperirtfiated right !un6


h [-eft upper lobe consolieiation
Tracheat stenos;s
d Foreign body or: right lawer bqonet'rialtree supplyir"lg
segrnent lobe
Ouestions 11-20 29

e Cystic changes on right lower lobe


f Bronchialthickening
g Bilateial hilar shadofts
h Right lower lobe segrnental consolidation in two X-rays
What are the next investigations that should be canied out in order?
a Bronchoscopy with lavage
b Bronchial lavage
c Cilia motility study
d ChestCT
e Lung biopsy
f Chesr MRI scan
g Barium swallow
h Transoesophageal echocardiography
i DNA linkage study for CF
jlg
k HIV antibody
I Skin allergy test
m Spirometry
3. Which two therapeutic steps should be taken for this patient?
a Chest physiotherapy
b l.v. antibiotics
c Lobectomy
d Lung transplant
e lVlG every 4 weeks
f Regular chest scans
g Overnight continuous positive airway pressure (CPAP)
h Genetic counselling
4. What is the most likely diagnosis?

Case 20
A baby was admitted to hospitalwith a history of apnoea and
possible seizures. He was born at term after a prolonged second
stage of labour. The Apgar score was 8 at 1 min and 9 at 5 min.
Mother and baby were discharged home 6 hours later and a health
visitor visited on two occasions. He is now 5 days old and breast-fed.
An hour after admission, the baby's Sat level dropped to 85% and the
-{}urse noticed a twitching of his right leg and arm. This continued for
about 10 min. The baby was transferred to a high-dependency bed
and regular monitoring was carried out. Various blood tpsts were
carried out and the baby was put on benzylpenicillin and gentamicin.
LP revealed no abnormalities. The baby carried on fitting
intermittently and he was given a loading dose of phenobarbitone
and put on maintenance therapy.
The next morning he had another 3-minute seizure, affecting mainly
the right side of his body. The test results are:
NHo 28 mmol/l
Lactate 1.9 mmol/l
30 100 Grey Cases in Paediatrics for MRCPCH

pH 7.35
ffo, 4.2 kPa
HCg3 21
BE 0.01
fr, 8.3 kPa
INR 1.1
Hb 17 gldl
PLT 270 x l9ell
CRP <5
BS 4.5 mmol/l
Mg 0.92 mmol/l
Ca 2.45 mmol/l
MSSU Negative
Urine toxicology Negative
Blood C Negative
1. What is the next single investigation you should carry out
a EEG
b Cranial US
c Cranial CT
C Met*bclic screen
., . ',.
: |t , lt :.. ::rf
r'.ia.,.ir (t..t..rt .l

tla-iLt, .', .'r il


ie i-;;:i"l.i i i-l l;.,.; i

ffi ffi
a Cerebral sedema
b Subarachnoidhaemorrhage
c Subdura! effusion
ct Srnatl latera[ uentricles
e Eiiated ventrieles
I Attenuati*n and {ow derlsitl, of left parietoternpora! lobe
g Sasal genglia cafaificatior:
Answers 11-20 31

h Brain atrophy
i Shitting of rnidline
3. Wnaiii on! single bedside clinical test that should be carried out
earlier? D
a Measuring blood Pressure
b Fundiexamination
c Repeated BS
d h/leasuring ternPerature
e Oxygen Sat
f ECG
g Testing all reflexes
What are three possible causes?
a Middle cerebral artery infarct
b Birth trauma
c Shaken baby syndrome
d lntercerebral bleed
e Hypoxic ischaemic encephalopathy (HlE)
t Neonatal seizures '
g Accidentaltrauma

ANSWEBS 11.20

Case 11
1. e Weight and height
2. e Perinealexamination
b Upper and lower GIT endoscopy with biopsy
3. c Nbh-caseating granulomatouschanges
e Giant cells
g Transmural inflammation
4. a Crohn's disease

Crohn's disease
This disease can involve any part of the GIT system. There is always
a delay in diagnosis of this iondition as presentation is not
ctraracteristic-but a high index of suspicion is needed. colicky
abdominal pain and d'iarrhoea with or without growth.failure are
f,,ghly indicative of inflammatory boweldisease, and investigations
sfr-suiO be initiated to find the cause. Blood will show iron
deficiency anaemia with a high ESR and CRP in the acute recurrent
phase. Thrombocytosis and hypoalbuminaemia rnay also be
present and are indicative of active illness" An abdominal X-ray
with barium may not be tolerated very well, but it will show skip
lesions and narrowing of the lumen, thickening and fissuring.of the
bowel wall and fistulJformation - all highly indicative of Crohn's
disease. The MRI scan is also very useful and helpful in diagnosis
if barium swallow or endoscopy cannot be carried out. A
tu.ftn"iiu.-labelled leukocyte ican is proving useful in follow'up and
diagnosis but is still in the early stages of development. The
12 i00 Grey Cases in Paediatrics for MRe FCH

diagnostic procadure is upper or lower GiT endoscopy withtic:s


itre-inftanrrnetlon is transmural, resulting in sinus tracts or fistu i
formation. Tfre bowel mlrcosa may look inflamed, and snnall she
apfithoid or linear ulcers r"nay be present. Deep fissures lead to a "
cobblestone appearance later on, as will structuring. The hallma
the finding of non-caseating epithelioid granulomata with giant
cells.

Case 12
1. d Third nenve PalsY
2.a ldiopathic
d Lyme disease
b BICH
3.a Fundi examination
b Borrelia antibodies
e Blood film
d Mycoplasma titres

Oculornotor nerve palsy (third nerve palsy)


Third nerve palsy can be isolated or associated with other nerve
palsies, mainly of the abducens cranial nerve. lt can be congenii'
and is usually'unilateral and complete. Only half of affected .

patients can be diagnosed during the neonatal period. lt can be


iamilial or due to trauma of the orbit at birth. Most cases are
idiopathie. The affected eye is exotropic and usuatly amblyop.ic
Brain tumours are among the many causes of increased cranial
pressu!'e and third nerve palsy. ln general, compression of the
oculomotor nerve by a herniated uncus produces only involverr:-
papillary fibres with unresponsive mydriasis' Complete paralysis :

il",e tf,ira cranial nerve may result in ptosis and'extrinsic ocular


rnuscle deficit. An MHI scan of the brain is indicated to exclude t: i
possibility of an intracranial mass. Dilated pupils will exclude
myasthe;ia gravis, but an edrophonium chloride test is indicatec
the pupils ar6 normal. Cosmetic surgery is required, but will not
improve ocular rnotility and visual function.

Case 13
1. Basilar artery rnigraine attack
,AV malformation
Cerebral infarct
lnfeetian
r': ,. Antcriat angiograrhy
Protcin level$
ffiNA dmeihte stram#
Th romrbophilis: screert
.&.nswers '11-20 33

l4eadaches in children

Types of headaches
Acute
Acute recurrent
Chronic progressive
Chronie non-progressive
Ciuster headache
Epileptic headache
Psychogenic
fvlixed

Acute headache may be due to migraine, eerebrovascular bleeej,


traurna, rneningitis, encephalitis or drugs.
Migraine headache is a recurrent headache and is characterised by a
positive famiiy history, visual aura, nausea, unilateral pain (throbbing!
and gastrointestinal syrnptoms. Other causes should be exeluded.
The incidence of migraine in children is 1.2*3"2% fft the age of 7
years, and 4-19% by the age of 15 years. ii is rnore prevalent in
{ernales and has a genetic component. About 2.8 schsol days per
year are lost as a result of n:igraine, and chilcJren ctrrrmonly have
migraine without an au!'a. fhildren do not usually have unilater;rl
headache. The pathophysiology of headache can be exp!aineti as
vasodilatation, vasoeonstriction, oederna, and inilammation of the
cerebral vessels, which can lead to pain. These are tl'le most comrnon
pathophysirJlogieal changes associated with migraine as suggested
by many ner-lrologists.
The clinicai features of migraine attack can be divided into phases.
The prodrome phase is thle first, and is associated with a change in
mood or activity level. This is followed by an aura {this occurs in
10-50% of paediatric migraine attacks) consisting of pirotopsia,
scotoma, numbness, tingling, ataxia. dizzlness and vertigo" The ne*
phase, headache, is mainly bilateral and classically not unilateral. The
resolrition phase usually foilows the headache phase, whie h may last
1*4 hours. The postdrorne phase occurs after the headae he is
resolved by sleep or analgesia. Patients will feel lethargic and
anorexic and experience mood disturhances.
There are a number of types of migraine. Migraine with an aura is
called 'classical migraine". Migraine without an altra is called
'comrnon migraine'. Complicated migraine is another form, and may
present as hemiplegic migraine, ophthalmoplegic migraine,
confusional migraine or basilar artery migraine. Atypical 'forms of
nnigraine are less frequent than others, e.g. with cyclic vorniting,
recurrent abdominal pain and herniplegia.
.Another type of headache is 'chronic Brogressive', which is
charaeterised by a daily headache with an increase in severity and
frequency. lntercranial pathology should be suspected. lt usually does
34 100 Grey Cases in Paediatrics for MRCPCH

not respond to analgesia and may or may not be associatec ''


neurological syrnptoms and signs. lt may or may not be ass.: I
with behavioural problems and needs urgent investigation, ,-.
rfeuroimaging in most patients.
When seeing children with headache, a thorough clinical exa-.
is very important and it is important to remember to examin: "

skin; chart weight, height and head circumference; listen for :'r
bruits; measure blood pressure and carry out a detailed CNS .
systemic examination (of the optic disc, eye movements, mc::
asymmetry, co,rrdination and reflexes).
There are high-priority patients who may need urgent neuro -
scans, including children with acute headache or chronic prc;
headache wiih abnormal or focal neurological abnormalities. '
papilloedema. Children with neurocutaneous syndromes (NF :
and headache also need urgent neuroimaging. lt is very unus - i
children < 3 years, and this is very significant. Other low-risk ;"
include those with chronic non-progressive headache, mixecj
headache, classic or common migraine, or a variation in heac:
location.

lndications for neuroirnaging in 78 paediatric patients with


headaches. tulaytal et ai. Ped. 1995;96:a13-4'17

Fleadache onset at younger age (< 5 years]


lncreasing severity or frequency of headache
Abnorrnalities on scular or neurological examination
Headache provoked by changing position of head
Focal symptoms or signs during headaches
Systemic symptoms, e.g. weight loss, fatigue
Doctors' and parents' concern about cerebral mass
Not specified
Atypical headache pattern

Drugs that are helpful in the treatment of headache are parace::


ibuprofen, codeine phosphate, ergot derivatives (to be avoide:
children), propranolol and clonidine. Other drugs that may be -
one or a combination of two of the above-mentioned drugs ar:
control!ing headache include phenytoin and phenobarbitone,
carbamazepine, gabapentin, sodium valproate and lamotrigine
calcium-channel blockers (nifedipine, cyproheptadine, flunariz -
Triptans (SHT,,B rD recepter agonists (e.9. sumatriptan) are very
effective in aborting migraine attack and can be used in childre-
16 and older. Use in children at younger ages is not yet license:
will be left to the individual specialist to decide.
Anti-emetics that may be used in the treatment of nausea and
vomiting accompanying childhood migraine are promethazine
(Phenergan), trimethobenzamide (Tigan ), prochlorper azine,
rnetoclopramide, and hydroxyzine.
Answers tr 1-20 35

ease ?4
1" b f-{ydrogen breath test
c Lactosequrantitative'Btudy
d Endoscopy and biopsy
2. a Lactose rnilk intolerance
b Colitis
3. c Hydrolysed rnilk products
d Lactose-free rnilk
f Weekly weight Rneasurement
g Re-ehallenge with lactose and biopsy

Congenital enteropathy {dietary !ntoleranee)


Dietary intolerance can present as vomiting andlor diarrhoea in the
first few weeks of life. Lactose intolerance usually presents with
diarrhoea. l-actose intolerance secondary to gastroenteritis is not
eommon in the early days of life" Reducing substanees will be a
positive step in the iflfant with diarrhoea. Prirnary iactose
iirtolerance vvill cause diarrhoea but lt is not a cornmon eondition.
These infants usually present with ciiarrhoea and failure to thrive
(FTT). Weight loss is comrnon but rareiy associated with vomiting.
The bload results witl be normal apart from signs of dehydration and
aciclosis. Hedueing substances will be positive, but a definitive
diagnosis can be rnade by small intestinal biopsy. A hydrogen breath
test wil! be helpfui, but it is diffieult to perform in small bab!es.
Lactose-free miik should be prescribeo for l:abies w'ith primary or
secondary lactose intolerance. Secondary lactose intolerance will
resolve and cows' milk should be tried for 3-6 months. Primary
laetose intolerance is a life-long condition but is not severe in later life
with cow's milk.
Glucose or galactose malabsorption, congenital chlorr:d iarrhoea,
abetal ipoproteinaemia, acrodermatitis enteropathica, and
congenital microvillous atrophy are other forrns of eongenital
enteropathy" Cows' rnilk protein intolerance is another form of
enter*pathy that may follow gastroenteritis and can
present either with vomiting as allergy or has diarrhoea as
enteropathy. The mucosa will be flat and recovery can be ae hieved by
changing to whey soya rnilk; challenge with cows' r:rilk should be
made after 3-6 months.

Casc 15
1. e Start prostagiandin infusion
f Half correction of acidosis by bicarbonate
g Arrange transfer to specialist centre
2. Postductal aortic stenosis
3. c Turner syndrorne
e Fetalalcoholsyndrome
g Down syndrome
36 100 Grey Cases in Paediatrics for MRCPCH

Coarctation of the aorta


There are Wvo types of coarctaton of the aorta: preductal (infantile)
and posductal {adult). They cause a na:'rcw'ing of the aortic arch
between the left subclavian origin anJ ducirJs arleriosus. The aortic
valve is usually bicuspid. The pulrncrar-ri !'enous and arterial pressure
will be raised as a consequence oi obsi';ciion ic aortic blood flow, or
of a left-to-right shunt with paten: C..rcius aleriosus or VSD. The rlse
in systemic blood pressure is due :c =€:cra..icei cbstruction, and
Increased renin production is due:c ieJ.rc€c:-e'ra, perfusion.
Coarctation can present at any time ..,,',- - :f-e :.:si month of life. ln
the first few hours of life the con,- 3-es: :':se-:a:,on is shock and a
respiratory problem, which may oe :e,a::: it c-,1i.:S arteriosus Mresial temporal lob
closure. Weak or absent femoral l- ses ,., :- a r,'t: gap between the --<<-nafla1c-<da
blood pressure of the upper and ic,',e- ,.r-:s is a^ lndicator of
possible left ventricular outlet obs:'-:i :-. :-e -.r. ccai-ctation. The
second heart sound, which will be 3-c "', :^ :: rJrmur most of the
*- a:eial febrir: se
. _
.

.E^
>.= - . = ;r ;s.
time, and a strong apex beat due ic .,e.:':',l a: l',oer-trophy, are other :--:q- i.l':-;-t
= -^,
features that may aid the clinical c a_.-.s s --e :- :d is acidotic, and
ventilatory support is required as \.,e as (e€p -J:re cjuct open by
prostaglandin infusion. Antibiotic c3re. S.:! c a s: be started, as
well as correction of acidosis rvitr f - 3 a-: 3:;a'a.nate. The team of
specialists should be informed anc a--3rgs':e-:s fortransfer should
start. lf the duct is closed and the baL," s ge:: -g d,ci-se, then a -:-- r c-:l th=-= ..
septostomy can be performed under :C-3 *- :3-.e by a well- - - -i: atF,cr c'-= ^.
trained cardiologist.
f,?-i rr a:eas C'A:.
Early CXR may show pulmonary cess- 3 a-: :"::l c:egaly, and ::-^: :ases in i.r- :"
appearanceof a'Swiss loaf'heart:a- E€ s3E- =: -.ctching occurs in - - lr |-u ..lc-: *
older children who become syriiri:-e: : s::' l- :CG in newborn -:, -e g ic i3:€ se
babies will show right ventrici-;lar i',:ea-::-, !:-:"carciography is -.ff- ^is7=?ai ^- ^

diagnostic and the pressure g.ac :-: 3:'::.s i-i txst::i1ion can be
estimated. Cardiac catheterisar;i^ .- a a-; :.,_:-::- . snould preferably
not be carried out in sick chilo.er a-::.2'3€ ::-e 't:e diagnosis is Use 17
unceftain. Surgical correction an: 33
be used, and balloon dilatation is e*e;i==- =- z'i.::-
:
angioplasty can
:e-:trirdren or in ' e ir':iesira r,t
"E
recoarctation after surgery. e lt4mxe s :.
, Anai sss.e
i rcreasea --E*
Case 16 3 i,iecke.'S s i31
1. a Complex partiai se r:'es
2. c Sleep EEG &iec*el's divrtia.unu
a Cranial MRI -- < 2. Ces f,- _ --,;
T ELb - , YE sJr
-- - t
3. b carbamazepine --- :- ^ j * t--- -
_.1 _,
Ternporal lobe epilepsy - (. := 6- = a-,-- €

This is the most commoF catse :o ::*: a, ;.E-.a s€:!:es in children


:- eY s {c" ls- s*:
.=
and adults. lt is usually p!'ece e3:. :- :--: -,3 -!::-fir3r and illusion, - -e r d. I

epigastric sensation in young af :-:- ",, :- -ii- -e;: oeviation and


Answers 11-20 37

posturing in children <.2 yearg of age). The seizure may start


as staring
or behaviour arrest and.then be foilowed by automatism flip smackinj,
licking, chewing or swallowing movement.). So,n" other bizarre
behaviour such as fingering or fumbling with clothes is arso fretuent
and can be misleading in the diagnosis-of this condition. Tonic
posturing and head rotation are often seen in those < z years
old
during their usual seizures; sometimes there is just jerking oi t*it.ning
of one side of the body which may or may not Ue tottowei Uy
generalised tonic-clonic seizure. This will last i-2 min and
be followed
by confusion and/or intense tiredness. ln older children and
adolescents, mesial te.mporal lobe epilepsy may be present.

Mesial temporal lobe epilepsy


This is characterised histologically by atrophy and gliosis of the
hippocampus, and often tfre amygOala. lt is associaled with long
unilateral febrile seizures in a0-60% of cases, and partial."irurd,
start in the.first 10 years of life. Seizures are marked by behavioural
arrest,.staring, automatism (facial), and dystonic posturing of the arm
contralateral to the dischargi ng tem poral iobe wh ire autoriratism
occurs in the ipsilateral arm.

EEG
There is an interictar anterior temporar spike focus or
sometimes a
paroxysmaltheta rhythm. An ictar EEG wiil show
a diffuse discharge
on the other side. cranial MRI using an epilepsy protocol wirirrerpio
identify areas of abnormarity in thJhippocampus or temporar robe.
rn
some cases in which surgery is required and the focus oi discharge
could not be determined, a pET scan or ictal and interictar SECT scan
may help to localise the focus of the discharge. Many cases
have
been operated on with temporal lobectomy. and the
outcome has
been very good.

Case'/,7
1. a lntestinal polyps
e Meckel'sdiverticulum
j Anal fissue
2. lncreased uptake
3. Meckel'sdiverticulum
Meckel's diverticulum
This arises frorn the vitelrointestinar duct, which reads
from the
p-rlTili"g gyl t" the yotk sac. tt occur s in i"/o of the poputution iZ in.
long ancl 2 ft. frorn the ireocaecar varvel. The viteiloiniestinar
duct
may cause other abnormarities. rf it remains open, this wiil iead
to
fistulae, or a fibrous cord from the umbilicus tb the ileum
may act as
an axis for localised volvurus or read to bower
obstruction" Meckels
become inftamed and cause bteeding or
fl::Ii"yl-t.nray
tntussusception, or may present as scute appendicitis.lt
usuallv
38 100 Grey Cases in Paediatrics for MRCPCH

of
contains heterotropic gastric mucosa, which is the main cause
in pre.school children. lt
;d;i;g" This is usuaiiy a painles.s.bleeding
of bright maro.on.blood'
i;;.;rlV a large haemorrhage with pas-s^agj by *-Tcpertechnetate iso.tope
fvfu"f,"f;Jr"an i's helpful in diagnosis
affinity for parietal cells. Diverticulectomy is the
icin, whicfr has an
treatment and there is no recurrence'

Case 18
1. c EchocardiograPhY
d Liver biopsY
2. Alagille sYndrome
lntrahepatic biliary hypoplasia (Alagille syndrome)
This is usually inherited as an autosomal dominant syndrome'
fVf oit....r have jaundice from the
neonatal period vrhich disappears
U, .frifOfrood or idutt life; sometimes it can present as pruritis
if," f cholestasis causes pruritis' jaund-ice'.
"lfv. "ngstanding and xa nthe Iasrna. Ons"qu a rter of patients will
f,Vp"r"frof
"stJrolaemiI affecting mainly peripheral pulmonary
nive carOiac abnormalities,
are atypical phenotypic features' whic-h include
urt"iv it"notis. There
forehead
;;;h;i;ri;m, a polnteo noie, deep-set eYes' overhanging
and a straight nose with a small pointed chin' A high serum.
will support the ciiagnosis. There is cirrhosis of the
pruritis,
river"=i"*ilevel
"frof in 15% of cases and'other liver disease in 10%. Treating
nVp"t"f,of"tterolaemia and cholestasis heips to prevent further
pfl"utions in these patients' Cardiac surgery is not always
is. vital.
i"dil"i; but regutar foltow-up with echocardiography.
"6F
prenatal diagnosis has not yet
G;";til corn"etiing is important and i\
been established.
r
Case 19 ll

1. a Hyperinflated right lung in two X-rays :


h Righ, lower lobi segmental consolidation
') d Chest CT
a BronchoscoPY with lavage
i DNA linkage study for CF
jlg
3. i.r. aitiUiotics for acute infection for 2 weeks
Chest phYsiotheraPY
4. Bronchiectasis

Bronchieetasis
This is a chronic lung diseases that can follow in{ection and especially
plji-r*.V tuUercut6sis, which has started to appear more frequently
in ,"""nt y"rrs. lt is usually associated with congenital lung
of a
aUnormatities such as bronchomalacia, sequestration, aspiration
i;;"ig; body that is missed, and chronic chesl infection associated
Answers 11-20 39

t'vith cF, ciliary dyskinesia, or primary irnrnune deficiency. The usual


symptoms are cough, rryheeze, poor weight gain, bad breath, purulent
sputum and recurrent chestjnfection. Fingeiclubbing is another
feature; it can be easily test6d by demonsirating diariond signs
(putting fingernails of thumbs apposite to each other the diamond
s.ig.n will disappear), or looking at the curvature of the naits or testing
fluid in nail buds. chest X-ray may show an area of consolication,
usr.rally one of the lower lobe segments, but chest cr is very helpful,
and will show the area of consolidation as well as the diiated
brpnchioles. Bronchoscopy will be helpfui if the presence of a foreign
body or any congenital bronchial or alveolar abnormalities are
suspected. A lung function study will help to see if the patient suffers
from an obstructive or restrictive rung problem. physioiherapy and
bronchodilators will help, but interrnittent antibiotics when sputum
shows infection are important to prevent further damage. surgicai
intervention is not necessary if the patient is followed rlp closily and
antibiotics are given when needed.

Case 20
1. c Cranial CT
2.f Attenuation and low density of left parietotemporal lobe
a Cerebral oedema
d Small lateral ventricles
3. b Fundi examination
4. a A right middle cerebral artery infarct
c Shaken baby syndrome
s Accidental trauma

Neonatal cerebrovascular accident


This is more common in fulr-term than preterm babies. Neonatar
infarction can be divided into three types: (1) Single-artery infarction,
which may be caused by injury to the cervical poition of t'he carotid
a.rtery during difficult delivery; but this is very iare and most of the
time a cause cannot be found; (2) Multi-artery infarction ."y t"
c.aused by congenital heart disease and less irequently by perinatal
distress or disseminated intravascular coagulation and
polycythaemia; (3)Arterial border zone infirction (insula) may be
caused during active resuscitation as a consequence of hy.potension.
Single-artery infarct, usually middle cerebral artery (MCAI or frontal,
occurs.withguJ gny obvious disease or risk factors. Newborns appear
normal at birth, but repetitive focal or generalised seizures occur at
any time during the first 4-7 days of life. lt may present as a
weakness of one side, which is sometimes very'difficult to diagnose,
but repeat examination for babies who present with focal or
generalised seizures will help to elicit eirly weaknesses. There is a
risk that 2o-40Yo of these children will deverop either hemipiegii or
s.ubtle hemiplegia. The rest usualty will be normal, but all depinds on
the area involved and how severe the early presentation was.

$i
l$l'
#

ffii
r
l

i
40 100 Grey Cases in Paediatrics for MRCPCH

Presentation with weakness and seizure is usually not a good sign. OUESTIONS 21-30
cranial cT with contrast or MRI is very important to demonstrate the
area of infarction and to see if it is associated with any parenchymal Case 21
bleed. US is also v6ry helpful iri the case of a large MCA infarct. The A 10-year-old child p
follow-up may show ventricular dilation or porencephaly contralateral over his legs and arr
to the hemiparesis. Maternal use of cocaine during pregnancy can illness. He was on a
cause infarction. Anticonvulsants are effective in controlling seizures. when he felt sick for
Phenobarbitone is more effective in the earty iays. with a later switch them due to chromo
to other anti-epileptic drugs (AED) in patients wr[h in:ractable pains in the past anc
seizures. Only 20-40% of these patients may neec long-term which responded to
anticonvulsant treatment, and surgery for patirents whh epilepsy and organomegaly. His t
intractable seizures should be considered- results are:
Hb 12.1
WCCs 8.4
PLT 15:
PTT 3ti r
BT 1rr
Film PL'I
1. What is the mos
2. What are three I
5. What is the trea

Case 22
A girl presented whr
feet'and has had po
development was w
previous concems. I
with her gait and is r

brother has describr


hospital 1 year ago r
antibiotics and had I
healthy, although tx
required surgery. lk
old house with darry
active, but now b wi
She swings when slr
eyes started to tum i

conjunctivitis was p
found on systemic a
ESR
Hb
Hb electrophoresb
Ret
Ca
lnorganlc phosphae
Vitamin D
*uESTI0N$ 21-3{)

ease 21
A 1O'year-old child prur"n'tuc with a history of bruising, which spread
over his legs and arms- He is an onry chirci with no taniity history
of
illness. He was on a school trip to the south of ttaiy : welfs ijo,
when he felt sick for 1 day. His morher had four ot
ti'rern due to chromosomai abnorrnarities. He has'-niscarrrigls,iiro
eompiai-ned of leg
pains in the past and recently he was diagncsed as having m;jraine,
w'hich responded to simpre analgesia. Hiis afebrire and tierels no
organomegaiy. His urine shows RBCs and no pus ceils. The test
results are:
Hb 12.5 sldt
WCCs 8.4 x 10s/l
PLT 15 x 10s/l
PTT 38 s (control 35-50 s)
BT 1 rnin (control 3S-00 s)
Film PLT small in size
1. What is the most likely diagnosis?
2. What are three possible causes?
3. What is the treatment?

Case 22
A girl presented who is described by her mother as 'unsteady on her
feet' and has had poo.r weight gain since the age of 3 years. F{er earry
development was within the normal ,ange and-there were no
previous concerns. she is now 6 years old, is stiil having
a fioutem
with her gait and is not able to ride a bicycre. Sornetime-s rrer oiaer
brother has described her as a 'drunken ir,ito'. she was admitted to
hospital 1 year ago with bad chest infection, and was treated with
i.v.
antibiotics and had fruid drained from her rungs. Both parents are
healthy,.although her father had a problem wiih his eyes tfrai
required surgery. Her younger sister is hearthy. rrre tamiry rives in
an
old. house with damp and alailing ceiring" she used to n"'uiigr,t
uno
active, but now is withdrawn and sreepylano not as noi.yi.i"iorr.
she swings when she watks and rras mirteo tremor in her hands. Her
eyes started to turn red in last 3 months and treatment for
conjunctivitis was prescribed by her Gp. No other abnorrnalities were
found on systemic and general examination. The test results aie:
ESR 90 rnm/hour
Hb 7.2 sldt
Hb electrophoresis No haemoglobinopathy
Ret 1.2%
Ca 1.78 mmol/l
lnorganic phosphate 1.97 mmol/l
Vitamin D 10 mmol/l
42 100 Grey Cases in Paediatrics for MRCPCH

1. What is the most likelY diagnosis? Case 24


2. Which four other blood tests will helo the i a3-:s-
s:
J. What other clinical manifestations may ie ' e : : A 7-year-old boy with :
P family doctor. His mot
born at term with nc n
Case 23 ages of 5 and 6 years.
A family of Middle Eastern origin presenie: "" :-- will suffer from bad b.
who has become more pale and tired in :-e 3s: healthy, as is his fathe
URTI 1 week ago and has since never re7a '2:
- his attempt to climb a
parents are first cousins and have an oi::' :- : systemic or general ex
The patient had always been describec es 3 ;' of pregnancy for reasc
is pale. His eye movement is chaotic ar; - s owns a horse and the
a 'clumsy child' and has said that in tF€ :s: hospital 3 months ag:
lot. He has not participated in any 9a-es '' which is still there. T:,
he gets tired very easily. He is on the 3': he feels upset not tc :
centile for height. His cousin died frc- = or play football, as tre
the age of 3 years. His parents visit i:e '
- haemoglobin level arl
and nis grandmother suffers f rom c-''; :
- There is no rash anC a
-3 painf ul.
He looks pale, with a distended ab::-:- :-:*: *i- - 2 cm,
with a possible renal mass' The f ur: 3-= - : i -': s no joint 1. What other tests s
problem. He is anaemic with nor'1a = =''t 3-'l rc-
''
lf,unts. a Blood film
iti. liu", function and renal f uncii:- ='2 - :*- -
' b Bone marro'r"'
{etoprotein level is within the nc--: -=-;: ' c Chromosorne
investigations.e- :: ::-:--'*:':
d Bleeding tin:t
i. Which three e PTT
diagnosis, and in what orcje'r f INR
a Blood film g Platelet coun:
b Abdominal US h Liver functic.
c l.v. pYelograPhY (lVP) iPT
d Abdominaland chest C- 2. What is the diag.
e Bone marrow bioPsY a Haemophii;a
f Abdominal mass bioPs'" b Glens syncr':
g HePatitis B serologY c TAR syndror
h HePatitis C serologY d ALL
i ChromosomalstudY
-: -:'r?-'ll !: 3: e von Willebra:
j Urinary vanillylmanie : =-: f Henoch-Sc^:
2. What is the diagnosis? g Ehlers-Dar. :
3, List three stePs of manags-:-: h Christmas e :

a HyPerhYdration
-:*: -t &'l-- i Aplastic anar
b Adienocorticotropr''l: 3. What two other :
c Oral alloPurinol ' a Factor Vlll as
d l.v. antibiotics b Factor X ass;
e Oral hYdrocortisc': c Bone marrc$
f Remove abdo- -3 -::-: d Chromosor'-,
g lnsert urinary ca:-?::' e Platelets ns:r
h Nil Per mouth f von WilleDr:
i l.v. antifungal a9:-: g Bleeding ti: ,

j Blood transfusi:- h Fibrinoge'r .:


k Transfer to sPecia s:: : -r::'':{ir !'r'r''
I Peritoneal ciia 's s
Ouestions 21-30 43

Case24
A 7-year-old boy with a history of recurrent bruising presented to his
lamily doctor. His mother also suffers from recurrent bruises. He was
born at term with no neonatal problems. He had nasal bleeds at the
ages of 5 and 6 years. He cannot play football, as he gets tired and
will suffer from bad bruises. His two older brothers are well and
healthy, as is his father. There are two bruises on his thigh following
his atternpt to climb a tree.'No other abnormalities are found on
systemic or general examination. His mother had a termination
of pregnancy for reason of fetal cardiac abnormalities. The family
owns a horse and the boy looks after it. The boy was referred to
hospital 3 months ago for a 1 x 1 cm cervical lymphadenopathy,
which is still there. The bruises are becoming more frequent and
he feels upset not to be allowed to go swimming with his friend
or play football, as these cause more bruises. His WCC,
haemoglobin level and lymph node biopsy were reported as normal.
There is no rash and a normal MSU. All joints are intact and are not
painful.
1. What other tests should be carried out?
a Blood film
b Bone marrow aspiration
c Chromosomal study
d Bleeding time
e PTT
f INR
g Platelet count
h Liver function test
iPT
2. What is the diagnosis?
a Haemophilia A
b Glens syndrome
c TAR syndrome
d ALL
e von Willebrand disease
f Henoch-Schonleinpurpura
g Ehlers-Danlossyndrome
h Christmas disease
i Aplastic anaemia
3. What two other blood tests should you carry out?
"a Factor Vlll assay
b Factor X assay
c Bone marrow
d Chromosomal study
e Platelets ristocetin aggregation test
t von Willebrand factors assay
g Bleeding time
h Fibrinogen level
44 '100 Grey Cases in Paediatrics for I"iBCPCH

i Plasmaphere
Case 25 j l.v.vitamin K
The following are the blood results from a ?year-old child with k Correct acidc
diarrhoea an-d vomlting that hai continued for the last 12 hours. His
mother suggested tnaf fris stool looks mucousy, with streaks of a
reddish mlxture. He has stopped drinking and eating, and is not Case 26
happy. His HR is 110 b.p.m., RR 40/min and BP 110/60 mmHg' He An 8-year-old girl pres
looks'pale and shut down but his capillary refill is < 2 s centrally. His days and an inability t
mother had a similar illness 2 weeks ago but recovered within 3 days' walked to school that r

His doctor prescribed Dioralyte and analgesia. His urine output is sick, and there have h
minimal. He was admifted to his local hospital for further was on a coach trip to
management. The test results are: developed a very bad
days and was able to r
Blood places, including the E
,Na 125 mmol/l
Now she says she'car
K 5.3 mmol/l
hard on them. The re{l
cl 90 mmol/l
upgoing. She is not re
Urea 21 mmol/l
the suprapubic region
Cr 230 mmol/l
the upper limbs and o
HCOs 14
as she has not passed
Urine (cerebrospinal fluid IC
Na 10
Osmolality 700 mosmolil wcc
Blood culture Negative Protein
MSU Negative Glucose
Abdominal US No abnormalities Gram stain
c/s
1. What other investigatrons shculd be carried out to help in PCR, HSV/polio/TB/M
diagnosis and management? Enterovirus
aHb lgG oligoclonalantibr
b INR SpinalX-ray
c PTT
dPT 1. What is the most
e Blood film a Spinalcord c
f Platelet count b Syringomyet,
g LFT c Acute transfl
h NH4 dMS
i Abdominal CT e Guillain-Ban
j rvP f Occlusive va
2.
k Blood gas
Whut it the'diagnosis. iudging frorn the blood test?
sh Spinal
uTl
shoct
? Wf,i"ft four step-s should torn ' tne iniiial management of this child' 2. What are the otlx
and in what order? a Spinalfi
a Fluid restriction to 300 rirl i'n: of sur{ace area b SpinalMRl
b l.v. antibiotics c Spinal and cr
c Urinary output to be added to total ffuid d CranialMRl
d -hourly U&E e Peripheral ne
e Transfer to sPecialised unit f DNA double r

f Platelet transfusion g Cerebralang


g Blood transfusion h Bone marrou
h Peritoneal dialYsis
0uestions ?t*30 45

Flasrnapheresis
Lv. vitamin K
Correct acidos!s

Case 26
An 8-year'old glrl presented whh
a history of back pain for the iast 7
an,inabitity to watk ;" ir," ir_ia iJurs (sr,e
::^y:jj-dio schooi
,'v,ixec! was ail right *nd
thar ;-norning). She had ,irild f"o"n
siek' and there have be-en no u"C
ot',u, sy*ptoms in the rest 24"ril""ri"g
hour.s. she
\,'as on a coach t'io to France
with her s.f,oof 2 wee*i ;"g; ,;; "
a very bad cotd on *," u.roi;;;y.
I:::l?r:o
+ays and was abre to do- most or tr'. She recovered within 2
places, inciudinq the Eiffert*;;;i;;;t.'iH"re with the schoor to various
"i$iiig are no other concerns.
Now she says s6e,can,t feeihe-r
t;;;;';;r;;hen
ha'd on them' The refiexes in'i";l;;;;;i;bs her mother presses
upgoing' she is not responding are brisk and rhe prantar
to pain oiiou.r, in her rower rimbs up to
region.,Arr .irnLl n!-*eu"urJ
lF
the :Yptuqybic
upper limbs and other system. inrr., and examination of
as she has not passed^an-y
,**"r.
i. Hen uraJoer-i. prlp'iiu,
urine- in
'ri rrrY r*i'ig
ti," resr hours. The test'resurts are
(cerebrospinalfluid "
ICSFji:
wcc
Protein 50 (all lymphoefies)
1.2 9/l
Glucose
Gram stain 2.8 mmolll (serum 5.6 rnrnolll)
c/s No organism
PCB, HSV/p otiotT Negative
BI M ycopt asma/
Enterovirus
lgG oligoclonal antibodies All negative
Spinal X-ray Not found
Normal
1. What is the most likely diagnosis?
!b Syringomyelia
lOinalcordcompressi6n
c. Acute transverse myelitis
dMS
e Guillain-Ba116syndrome
f Occlusire ,ascuia, disease
S UTI
_ .t
2.
Spinalshock syndrome
What are the other most useful
investigations?
a SpinatCT r
b SpinatMRt
c. Spinaland cranial MRI
d CraniatMRl
e Peripheral nerve conduction
f study
DNA double strand
s Cerebral angiogiaphy
h tsone marrow biopsy
46 100 Grey Cases in Paediatrics for l"='-- -'

i Et\4G Gase 28
j Urinary VMA"
3. What is the immediate treatmen:7 The following CSF rer
a High-dose pulses o{ methylp:31- s: .- 1 --' to A&E with a mild te
followeci b$ oral corticoste!'c,ls ':- i ,'.=' . seizures. His mother
b High dase of oral prednisois::':' : ' =.'= days ago for tonsilliti
C IVIU appetite improved bu
d Plasmapheresis time to time without
e Surgical cord decomPressi:- was still drowsy. His
f lntensivephysiotheraPy b.p.m., RR 22lmin, an
g Bladder catheterisation care His mother said he sr
each year, and had oi
past at age 6 months
Case 27 seizures, and 5 mg re
A7-year-old girl presentedwith a l's::-. -' - carried out. He maint
hair as well as pubic hair. Her mo':r:' : :.' :: ward after he was sta
of 13years. in the lastfew days she s:=:::': results are:
sickness with abdominal pain. Her
",=
;-: : -: WCCs 120 i
her height on the 75th centile; her' -::-:- : n Protein 200
ceniile. Her pubic hair is stage T3, ar ?'. :'.
I

Glucose 1.1 r
stage 1. She has never had any vaJ - Serum glucose 5.1 r
mother and spends 2 weekends pe- * Gram stain Negi
married and has iwo children w'i:- - . RBCs 0
wakes up ei night, cornpiaining c' a:
nrother, telling i:er about her nig-:- 1. What abnormalhi
yor.,th ciub and finds it difficult tc :-
;hanEing room. Her" GP assurei ':- - :--.
w*rry about when he saw her io: :-= '
the oubic hair. Mcst of the tirre : : - =- ' :l
se hooi. as her mother works as a '- .' - .
a re:
Testnsierone l.0nmol/l
Cariisol 22C nrnolil
TSi'i v.v mUli
'14 pmol/l
Free ti-rvroxine
FSt-: 21.2ull (follic-:-:
'--.1 ?C,,!t
L, u, l {fnll ..
\tvlll9v =-'-
9

'.)estradiol 45C pmolil


-_: - >l
-')
1. Wt"rirh three other investig:: : -
a [-H/FSH stimulation tes:
b Chronnosonral study
c Abdominal US a Cerebral oed
d HistosalpingograPhy b Middle cereb
e Abdominal CT c Subarachnoir
f UrinaryiT-hYdroxYPrc3:s: d lncrease in sr
g Glucagon-suPPressiorl::s: e No abnormal
h Cranial MHI f Basalganglia
i Short Synacthen test g Absent corpu
2. What are the three differen: e :
Suestions 21-30 47

esse 28
The following CSF resuits are fronn a 3-year-old boy who presented
to A&E with a mild temperature and generarised tonic-cronit
seizures. His mother is a 6urse and siirted hirn on a*orvciiiin +
days ago for tonsillitis. He kept on having a mild temperature; his
appetite improved but he was stiil retharlic and keptl.ving t.o*
time to tirne without a reason. His seizuri stopped'afteis irin but he
was still drowsy. His temperature in casualty'was 36.4"c, his HR 150
P_tp.m., HR 2Zimin, and Sat 9g% on 5 l/min oxygen via a facemask.
His mother said he suffers from tonsiiritis quiiioftun, up t" a ti*".
each year, and had one bout of left upper lobe pneumonia in tne
past at age 6 months. He started having generalised tonic-cronic
seizures, and 5 mg rectal diazepam wal iiven and a cranial CT was
carried out. He maintained his airway anl *as transferred to the
ward after he was started on antibioiics and acyclovir. His cSF test
results are:
WCCs 120 (60% tymphocytes)
Protein 200 gll
Glucose 1.1 mmol/l
Serum glucose 5.1 mmol/l
Gram stain Negative
RBCs 0
1. What abnormalities are visible on his cranial CT?

a Cerebral oedema
b Middle cerebral artery infarct
c Subarachnoid haemorrhage
d lncrease in subdural space
e No abnormalities on both scans
f Basal ganglia infarction
I Absent corpus callosum
48 100 Grey Cases in Paediatrics for MRCPCH

2. What other single test should you carry out?


i Skin allr
a Mantoux test j Upper:
b lgs(immurnoglobulins) k DNA lin
c Lymphocyte subset I Faecal r

d HIV test
e Herpes simplex PCR on CSF Case 30
f CSF PCR for Staphylococcus, Streptococcus and Neisseria A 14-year-old b<
g EEG
enlarged cervici
h MRlscan
tonic-clonic sei:
3. What is the diagnosis?
headache. Therr
episode when h
extraction. Bloo
Case 29
electrolytes, an<
A S-month-old baby presented with a history of recurrent chest hyperpigmenter
infections for the last 4 weeks. He has not gained weight for the last 4 He is also weari
weeks (his weight is on the 3rd centile and his bir-thweight was on the by an ophthalm
25th centile). There is no diarrhoea or vomiting. He had three CXRs carried out privi
and was reported to have hyperinflated lungs. He has just started were normal.
solids, which he likes. He looks puffy, pale and thin' He is hypotonic 1. What is the
and has stopped smiling. His abdomen is soft and a mild wheeze was
discovered during a chest examination. There is no family history of
a Reflex i
chest or gut problems and his parents are not reiated. This is their
b Vasova
first baby and there were no neonatal problems. He has fed very well
c Panic a'
from the first day of life and passed meconium in the first 18 hours
d ldiopatl
after birth" He is anaemic and a sweat test showed Na of 46 mmol/l
e Hyperki
(N = 20-60 mmol/l) of reasonable sweat. There are no other concerns f ldiopatl
and both parents are heatthy. There is no family history of any
g Tics
illnesses. The lgs, stool for reducing substances. ptatelet count and
urine culture were all reported as normal'
ANSWERS 21-3
1. What are the four differential diagnoses, and in what order would
you list them?
Case 21
a Lactose intolerance
b Gast(o-esoPhageal reflux (GOR) 1. ldiopathic tl
c Tracheo-oesoPhagealfistulae 2. ldiopathic
d Cystic fibrosis' Viral infectir
e Schwachman sYndrome Autoimmur
f Protein cows' milk intolerance 3. Supportive
g Autoimmune deficiencY sYndrome
h Congenitalcystic lung disease Thrombocytope
i CongenitalenteroPathy This is an autoir
2. Which othei three tests should be carried o.r"i? Neonatal throrn
a Sweattest with hydrofludrocor-t.sc:'le s{.,lppression retardation (lUG
b Hydrogen breath test maternal haemc
c lgG subclasses (HELLP) syndror
d Bronchoscopy and lavage (thrombocytope
e HIV test syndrome; neor
f pH study usually idiopath
g Chest CT tissue disorders
h Barium swallow
Answers 2l-30 49

I Skin allergy test


j
k
t
iifi 11
raecai
il',
elastasis
if 1"ff t f
P
sF
oscoB v

Case 30
A 14-year-o,O Our,.l"1blsod
*=nlarged cervicai'lympf., taken by his Gp for investigarion
-H"ii"n*O
gf rnds. of
to n ic-cr o n ic seiz having gene_ralisec
i,ssdache. There
u res i **"a to, e-# r',' ioi cwe o b y
"ni"r"i*.'ii,t'tu epitepsy
is:i-o_rr*i;ii"irtorylt
and he harr
;*L11f-H.B,l5E :l:lo:o "' i""i" i L Jpo, ntmenr ror a to orh a s!mitar
.. -
s *:i:;!,
ffiii'*.,*rlT#il,"#,l#"{#,'a;
r-le rs aiso wearinc flrt x
c y an r,i l"j' iJ.,
oph r h, i * ; rr"
g
i :.. :;; i". i i'r? Ho.,t l';#X,:i,rJ#]:,
;*ss":*ri,T.1l,1i:T;L'",[11il_i:*:.u:[{::{mitu,
l. What is the rnost likely
diagnosis?
a - -
Reflex anoxic r"i.rr"r-
b Vasovagal attack
c panic attack
d ldiopathic generalised
seizure
e Hyperkeplexia
r ldiopathic focal epitepsy
g Tics

ANSWERS 21.30

Aase Z,l

thrornbocvtopenia purpura
;. l3[B:|il: *rp)
Virai infection
Autoimmune/drugs
,
r. supportive with 169ular
foliow_up
TFrrornbocytopenla
in cFrildren
This is an autoimmune
disease that affect
thr.ombocyropenia at any ase.
I:^":_*, is associareof,lll,I::

g;ffifi***figutfi"rl*n;
t is s u e ir ;.
"ii"
i. [ ;: L iX;,' H;: ;",,,", I #:|,3 f l.g. j n r".ti o-n,'"f ,in
Tv.adenovirus, i u -
".t "
8.,_,ifolate
50 100 Grey Cases in Paediatrics for MRCPC*

indicaties problems ;' :-


deficiency, haemolytic uraemic syndrome, Ka s bac h-Me rritt ai"e disturbances in b::-
syndrome, endocarditis and aplastic anaemial is i:l^,coriant. is absent in 7S-8Occ :':
The |TP-affected child usually presents with b'u'ses 2-r w'eeks after levei is eievated. The :-
viral illness. The disorder is characterised by a' -:':aseC amount of feti:protein level is h g-
lgG on the platelets. The reticuloendothelial 3e s ", '.- ' :he spleen rncreaseci incidence :' -
will usuallyclearthe lgG-coated platelets. Easr' :'- s -i. purpura iyrnphnb!astic leu kae -
and petechial rashes arethe acute presenting '?3:*'=s Ecistaxis i:*fore the age of 20 '. =.
is the presenting feature in some patients. Oi":'s :':se:i with r:as,s of clinical situa: :
mucous membrane bleeding (very small nu-::'s S3.:e patients increased EF{A breaxc:
may have splenomegaly but in the maioritr- 3?'.'. a'c systemic useiul tests that car ::
examinations will be normal"The blood f rlr ," s*:;' s:nall platelets, suppoft are very irnP: -
and bone marrow aspiration is not indicaiec -- :ss s-scicion of prenatal diagnosis ca-
malignancy is raised. Recovery of platelet c:-^: "', :- - I month
occurs in most patients andTSYo of patien:s '." 'e:: ' e' within 6
months. lf there is no mucous membrane 3 3e: -3 :' :'sk of ICB
(intracranial bleeding), there is no need tc :'::: :-3 ;?irent with lVlG. Sase *3
Platelet transfusion is not indicated unless :-:'. s a:: ''e bleeding ;" i Urinary va^
and the platelet count is very low' Genera ?=. :: ::: -: avoiding d Abelomiriai :-
accidents and contact sports for the per,:: :' 1,", : a:e et count is
.

f Abdon':inal -:
very important. This disease will perslsl {:' -:'3 :-3- 5 years in :. Ft--{yr:clonic eilcei -
fewerthan 5% of affected individuals a^: s: :-e:::-; ls not
:

indicated inthe majorityof these cases --3 - ---.. :,


-a:e rs very low + r.j utcr !ryu. _, _

c Sral alle ,. - - :
in patients with lTP. r. Tra'sfc; .: .-:
Case 22
1. Ataxiatelangiectasia
2. lg
Celtular DNA
Chromosomes
CranialMRl
3. Malignancies(lymPhomas)
Bone marrow failure

Ataxia telangiectasia
This is a multisystem disorder afJec: -; :-: ::-:"3 ^3-'"ous system
{CNS) and immune system. lt can ce i'a- s- ::: :-': -gr autosomal
recessive inheritance. The genetic i:-3-r i- - as :::- ::ated on the
long arm of chromosome 11' The :'a- ':s::: :- :i- ::gin as early
as t-he first year of life, with ataxia. l-: :- : -s-; " lresents with
clurnsiness and frequent falls, whl:* :e: =-= =':;':ssive.
sometimes this may be accompan:ec :, :-:' :a:-e:trtic movement'
Oculomotor apraxia is present in 9C'. l' :3: e-:s a^: can be early'
lntellectual development witl be nc f :'- 3 - :3 " ' ': : -t will decline
slowly as the child grows up. Telang 83"'as : 's-! r oevelops in
years 2-4 of life and sometimes ei i'€ a3€ :' 'I : a;oears as a
-
btoodshot bulbar conjunctiva. h aisc a3:€3"s : :-3 rcper part of
the ears, on the flexoi surface of iir.:s B- : as : : -:e:f1y distributiOn
on the face. Frequent lung lnfectio- :3- :-'e-' s?' :'s' and
Answers 21-30 51

indicaties problems with the immune system in these patients, There


are disturbances in both B- and T-helper eells. Serum and saliva lgE
is absent in 7O-80% of patients. lgE is absent in 90% but the lgM
level is elevated. The thymus ls less well developed and the alpha-
{etoprotein level is high in the maiority of patients. There is an
increased incidence of malignancy. in particular lymphoma and
lyrnphoblastic leukaemia; at least two-thirds of these patients dle
before the age of 20 years. The diagnosis is usually made on the
basis of clinical situation and signs. Genetic study as well as
increased DNA breakdown when exposed to radiation are other
useful tests that can be carried out. Regular follow-up and family
support are very important. Genetic counselling are vital and
prenatal diagnosis can be done.

Case 23
'1. j Urinary vanillylmandelic acid
d Abdorninal and chest CT
f Abdorninal mass biopsy
2. Myoclonic encephalopathy (neuroblastoma syndrome)
a e Oral hydrocortisone
c Oral allopurinol
k Transfer to specialised oncology unit
$!'!yoclonic encepha!opathy {neuroblastoma syndrome}
This is characterised by chaotic eye rflovements (dancing eye
syndr*rne), myoclonic ataxia and encephalopathy. it rnay occur
idiopathically or secondary to occult neurobiastoma. The outcome
is usr.raily the san"re, whatever the mode of presentation. lt can
present at any tirne from 6 rnonths to 6 years of age. Clumsiness
or abn*rrnal eye nrovement is usuaily the first presentation. which
rnayiake a week or lcnger before being brought to the attention of
a medical professiona!. tdore than half will present with irritabillty
*nd behavioural changes that incicate encephalopathy. Threre is a
spontanBoi:s, conjr-rgated, irregular ierking of the eyes in aXl
directions, v.rhicfi is ralieci opsoclonus. This eye movemeni
persists during sleep and is'rnore severe when the chilcl is agitated
-['he
or tired. diagnosis ean be rnade based on clinical L:ackground,
and louk!ng for tlre c&use is Important" Looking for urinary
honrovanlliic acid and vanillylmandelie acid is very important, as
is k4lBG scan or"! bone marrow to stage the neuroblastoma. A CT
or fiRl *f ti'le chest anrd abdonren, looking for the occuit
neurobias{offra, can be carried out. Hither ACTH or oral
cortieosteroids provide pat"tial or cornplete reiief of syrnptoms In
E0% of patiet'lts, inciuding those wlth neuroblastoma. Partial or
cornplete.rerrrission sf the neurological syndrome may occur
regardiess of whether neuroblasto:,:a is present. lf a
neuroblastoma is present, i? sho, 'e removed.
52 100 Grey Cases in Paediatrics for MRCPCH

Casc 24 to perform and r


There is no risk
1. a, b, d-i, Allexcept a chromosomalstudy peritonitis, viscu
r

2. von Willqbrand disease indication for dii


3. a Fact6r Vlll restocetin assay such as irritabili
von Willebrand factors assay (hyperkalaemia,
hypercalcaemia,
von Willebrand disease dialysis.
This usually presents as mucocutaneous bleeding, but other features
may arise. Bruising after trauma is another mode of presentation. lt is Case 26
inherited as an autosomal dominant disorder but homozygous cases
do occur. There is failure in factor Vlllc, Vlll RAG, and Vlll VWF, and 1. c Acute tr
fibrinogen synthesis. The bleeding time and PTT are prolonged, and 2. c Spinal ;
fibrinogen, PT and TT are normal. Ristocetin-induced platelet e Periphe
aggregation is impaired because factor Vlll VWF potentiates platelet 3. a High do
adherence to the subendothelial connective tissue. Epistaxis is a oral cor
problem in patients with von Willebrand disease. Treatment with f lntensiv
DDAVP is useful in raising Vlll VWF in most types. Cryoprecipitate, g Bladder
nasal cautery and tranexamic acid are helpful.
Acute transYers

Case 25 This is character


tracts on both sl
1. e Blood film condition, e.g. p
d, c, b, PT, PTT, INR insufficiency, dir
f Platelet count malformation. T
i AbdominalCT perivenous disir
k Blood gas and it often foiic
aHb adenovirus). Lyr
2. Renalfailure vaccine can alsc
3. a Fluid restriction to 300 ml/m2 of surface area children older ti
c Urinary output to be added to total fluid symptoms to oc
b l.v. antibiotics sphincter paraiy
e, Transfer to specialised unit may evolve ove
weakness and .r
Haemolytic uraemic syndrome irritation with nt
This is characterised by renalfailure, thrombocytopenia and micro- with pyramida!:
disturbances y,,i
angiopathic anaemia. tt affects children at any age and there are two position. lt is ve
types.The endemic type, which affects infants and young children,
but rnost motoi'
usually follows diarrhoea. lt has a good prognosis and many cases
regions (in 80'.
are missed, as presentation can be mild. The sporadic type can affect
of patients. The
older children and no cause can be found. The prognosis is poor if it
only 25% of pa:
is not detected early and treated effectively. A blood film will show
rule out comp'3
ghost cells and red cells that look as if some parts are missing or
present and tn:'
have been 'bitten off' by something. The platelet number is low; the majority (609'" :
bleeding time will be long but the PT and PTT are normal, as is the may develop !','i
level of fibrinogen. Fresh plasma may help to restore lost volume,
the state at pr-;g
and plasmapheresis, aspirin, and heparin are useful but not very usually suppo':
effective. The treatment that will save renal function is dialysis, and to reduce the s,.,
can be peritoneal or haemodialysis. Peritoneal dialysis is much easier

i
Answers 21-30 53

to perform and no machine or highly skilled training is required.


There is no risk of transmitted diseases, but it may increase the risk of
peritonitis, viscus penetration, pleural effusion and sepsis. The
indication for dialysis in renal failure includes uraemic CNS signs
such as irritability, seizures or'semi-coma; biochemical distu rbances
(hyperkalaemia, hyponatraemia, metabolic acidosis, and
hypercalcaemia). A urea level of > 60 mmol/l is another indication for
dialysis.

Case 26
t. c Acute transverse myelitis
2.c Spinal and cranial MRI
e Peripheral nerve conduction study
3.a High dose of methylprednisolone until recovery, followed by
oral corticosteroids for 6 weeks
f lntensive physiotherapy
s Bladder catheterisation ca re

Acute transverse myelitis


This is characterised by signs of lesions of both motor and sensory
tracts on both sides of the spinal cord. Many disorders will cause this
condition, e.g. postinfectious myelitis, multiple sclerosis, vascu la r
insufficiency, direct viral infection, X-irradiation and vascular
malformation. There is inflammation and demyelination on
perivenous distribution. The most common cause remains unknown,
and it often follows viral infection (with CMV, HSV, hepatitis A and
adenovirus). Lyme disease, Mycoplasma pneumonia and typhoid
vaccine can also cause transverse myelitis. Most cases occur in
children older than 5 years. but it may occur at any age. The first
symptoms to occur are pain in the back, followed by paraplegia with
sphincter paratrysis, and marked sensory disturbances. Sometimes it
rnay evolve over days with a history of paraesthesia, then followed by
weakness and urinary retention. Some patients may have meningeal
irritation with neck stiffness. lt is characterised by flaccid paralysis
with pyramidal tract involvement. There are gross sensory
disturbances with more pain. impaired sensation and loss of sense of
position. [t is very difficult to localise the sensory disturbance level,
but most rnotcr and sensory disturbances occur in the thoracic
regions {in ES% of cases)" Respiratory involvernent occurs in one-fifth
of patients. The CSF will show pleocytosis and a high protein level in
oniy 25% of patients. Spinal and cranial MRI scans are irnportant tc
rule out compression lesions. il4oderate swelling of the cord may be
present and there are increased signals on Tr-weighted MHl. The
nrajority (S0%) of patients will rnake a good recovery. Less ihan 10o,'"
may develop MS and 10-20% will fail to irnprove. The more severe
the state at presentation, the mrorse the prognosis. Treatment is
r.isuaily supportive, and a high dose of rnethylprednisolone w,ill help
to reduce the swellrng" Treating the cause is also helpful.
tr

-iiiiFis#5 u
o
tr

,i
TNr-t cr+o o
I\f o
g$i iD
=Jl
+9 C

isi r$gg igirliflgrggrrgiH F EEi?5F


;;3{f o
()
E-e 8: o
E e 9 to,=' o
^aa=-
gE
u3 b

i
=
6'99;
(D!x
e =*E o

$it
; g 5;i i [$
iirtffiii
tii* $$g#iis; +E F
Hry
r$1:Fnail3+=
q
(o
o
9
o
a
f
o
=
:.
(-1
o

:D
C)
C)

*$ii$$rg$li{iFEii -

i$ ii$ iiiEigiF$#iF
;: iEi 3r'igrg$3ra'*$grg
r .J.;* -* f Es=5
f,; i=fr ;6 iuif*igfif,;flFifl

:54 3{e'ri: i gg U)N)J o


B $g 5;P{ *a= #r 6
;E9 r9IH[3e 3e
-ro) tro) o_
$ -o(to
ot
gt
o
\t

X
:;3 *i59ris
m-a
f
f -.-^"
?-3-
a ar 0):.-.'. d
+(oE It
lornG)r--oc)
H z€ oH gx
sl3,iiil.
+ UD v \t
slii*ary#ffff;gff$ :1
d'o'z
.<- ur- o
;g
s

^. =r
atrr
i
-. *
o- :, (D l'i+
F 3 of
JsE
0 olo1
Cht^ +
d6€ ao-
a<DY
J 9e
-or
e#sridE
=.5H;e$Bfs&
ooox9cf, 9.q=
U)ef
=
3
6'
= o
o.a
crg
-.t

;=a 6':.6:
-.d
[.sgBIsa
qo el
o-J
+Q-
o,=
:,
o
o;.
g6'
oo
jo-
='
i
d=i1 Y.-(f + o: gg ds
=: tEif $;3
E3 E rg 3 a o=
Ta 'rii'*uflgigiiiEtHi
r s=t EE, ac,
aa
oO :,
:,
=3s
-.
311.
1q -.',Uog:"
H 3i g
=l f
o
o
o
rOl
Ea
" e€ +ia's I = 6',
:j i=I s 916
d i< Q s
P'< r
o
:
o ir$irnnff$ffffg$*i
? 1o
= x' o aS E acr c
o
.]f,
r, o - -L =< -.+O
i]'tn !1
X=+
=_C/,o -<=.pec==^
i.I):-9c)[j
o
l-;r.:
r CJ -n .N.6
ri
9l==.
(J O-- cn

^ -. O aj ;;o
O L tn
=:r o {
.==E
q SqEgI.E
o

= =(/,uo)
C (/,
I?.63;
JO +
a
t\t
-'
iiffiiri$i$5ryfrg,
u; J
a-' o:lo I
(r)
(>
E3' t- o: [ 36 (rt
ut
56 100 Grey Cases in Paediatrics for MRCPCH

relative who is a known case should raise clinical suspicion. A 100 mg OUESTIONS 31-40
quantity of sweat is no longer required in places where the
macroduct system is used as opposed to the older methods, in which Case 31
evaporation led to falsely elevated sweat electrolyte levels. Sweat A 1O-year-old boy p
testing can be performed in babies who are > 48 hours old. There is a behaviour at school
normal rarge of sweat sodium of < 40 mmol/l; the test will need to be anaemia, which wa
repeated if the sweat Na result is between 40 and 60 mmol/|, and if > child in the family; r
60 mmol/|, the diagnosis is CF. The diagnosis of CF should be based is withdrawn and dr
on the result of Wvo sweat tests, not one. A false-positive sweat test 'violent child' by hir
can be associated with dermatitis/eczema, coeliac disease, untreated months, and has be
hypothyroidism, malnutrition, and flucloxacillin use. A CF diagnosis behaviour. Sometin
should always be made on the basis of high clinical suspicion' His eyes were checl
his local optometris
Case 30 showed papilloeder
in another country i
1. a Reflex anoxic seizures had arguments allt
The boy's mother h
Anoxic seizures sorted out. He threE
These are due to cortical hypoxic failure of energy metabolism as a first floor window, r
result of anoxia and hypoxia. They may follow bradycardia < 40 his iron tablets and
b.p.m., tachycardia > 150 b.p.m., asystole > 60 s, or systolic BP night shouting and
< 50 mmHg. Cortical hypoxia results in loss of consciousness and resolved by analge:
postural hypotonia. Anoxic seizures can be due to several and makes no eye c
mechanisms, which include breathhoiding, obstructive apnoea, back. He wants to g
Valsalva manoeuvre, cardiac diseases, fainting attacks and brain All aspects of the s1
compression. Reflex ahoxic seizures result from emotional stimuli, or are intact but he do
stress, minor pain or irljury. This is actually a syncope attack followed fundi examination s
by seizure. lt is usually described as the patient going pale, with eyes His BP is 120f5 mn
that may be deviated, and the pulse slow. ln some cases true epileptic results are:
attacks occur, induced by hypoxia' Many of these attacks have an
abrupt onset and tonic clonic movement and the patient becomes Hb 8'l
rigid. Postictal confusion is common and can be familial, as in MCV 45
visovagal syncope. Reassurance is usually what should be offered, MCHC 25
as well as investigation if the history suggests that this is necessary' PLT 35
BS 4.a
Urine Pr-i

1. ls his abdomir:
so, what are ih:
a Normal
b Shows ei'ii
c Abnorrna
After admission. l-e
normal. After mi:-
seizures for 5 mir-. .r

diazepam. There "i.,:


urgently, which a s:
His behaviour fi-::_
having encephal :'-.
56 100 Grey Cases in Paediatrics for MRCPCH

relative who is a known case should raise clinical suspicion. A 100 mg OUESTIONS 31-4O
quantity of sweat is no longer required in places where the
macroduct svstem is used as opposed to the older methods, in which Case 31
evaporation ied to falsely elevated sweat electrolyte levels. Sweat A 10-year-old boy I
testing can be performed in babies who are > 48 hours old. There is a behaviour at schoo
normil ra:rge of sweat sodium of < 40 mmol/l; the test will need to be anaemia, which wa
repeated if ihe sweat Na result is between 40 and 60 mmol/1, and if > child in the family;
60 mmol/l, the diagnosis is CF. The diagnosis of CF should be based is withdrawn and d
on the result of two sweat tests, not one' A false-positive sweat test 'violent child' by hir
can be associated with dermatitis/eczema, coeliac disease, untreated months, and has br
hypothyroidism, malnutrition, and flucloxacillin use. A CF diagnosis behaviour. Sometir
should always be made on the basis of high clinical suspicion. His eyes were chec
his local optometril
Case 30 showed papilloede
in another country
1. a Beflex anoxic seizures had arguments alll
The boy's mother h
Anoxic seizures sorted out. He threi
These are due to cortical hypoxic failure of energy metabolism as a first floor window,'
result of anoxia and hypoxia. They may follow bradycardia < 40 his iron tablets and
b.p.m., tachycardia > 150 b.p.m., asystole > 60 s, or systolic BP night shouting and
< b0 mmHg. Cortical hypoxia results in loss of consciousness and resolved by analge
postural hypotonia. Anoxic seizures can be due to several and makes no eye (

mechanisms, which include breathholding, obstructive apnoea, back. He wants to g


Valsalva manoeuvre, cardiac diseases, fainting attacks and brain All aspects of the s,
compression. Reflex ahoxic seizures result from emotional stimuli, or are intact but he do
stress, minor pain or irljury. This is actually a syncope attack followed fundi examination I
by seizure. lt is usually described as the patient going pale, with eyes His BP is 120[5 mr
that may be deviated, and the pulse slow. ln some cases true epileptic results are:
attacks occur, induced by hypoxia. Many of these attacks have an
abrupt onset and tonic clonic movement and the patient becomes Hb B.:
rigid. Postictal confusion is common and can be familial, as in MCV 45
visovagal syncope. Reassurance is usually what should be offered, MCHC 25
as wellis investigation if the history suggests that this is necessary. PLT 35
BS 4.:
Urine Pr,

1. ls his abdomin;
so, what 3rB ih:
a Normal
b Shows er,;:
c Abnorma,
After admission. n:
normal. After m:d-
seizures for 5 min" i
diazepam. There lrr
urgently, which els:
His behaviour fl;:_
having encephal: .
rg OUESTIONS 31-40

)h Case 31 r '

A 10-year-old boy presented with a history of lethargy and abnormal


o
behaviour at school. He is taking iron tablets for iron-deficiency
le
anaemia, which was diagnosed by his doctor. The boy is the only
child in the family; they live in a three-bedroomed Victorian house. He
I
is withdrawn and does not want to talk. He has been described as a
'violent child' by his mother on several occasions in the last 3
months, and has been expelled from a school club because of his
behaviour. Sometimes he complains of headache, with fuzzy vision.
His eyes were checked twice and no abnormalities could be found by
his local optometrist. A fundi examination during this admission
showed papilloedema. Twice a year, he visits his father who has lived
in another country for the last 4 years. His best friend left him, as they
had arguments allthe time in the last 3 months over simple things.
The boy's mother has had to give up her work until his problem is
softed out. He threatened her that one day he would jump from the
first floor window, where he sits most of the time. He stopped taking
his iron tablets and has also started losing'weight. He wakes up at
night shouting and complaining of headache. which has been
resolved by analgesia on several occasions. The boy looks withdrawn
and makes no eye contact. He hates school and does not want to go
back. He wants to go and live with his father.
All aspects of the systemic examination are normal. All cranial nerves
.or
ved are intact but he does not like to have a light shone into his eyes. A
fundi examination shows papilloedema and no other abnormalities.
/es
)ptic His BP is 120tr5 mmHg, HR 70 b.p.m., and RR 20/min. Other test
results are:
Hb 8'7 s/ot
MCV 45 ft
.1
MCHC 25 g/dl
PLT 357 x 10e/l
BS 4.5 mmol/l
Urine Proteinuria and phosphaturia
1. ls his abdominal X-ray (shown on the next page) abnormal and if
so, what are the abnormalities?
a Normal
b Shows evidence of constipation
c Abnormal
After admission, he had a cranial MHl, which was reported as
normal. After midnight, he started to have generalised tonic-clonic
seizures for 5 min, which stopped after administration of rectal
diazepam. There were no more seizures and an EEG was carried out
urgently, which also was reported to show no epileptiforrn activity..
His behaviour fluctuated and a decision was made to treat him as
having encephalitis. All his filrns were reviewed by a radioiogist,
58 100 Grey Cases in Paediatrics for MRCPCH

He was treated and 6 r


who said he may have ureteral stones and a calcified lesion in the showed that he still l',:
bladder. been some imProvern
school in the Precedin
5. What should the t
a Admit to hosl
b Contact the E
c Refer to PsYc
d lnform social
e Start treatme
care team

Case 32
A 6-month'old infant
poor feeding for the I
the spleen is tiPPed" I

there is no consangu
and there is no famil'
suffering from recurr
found for his probler
protuberant abdorne
on the l0th centile.
Hb
WCC
(b) PLT
Atk. Alk. PhPh
YGT
ALT
2. What single test should be carried out? Atb
a AbdominalUS Total protein
b IVP Bilirubin
c cystoscopy Conjugated bilirubir
d Ferritin level
e Lead level PT
f VMA Urine
g Barium swallow Stool
h Urine toxicology Alpha-fetoprotein
iLP 1. Which {our invc
j Ammonia level order?
k Mantoux test a Blood gas
3. Which other two investigations should be carried out? b Hepatitis B
a Btood film c Hepatitis C
b Urinary coPPer level d EBV serc':
c Long-bone X-raY e Human im'
d AbdominalCT f Abdomina
e CSF for HSV PCR g Bone ma"
f Repeat EEG h Blood fi!"
4. What is the diagnosis?
Questions 31-40 59

He was treated and 6,,neeks later anotl'ler blood test $/as taken, which
showed that he still had the sarne problern, even though there had
been some improvement in- his behaviour and he had attended
schosl in the preceding 2 v{eeks.
5. What should the management plan be?
a Admit to hospital until he finishes the course of treatment
b Contact the Environrnental Health Officer
c Refer to psychiatrist
d lnforrn social services and child protection team
e Start treatment again with strict supervision by a community
care team

Case 32
A 6-month-old infant girl presented with a history of iaundice and
poor feeding for the last 15 days" The liver measurement is 3 crn and
the spleen is tipped. Both parents are refugees from Kosovo, and
there is no consanguinity. An older sibling, aged 3 years, is healthy,
and there is no family history of such an illness. The father is
suffering from recurrent renal calculi. No metabolic cause could be
found for his problern. The girl is also hypotonic, with a large,
protuberant abdomen. She is passing normal stool and her weight is
on the 1oth centile.
Hb 10.3 g/dl
WCC 16 x 10s/l
PLT 90 x'l0s/l
Alk. Alk. PhPh 200 mmol/l
yGT 300 mmol/l
ALT 18O IU/I
Arb zotul
Total protein 60 g/l
Bilirubin 200 pmol/l
Conjugated bilirubin 80 pmol/l
PTT 140 s
PT 90s
Urine Glucosuria, protein ++, and aminoaciduria
Stool Normal in colour
Alpha-fetoprotein High
1. Which four investigations should be carried out and in which
order?
a Blood gas
b Hepatitis B serology
c Hepatitis C serology
d EBV serology
e Human immune deficiency virus (HlV) antibodies
f Abdominal US
g Bone marrow aspiration
h Biood film
60 100 Grey Cases in Paediatrics for MRCpCH

i Liver biopsy
j Caeruloplasmin level
2. Which two abnormalities appear on the wrist X-ray and photo?
All serology came back as negative and she was given FFp and
vitamin K. She fias referred'to a metabolic team for further testing
and they suggested some tests to be carried out locally.
3. Which order of other investigations may help the diagnosis?
a Corrected calcium
b Phosphorus
c Vitamin D level
d Chromosomes
e AAs
f Liver biopsy
g White cell enzyme
h Bone marrow aspiration
i Urine for organic acids
j Parathyroid hormone level
4. What is the diagnosis?

Case 33
An 11-year-old girl u,i
She lives with her far
home several years a
The girlattends a ma
about her behaviour ,

them and is usino ab,


school and hates-pE :
manages the after-sc'
swimmer. Her father
never works at nigh:.
occasions after conce
home, which is nov, s
problem and attenos
showed no evidence ,

A urine dipstick is s'e:


hospital 2 weeks a3:
was sent home y,.i:i -
were informed ani a

PTT 33 s
ll I iq
BT :.s
INR .a
Hb '3:
Ouestions 3l-40 61

Case 33
An 11-year-ord qirr was referred
She tives with hlr frrl:;!;g
to A&E irrtt., . history of haematuria.
home severar years ago and
t*,;:;;il siblings. Her mother teft
trre ta,rer "clre. to," a, of the chirdren.
I he girt atrends a
mai-nstrea;
about her behaviour wi*r
,;n;;i;il'n", teacher is worried
oiheicilir;;;;,
them and is usins abusive tg";;;;';'r,"L"nt. ,r she is not mixing with
schoor and hates pE and,"iri*10i""s to run awav from
ma.nages the after-sch,ool
i"-r.onr. The mare sporis teacher
club ,"Aifri"[J'she would be good
swrmmer. Her father a
q pr.t ti.n" i"[ at a tocalchip shop
never works at night. {g".
Ttre giit was ;;;; ;; a psychiatrisr and
occasrons after concern on a few
was express*O a6out her running
home, which is now settrins away from
probrem and attends H;;E;;EIorotn", has a tLarning
the sJme examination by her G-p
showed no evidence or oigrro;-;nJr,?,i",
".noor-"e'n fissure
or vatvovaginitis.
A urine dipstick is crear.a,nd.a
hospital 2 weeks aso
curture is negative. she was admitted
to
1ii1 IOOL-i""r o"iI)*rr,.,, resotved and she
was sent home with no,arrangement
were inforrned and a letter for Social services
was sent io her ".follo*_up.
Gp.
PTT
34s
TTl 1s
3T ?tr-
-i-
13 g/di
62 100 Grey Cases in Paediatrics for MRCPCH

PLT 180 x 1Oe/l Arterial gas


86 fl pH
MCV
29 g/dl EO, 5.'
MCHC . fr, 4.!
1. what are the likely diagnoses and in what order would you select BCH03 14
them? Be -'l
a UTI
Which four urg
b Valvovaginitis 1.
a l:'nmediate
Grumbling aPPendix
d Threadworms b 10 ml/kg o'
e Ureterocoele c CXR
t Constipation d Start i.v. ar
g CSA e Full septic
f Naso-phar
Shewasadmitted2weekslater,havingtakenanoverdoseof g Give propl
p"o.rti*ol in an attempt to kili herseit. ttrat day she_was at a school h Echocardir
;arb. aii; was treated foi a drug overdose and her LFT was normal 2. What abnorma
when tested on several occasions.
2. What should be the three steps of management?
a Admit frerto a Psychiatry unit
b Refer her to Social Services
c Carry out a forensic medical examination (FME)
d Refer her to a PsYchiatrist
e Temporary foster care
f Discharge her home
g Refer her to a child protection team

Case 34
A 4-day-old baby boy presented with ryanosis and shock. The Sat level
was zo% in 10 l/min oxygen via a facemask. He was born vaginally at
term and there were no [roblems in the first 3 days. Now he is not
interested in his food and in the last 12 hours has begun to breathe
h;;riry. There is no history of apnoea and he vomited once this
parents' bed and the temperature Consolidar
-orning. He sleeps in his cot by his
a
out"iOe'is below b.C. His antenatalscan shows no abnormalities and b Consolidat
two otner siblings are healthy. His RR is around 15/min and his HR is c Hyperinfla
iOO U.p.rn. His t6mperature, taken centrally- is 34qC; his BP is d Bronchial
(CRT) centrally of 4 s. Heis
uniecorOaUte, with a capillary refill time e Collapse o
,""ttr"o, cold'and sleepy. Hiswvin sister is very well and healthy. The f No abnorr
U"Oyi ,"tt er is a policewoman and his father is self-employed, and The baby was tranr
*rr'"titt ai work when the baby was brought to hospital. An i.v. cannula back and show:
*r" ini"n"O and blood was sent for analysis. The blood glucose level
*.i i.Z mmol/|, which was confirmed by laboratory results' TSH
T4
Blood glucose 1.2 mmol/l MSU
CRP 59 Nitrite N
U&E'S Normal
LFT's Normal "
All blood, urine. a.
Blood count showed neutroPhilia for herpes, aden:,
Ouestions 3t-40 63

Arterial gas
pH 7.22
EO, 5.1 kpa
rc2 4.5 kpa
cur
BCH03 14
Be -17
1. Which four urgent steps should be carried out?
a lnnmediate intubation and ventilation
b 1O-ml/kg of 0.9% NaCl given intravenously
c CXR
d Start i.v. antibiotics (benzylpenicillin and gentamicin)

f9 l.rtt septic screen, including Lp


Naso-pharyngeal aspirate t-rupel
nool g 9iyu prophylactic antibiotics to siblings and parents
h Echocardiography
^l
ot
2. What abnormalities aie visible on the chest X_ray?

t level
iy at
j'.
ne

?ture a Consolidation of left upper lobe


and b Consolidation of right upper lobe
iR is c Hyperinflation
d Bronchialwallthickening
eis e Collapse of right upper lobe
''
i The f No abnormalities
l, and The baby was transferred to the plCU; his
cannula other blood tests came
back and show:
;e level
TSH 150 mUll
T4 12 mU/l
MSU > ,IOO WCC
Nitrite Negative
All blood, urine, and CSF tesls^are ncrrnal, as
well as virology testing
for herpes, adenovirus, and RSV.
64 100 Grey Cases in Paediatrics for MRCPCH
c Viralserology
Plasma and urinary AAs and urinary organic acids are normal, and
d EEG

the baby was taken off the ventilator, but still is not well. He was
e ECG
f Urine toxicolo,
Jiignor'"0 as hiving sepsis and treated for 7 days with i.v' antibiotics' g Abdominal US
3. What are two other possible causes? h Urinary VMA
a Sepsis 2. What abnormalitY
b Hypothyroidism
c Chest infection
d Metabolic disorder
A Hypothermia
I Complex heart lesions
g lmmune deficiencY disorders
h Encephalitis
i Meningitis
j CF
k NAI rq
*1
Case 35
A girl aged 12 years presented to A&E with a history of frequent
fre"aOacfies. The headaches usually occur during the daytime and
i*rF

1
1,

sometimes in the early morning. There is no history ol vomiting, falls,


:I
1
or abnormal movements. She ii tiving with her grandmother and has ..

been in the uK for only 9 months. Both parents live in Somalia. There
i, preuious history of illness. she is not taking any.medication and
i
her^ogiandmother is healthy. There is.no history of a high temperature
or w-eight loss. Her headaches occasionally improve-with.
prn"."i"*ol. The examination reveals no abnormalities, including
iundi. Various blood tests were carried out and the results are:
Hb 13.4 g/dl
WWC 6.8 x 10e/l (N 42%, L46%\
PLT 555 x 10e/1
CRP <5
ESR 21 mm/hour
LFT and U&Es Within normal range a Dilated lateri
she was admitted for observation as she has been feeling sick since b Dilated third
the morning. Lv. fluids were started, she slept atl night a-nd o.nly c Dilated fourt
required an-algesia by lunchtime the next morning. she is still not d Lateral ventr
drinking, the 'r.v. fluids have been stopped and she planned to go e Posterior fcs
h;;; uli tne early evening. Her headache got worse overnight and f Brainstem'tt
she did not sleep'throughlhe night, even after having been given g Cerebral oe:
inr"" tvp". of analgesii. The next morning, she is very weak and h Parieto-ten ;
can,t open her eyei or lift herself up. Her fundi are normal apart from i lnternal cap:
Uirrila hatf of ihe disc margin on the left. Her BP is 110/60 mmHg, j Hypothalam
"HR 80 b.p.m., and BR 20/min. k Pontine infa
I lnterventricr
1. which three other investigations should be carried out and in 3. Which three pro
what order? diagnosis and ir
a CXR
b CranialCT
Ouestions 3i-40 65

c Viral serology
nd d EEG
e ECG
;

iotics. f Urine toxicology


g Abdominal US
h Urinary VMA
2. What abnormality appears on her cranial CT scan?

rd
g, falls,
:d has
. There
:n and
3 ratu re

jtng

a Dilatedlateralventricles
. srnce b Dilated third ventricle
.I c Dilated fourth ventricle
not d Lateral ventricular mass
Eo e Posterior fossa mass
: and f Brainstem tumour
ven g Cerebral oedema
:nd h Parieto-temporalattenuation
rl from i lnternal capsule infarct
nmHg, j Hypothalamic infarct
k Pontine infarct
rd in I lnterventricular shunt in situ
J. Which three procedures should be carried out to confirm the
diagnosis and in what order would you place them?
66 100 Grey Cases in Paediatrics for MRCPCH

a CSF for culture and sensitivity as well as PCR for


Mycobacterium 1. What shottld be oo
b Refer to psychologist a Reintubate
c CranialMRl b CXR
dEEGf e F{alf oorrection
e Mantoux test d eranial US
f CXR e Start nitrous cl
g Mycoplasma titres f Staft high-freo
h Transfer to neurosurgical ward for IVP shunt g lncrease RR to
h Give i.v. 10 ml
Her CSF shows a protein level of 1.8 g/1, lymphocytosis, a glucose 2. Why is the CO, lev
level of 2.1 rnmolfl and a serum level of 6.7 mmol/|. A Gram stain was normal Parameterl
negative. Her MRI shows only dilated ventricles with meningeal a Low RR
irritation. b Low lT
4. What is the most likely diagnosis? c Low PEEP
a Optic neuritis d SePsis
b Guillain-Barr6syndrome e RDS
c Brain tumour f From correc
d Tubercularmeningitis g Pneumothora
e Encephalomyelitis h Blocked tube
f Brain abscess The baby started hav
g Autoimmune deficiency syndrome loading dose of Pher:
3. What is this reccr
Case 36
A pregnant wornan was admitted to a labour ward at term with a
history of vaginal bleeding and abdominal pain. The CTG for the baby
showed late deceleration and an audible fetal heart sound of > 100
b,p.m.The fetal heart rate dropped 5 rnin later to 80 b.p.m. An
ernergency LSCS was done under general anaesthesia (GA). The fetal
heart (FH) sound was audible but there is no record of the exact rate.
The baby was delivered and remained unresponsive, with a HR of <
60 b"p.m. and no respiratory effort. The baby was floppy and dusky. i
lntubation was carried out and IPPV was given with 5 Umin oxygen. .,:w
Within the first minute, the l-lR was back tn > 100 b.p.rn., a UVC was
inserted and 10 rnl&g of 0.9% t{aCl given. The bai:r'y was transferred
to the Nl.lU and put on a ventilatsr as there was no respiratory effort.
The eMV test resutts are: F 20i4, tT 0.36, rate 50, FiCIr 100%, MAP 45,
HR 135 b.p"rn., and ternperature 3S"8'C. ,At the age of 60 min, the 4. ls it normai c' a:
ventilatiorl $et-{rp was ehanged to rate 50 and pres$ure 2414, and the The baby was eh1.-::
FiO, rernained at 180%. The te$t results are:
taken, which sho'', s I

he started having ::
p${ ffiOr{kPa} Iror{kP*) HCOr(kPa} Be and bradycardic :c
changes in arteria
Cord gas 6.88S 1?.4 4.0 I -19
Age 30 rnin {Vi 7.CI1 7.23 6.2 13 -t I 5. What shoulc ":
Age 120 min {A} 7.24 6.CI1 7.3 tb -10 a Reintuba:=
Age 3 hours {Ai a, 9.9 16 -2 b Relcad v' :-
Age I hours (A ) I .'Jt+ 4.5 10.'r 15 a
-1
e Give:ra -::
dLP
Ouestions 3140 67

of 120 rnin?
1. What should be done at the age
a Reintubate
b CXR
I Hrif ootrection of acidosis by bicarbonate
d Craniai US
e Start nitrous oxide (HFo)
; Si;H t,ign+requencv oscillation
t tn"r"r.E to 7o and P to-26i5
RR
X ti'trAg ot o'9v.' Naclover 30 min
U'lv;.r. iil 3 hours and all other gas
2. wt v-it ,nii cO, r"'"i-ttirr high at age of
normal Parameters?
a LowRR
b Low lT
c LowPEEP
d SePsis
e RDS
f From correction with bicarbonate
o Pneumothorax
f, Btocked tube
of seizures and was given
a
The baby started having various tvpes
i;;;i"s e;;; o1pnenodarbitone of 20 ms/ks'
3. What is this record called?

baby
l0

fetal
rate.
j I{, .i
rf< iI t[ :].
i;
sky.
,,i.+a{; dz
:{
,*;***.q-fr$ lf, -'-,, ,1,,ii,,,' , ,. rr.i:
ias
':gd
#fort.
P 45,
4. ls it normal or abnormai?
18 tror'rrs' a head US scan was
J rhe The baby was extubated at the age of
oeelema' and 2 hotrrs later
taken, which shows gl*i'fituO ferebral
he startecl having -pffiil;';;-**"u'
D*t'tut"ted to 40% of tuo'
to fit' There were no
Be ;;i;;;;t;rdic"to'oo-zo ul*ini heorcontinued
oxvgenation requirement'
il;;gJsi;;rterial gis ana rrnap'
-19 your managernent plan be at this stage?
-11 5. What should
-10
a Beintubate
-2 b Reload with Phenobarbitone with pf'enobarbltone
-2 c Give *ainte[anc" t'"ut*unt
dLP
68 100 Grey Cases in Paediatrics for MRCPCH

e Repeat septic screen


f Repeat cranial US
macular-papular rash
The baby improved and there were no more episodes of apnoea. He chest. Her tonsils are
started feeding the,next morniqg and all medication stopped after E white sheet, which is
days; he was discharged home after 10 days.
6. Whiclr abnormalities appear on his MRl, taken 1 week later?
Hb 13.1c
WCC 7xi
PLT 2SO )
Bilirubin 19 m
ALT 32IU
Atk. Ph 150 t
CRP 25
Blood film No a
Throat swab Neg:
1. What is the most
a Kawasakidisr
b Follicular tonr
c Glandular fev
d Adenovirus ir
e Viral hepatitis
f TuberCular !.i
g Histocytosis
h Rubella virus
2. What one investig
a Echocardiogr
b Lymph node
c Mono spot te
d lgM for EBV
e ESR
7. What is the prognosis?
f Skin biopsy
a Poor
g Throat swab
b 50% chance he wilt develop hemiplegia
h Repeat blood
c 1G-15% chance of normal development
d Good Case 38
e Speech problem A child aged 10 was a
present for the last 24
Case 37 3 hours. The surgeonr
was to be reviewed lr
A family doctor referred a 12-year-old girl with a history of illness for and she was kept nii I
7 days. She had mouth ulcers prior to her present febrile illness. She overnight and she rt.a
has had a high temperature on and off for the past 7 days. She who prescribed more
developed ceruical lyrnphadenopathy more on the left side as well as reviewed her in the m
on her left axilla, and a small lymph node measuring 1 x 1 cm on the and the surgeon ag.e.
left of her groin. She also complained of a sore throat for the first 4 hemicolectomy was :
days and had puffy eyes, which have settled down now. She has appendicular mass. Ti
never had conjunctivitis, or swollen palms or soles of her feet" Her involved. The test resr
spleen measures 1-2 cm belowthe left costal rnargin and she is very
slim. On the seventh day, she developed a very small' Hb 12.2
WCC 15 i
PLT 56C
Ouestions 3i-40 69

macular-papular rash_, which is distributed on her back


and upper
chest. Her tonsirs are rarge, inframed, toliicrtateo and
covered- with a
white sheet, which is trrole on the,righisiJe tfran on
the left.
Hb 13.19/dl
wcc 7 x10s/l (50% N, 50% L)
PLT 250 x 1@/l
Bilirubin 19 mmol/l (conjugated 1%)
ALT 32tufl
Ark. Ph 150 tu/l
CBP 25
Blood film No atypical lymphocytes
Throat swab Negaiive
1. What is the most likely diagnosis?
a Kawasakidisease
b Foilicular tonsiilitis
c Glandular fever
d Adenovirus infection
e Viral hepatitis
f Tubernrrla'
. __-: ve;_, l.
g Histocytosis ifi.iphadenopathy
;

h Rubella virus infection


2. What one investigation is needed to confirm the diagnosis?
a Echocardiography
b Lymph node biopsy
c Mono spot test
d lgM for EBV
e ESR
f Skin biopsy
g Throat swab
h Repeat btood film
Case 38
A child a-ged 10 was admifted with right iliac fossa pain
present for the last 24 hours, and haJbeen getting that was
worse overlhe rast
3 hoyrs.. The surgeons raised trre possiuitity-ot a rirr
uno tnu .nio
was to be reviewed the next day. hnargesia and i.v.
fruids wlie-given
and she.was kept nilby mouth.-Her abiominalpain goi*orr"-'
le overnight and she was seen by a surgical senior house
officer (sHo),
who prescribed more anargesia tmor"pr,inet. The paeoiai.iclea*
AS reviewed her in the morning, a diagnbsis of appendicitis
IE *ai maoe,
319-ilu."u.rgeon agreed witl tnis. 5n" *"i operated on and a
nemrco,ectomy was carried out due to adhesion following
an
appendicular mass. The ireocaecar part of the intestine
is-atso
involved. The test results are:
Hb 12.2 gldl
WCC 15 x 10s/t(N 70%)
PLT 560 x 10s/t
7

70 100 Grey Cases in Paediatrics for MRCPCH

Normal K 4.2 m
U&E U 3.8mr
MSU < l0leukocytes and no organisms
LT 44 mr
in order?
1. what are the,most likely three differential diagnoses ALT 38 lU,
a APPendicitis APH 126 rt
b Crohn's disease Atb 32 gll
c lntestinaltuberculosis Glu 5.3 m
d lleocaecal abscess Lactate 2.1 m
e APPendicular mass NHo 40 mr
f Perforated aPPendix Venous gas at 10.00 p.
g Ulcerative colitis
in pH 7.38
2. ivhi"r, two invesiigations will help to reach the diagnosis' ffo, 2.4k1
order?
a lmmediate ESR
rc2 8.4 kg

b UPPer GIT endoscopy


Heo3 M
c Mantoux test
Be -13
Urine organic acid
O Hi"toputhology of the surgical mass Serum AA
e Lower GIT endoscoPY Urine for reducing sut
f Abdominal US Cranial CT
g Abdominal MRI Ketonuria
1. What is the possii
Case 39 a Benign neon;
AS-month-oldboywasadmittedtoageneralpaediatric.wardwitha
b Benign infani
$-10 m.in of his c Benign myoc
nl"totV of jerking episodes in the last tO days' Within d lnfantile spas
ooinq to sleep, nis righi ir* .ta.ts ierking, with his fist clenching, and
e Myoclonic as
to do the same, which is
Yriiii" . t"* I".onoJr,i. teft arm siarrs wakes up f Myoclonic en
movement in his legs' He never
iolio*rO by this type of 2. Which other tes:s
andthereisnog"ne,alisedtonic_clonicphase.Hismotherholdshis
but ihis Jo". not stop the movement. on one occasion
his a Cranial MRi
"r. *u.Involved; this lasted
"ii"g, only few seconds' The longest b Repeat sleeo
i.." a
continuous c Video teleme
Juration of these episodes was 3 min' but this was not d Nothing
interrupted by holding one of his limbs'
anA coutA be e Repeat meta:
There were no other concerns about him; he smiled
at the age of 7 3. What treatment s
*i"ti,'nr" good head control and is able to roll from front to back' a Vigabatrin
Hesitswithsupportandtherearenoconcernsabouthisvisionor
'f,""iing. b Sodium vaip'
He has'no tirtfrmarfs and there is no history of epilepsv
in c Lamotrigine
irru tutiirv. He has one sibling, aged 4, who is in good health' d Nothing
e Phenytoin
Hecansmilewithnofacialasymmetry,followobiectswithoutand
pi6r"* his eye r.no'"rnuntt are all normal' His power' tone and
I"n""". "^A
are all intacl and equal in the upper and-.lower limbs. He is Case 40
,ut"toreachforanobjectwithouttremorordifficulties. An infant aged 11 r:
other test results
His asleep and awake EEG were reported as normal. solids and SMA Goic
are: rice and pasta. He is '
His father works long
Hb 11.6 g/dl his mother is a house
wcc 7.2 x l0sll bedroomed terraced
PLT 250 x 1Oe/l with juvenile chroni:
Na 138 mmol/l
Questions 31-40 71

K 4.2 mmol/l
U 3.8mmol/l
Cr 44 mmol/l
ALT 38 tun
APH 126 tu/t
Alb 32 sll
Glu 5.3 mmolil
Lactate 2.1 mmol/l
NHo 40 mmol/l
Venous gas at 10.00 p.m.
pH 7.38 pH 7.40 at 10.00 a.m.
ffiO. 2.4 kpa EO, 4.5 kpa
rc2 8.4 kpa rc2 9.6 kpa
HCO3 14 HCO3 16
Be -13 Be -0.3
Urine organic acid Normal
Serum AA Normal
Urine for reducing substances Normal
Cranial CT No abnormalities
Ketonuria Negative
1. What is the possible diagnobis?
a Benign neonatal familial convulsion
rha b Benign infantile convulsion
f his c Benign myoclonic epilepsy of infancy
1, and d lnfantile spasms
e Myoclonic astatic epilepsy
f Myoclonicencephalopathy
; his 2. Which other tests may help the diagnosis?
on his a Cranial MRI
b Repeat sleep EEG
,JI c Video telemetry
d Nothing
e Repeat metabolic screen
3. What treatment should be given?
a:k. a Vigabatrin
JI
b Sodium valproate
;v in c Lamotrigine
d Nothing
and e Phenytoin
e and
te is Case 40
An infant aged 11 months presented with a history of FTT. He is on
results solids and SMA Gold, up to a quantity of 600 ml/day. He likes boiled
rice and pasta. He is not suffering from diarrhoea and has no rash.
His father works long hours as a cook in a local lndian restaurant and
his mother is a housewife. The whole family of nine live in a three-
bedroomed terraced house. one of his oldbr siblings was diagnosed
with juvenile chronic arthritis (JCA), and is on treatment and
72 100 Grey Cases in Paediatrics for MRCPCH

responding very well. His sister, who is 11 years old, started her 3. a Blood fitm
period 2 months ago and has been diagnosed as having mild c Long-bone X-re
anaemia. The parents are struggling to pay their mortgage. 4. Lead poisoning
His weight is 5.9 kg (on the 10th centile), his height is 84 cm (on the
5. b Contact the Enr
25th centile) and his OFC is 45.6 cm (on the 50th centile).
e Start treatment
community car
No abnormalities are found upon systemic examination, but his nails
are brittle and a pansystolic murmur can be heard on the left sternal Chronic lead poisoning
border. Other test results are: This can occur after che,
Hb 4.3 s/ot windows or doors. Leac
WCC 7.9 x 10s/l(N 45%, L 55%, E 1.3%) fishing, and old water pi
PLT M5 x l}sll southeast Asia and the s
PCV 35 fl using Surma, an eye ma
Ret 1.4% a compulsive eating dis(
MCHC 19 g/dl diagnosed as having iro
MCV 56 fI to respond and the clinir
Serum ferritin 'l be anaemic, complain o.
rBc 30% behavioural problems ai
performance. Those w,iti
1. What are the three most likely diagnoses in order?
convulsions, irritabil ity a
a Thalassaemia major and death. The lead leve
b Thalassaemia minor hypoch romic, m icrocyt' :
c JCA
bone X-ray may shorv ie
d lron-deficiencyanaemia
long bones, and some:;-
e Spherocytosis
ray. Lead poisoning is or
f Lead poisoning
pressure, and papilloecs
g Coeliac disease
affected environment as
h Crohn's disease
Officer is very impor-ier:
i Ulcerative colitis
penicillamine in mild cas
j Cows' milk intolerance (EDTA) in severe cases
2. What would you expect to see on the blood film, in order?
giving dimercaprol as a-
a Spherocytes
b Microcytosis
c Elliptocytosis
d Anisocytosis Case 32.
e Target cells 1. f Abdominat US
f Basophilic stippling b Hepatitis B sero
g Howell-Jolly bodies c Hepatitis C sero
h Sideroblastic red cells Blood film
i Hypochromic red cells 2. On X-ray - cuppir-;
j Spindle-shaped red cells ostecDi
On photo - rachiti:
3. a Calcium
b Phosphorus
ANSWERS 31-40 c Vitamin D level
j Parathyroid ho.r
Case 31 e AAs
h Bone marrow aj
1. c Normal f Liver biopsy
2. e Lead level 4. Tyrosinaemia
Answers 31-40 13

s her 3. a Btood film


"td
- 9 Long-bone X_ray
4. Lead poisonino
': ron the
5. b Contact tiie Environmental Health Officer
e Start treatment again with .tr1.i by the
community care team "rp"*ision
;i his nails
efi sternal Chronic lead polsoning
This can occur after chewing or sucking
lead paint from old
windows or doors. Lead frolr Uurning -Uatieries,
lead shot for
fishing, and old water pipes p"r"; ;iiltll
rist. Chitdren from
southeast Asia and the subcontinent may get
lead poisoniiliro*
using Surma, an eye make-upJh"/rr"
LJuarrv known to hive'pica or
g.compulsive eating disorder. They'are uiuaily'ana"n,i" .no .1"
dragnosed.as having.iron deficieniy.n."rniu
to respond and the crinicar situation changes. Theyon tr"rt."r,t t.it
and
be,anaemic, complain of abdominar pain,"f,eaoache,
wiil coniinr" to
and have
beh-avioural probrems arong with a creieiioratiori
performance. Those with sivere poisoning in schoor
may present with
convulsions, irritabirity and unconsciorinEss,
and death. The read rwer can u" n","u.ui"i
*nicn may iead to coma
ano a brood firm wiil show
hypochromic, microcytic red ceils, ana olsopr,iric
bone.X-ray may show read rines,in,hicn-#ect
stippring. The rong-
mainry the ends of the
long bones, and sometimes read fl;k;s
ri'ay ue seer"1 cn abdominar X-
ray' Lead poisoning is one ortne cause.'ol
pressure, and papiiloedema is common.
in"r"rred intracraniar
Removar of the chird from the
affected environment as weil as intorminj
the Env,ronmental Hearth
officer.is v.ery important. crretatingig"nt".
be used such as D_
p-enicillamine in mird cases and etiyt6neoiaminetetraacetic
"*n acid
(EDTA) in severe cases. The
'ie effectivJr"r. -
giving dimercaprol as an i.m. injection. -- "t
r? can te'en-tia-nceo oy
=DTA

Case 32.

1. f Abdominat US
b Hepatitis B serology
9 Hepatitis C serology
Blood film
2. On X-ray - cupping, widening and flaring of epiphysis
osteopenia
On photo - rachitic rosary
3. a Calcium
b Phosphorus
c Vitamin D level
j Parathyroid hormone level
e AAs
h Bone marrow agpiration
f Liver biopsy
4. Tyrosinaemia
74 100 Grey Cases in Paediatrics for MRCPCFT
acquaintances or sitra
always be a high degr
Tyrosinaemia cases in children. Eac
dealing with children
This is one of the tyrosine metabolic disorders: tyrosinaemia type 1, guidelines as well. ln
Wiosini"11ia tfpe 2, alkapttrnuria, and others. Tyrosinaemia leads to should be sought, as
;;6h ;i;ila ievet'of tyiosine. tn rype l,lhere The
wi1 be hepatocellular
affected person usually including the psychiar
ar,,.'igE, ;"uking in Fanconi syndrome.
are very important.
oi"."iti with FTi, profound rickets, thrombocytopenia andmay an
occur.
SUnormat clotting disorder. Hypertrophic cardiomyopathy Reference: Kempe X.
There is an increised risk of l'repatic carcinoma in patients who problem. The 1977 C.
.r*ir" t" adulthood. As a result there will be abnormal liver and 382-9.
fi.i"irv tl"&ion, with a prolonged PT that will not respond to vitamin
'i. are usually high. The
ih; pf"",,u tyrosine and methionine levels
Case34
."** itpt level is very high and there is generalised
"+etbprotein
no""iAutia and tyrosyl u ria. Gamma-a minolevulinic acid excretion 1. a lmmediate int
"rni
G increased. Neonates with hepatitis or older children with a family b Give i.v. 10 m
t i.to.v of tyrosinaemia should be screened for this condition, d Start i.v. antit
espe"faf fV ii there is hepatocellular dama.ge. Confirrnation is by h Echocardioori
;il;;j55ay in cuttured fibroblasts. A diet low in phenylalanine and 2. f No abnormlti
tyrosine is imponant in these patients, but liver transplant surgery 3. b Hypothyroidis
iiu". ifi" only chance of survival in severe cases. NTBC is effective in a Sepsis
ieversing liver damage by reducing the level of succinylacetone to
normal. Hypothyroidism
This is the commones
Case 33 problem worldwide, a
Complete absence of t
1. S CSA., endemic hypothyroidir
a UTI cases but can be famil
d Threadworms causes of thyroid dysh
b Valvovaginitis responsiveness and fa
2.c Carry out a FME (urgentlY)
iodide organisation an
b Refer to Social Services
syndrome. Not allthyr
s Refer to child protection team
loss of thyroid tissue, i
enlarged. TSH deficier
Child sexualabuse abnormalities is a rare
Child sexual abuse was defined (Kempe 1978, p' 382) as the features of hypothyroi
inuofu"rn"nt of dependent, developmentally immature children and tongue, hoarse cry, far
adolescents in sexual acts that they do not fully consent to, that they mottling, cold hands a
,i" not able to give informed consent for, or that violate the social with time. There are nr
taboos of family roles. A history of child sexual abuse is not hypothyroidism, inctuc
,n"orn*on among the general population, and the figu!9 is as high gestation longer than r
;;ab% in girls anJ tSy"ln boys. Females on average suffer sexual o{ stool, respiratory distrr
;b;;";ig;ificantly more often than males. The most common type large anterior fontanei,
sexuat in males and females is fondling of the genital areas' should be done at I v,.t
"6u.umost common form of sexual abuse of males involves
The second mother's thyroid slatur
in" p"*or*ance of oral sex, while abuse of females may vary from diagnosis can be confir
removal of clothing to direct genital contact. There is a strong TSH of > 20 mUfl. Mea,
association between physicaiand sexual abuse. One-third of scan will help to localis
pf,Vri"if fV abused cfritdren may be.sex.ually abused and 12-15% of neck will help to confir
le>iuarrv auused children may be physically abused. The main
p"tputtrtot" of sexual abuse of girls and boys are males who are
Ans,wers 31-40 75

acquaintanees or strangers, rather


than relatives. There should
be. a- high deglee ot srspi"ion-in
?L*"v: oraer to prevent sexuar abuse
cases in chirdren. EaLh trust rilrlJ h;;"-i o*n guiderines
for
1, dealing with chirdrel
ylo
guidelines as wer. rn cdses in
*" " and there are nationar
r*r.rrv'Juusec
to *r,i"n otiit exists a second opinion
shcutc be sought, as many experts .""
rla r
incluCing the psychiatric genitbu ri"r.,
h;l;.;;t]"*-_r;;i;"f,
rlly iCUf clinic anJ-cornrJiring,
are very important.
cur. Reference: Kempe X- 1gg7. sexuar
abuse: another hidden paediatric
The 1e77 C. Anderson Aia-ii.r, i".t ures. paediarr,fieitst
nin
ffi:l:.
te
A Case34
tion 1. a lmmediate intubation and ventilation
ilv b Give i.v. 10 mUkg of 0.97o N;Ci -
d start i.v. antibiot-ics (benzytpenicilrin and gentamicinl
h Echocardiography
and 2. f No abnormilities'
Y 3. b Hypothyroidism
ve in
lo
a Sepsis

Hypothyroidism
This is the commonest treatabre cause
problem worrdwide, a.nd is.most tr"qr""tivof mentar retardation. rt is a
due to iooin" Jeici"n.v.
complete absence of.thyroid t;ssue L t-rre commonest
endem ic hypothyroidl?T .-Il" aetio.t o9y cause of non_
iemains unt nown in-rn"ny
cases but can be famiriar and associatli
with oo*n rvnJio,,".'ort.,,
causes of thyroid dysh.ormonogenesis inciude
a oecrease in rin
responsiveness and failure to c-oncentr"t"
iooiou- ft," ot
iodide organisation and-sensorinuul"i Jltness "."."i.iion
pendred
is cailed
syndrome. N.ot allthyroid. dyshormono!"neri,
is associated with a
loss of thyroid tissue, and in ,o."
tr"ryroid gland will be
""rl"rlhe
enlarged. TSH deficiency due to pituirary or hypothalamic
abnormalities is a rare cause oilivpotr,v'ro-idisir.
rvranv ciassic
features.of hypothyroidism are n,iit*iO-in
and newborns. A large
tongue, hoarse cry, facial puffiness, umUiticat
: they hernia, hvpoio-nia,
mottling, cold hands an_d ieet,.ana.ietnrigv are
;ia I
with time. There are non-specific
subtle.'"Jrpp"",
may suggest
hypothyroidism, incruding'uncon;uiateJ'rrvperoirirubinaemia,
"ignr1l.,1i
high gestation longer than 42 weeks, cier"ayeo passage a
ual of rneconiu-mino
stool, respiratory distress, severe infection, teeiing
Vpe of large anterior fontaneile. Neonatai r.rl"ning
oiffi"riii"., i"o ,
:as. tor tripottrv;"iJiJ;
should be done at 1 week oj age ,=-"rtti"i't"sting
ves rnay refrect the
mother's thyroid status and giie a tatie-negative
'rom result. The
diagnosis can be confirmed 6v, c*"r"use-ii prasm;i;'";;;
TSI'I of > 20 mU/r. Measurement rise in
of 13 is of littre varue. A radionucride
scan will herp to rocarise and measurl the
tof size otir," glundl'tiiit tt
neck wil! hetp to confirm the absence
;; thvroid liano. i[ir"io "
rfe
"i
76 100 Grey Cases in Paediatrics for MRCPCH
as.base-line cortisol' GH' LH and
antibodies should be tested, as well h.v. 10 ml/tg of 0.996
thvroxine (L-T/. should staT-11soon
;$i:-R$t;"."nt theiapv withGenetic 2. f, h CO2 high from ac
i, counseiling is important and 3. Cerebral function mo
as the diagnosis
""nnIri.,"J.
the prognosis is exce!!ent' 4. Abnormal. There is e'
5. b Reload with pher
c Maintenance witl
Case 35 g Repeat cranial Ul
1. b Cranial CT 6. MAC|
a CXR Bright basal ganglia
f Urine toxicologY lnternal capsule invol
2. a Dilated lateral ventricles 7. 10-15% chance of no
3. c Cranial MRI e Speech problem
e Mantoux test
as PCR for
CSf for cult]u're and sensitivity as well Neonatal seizures
" MYcobacterium
4. e Tubercular meningitis Neonatal seizures are po(
distinguish from normal i
Tubercular meningitis or hemispheres. The pos
the leading causes of death in abnormality. Seizures are
Tubercular meningitis remains one of tuberculosis has been desaturation and an incre
children. The numbe, oi"ur". of
pulmonary
cases occur in areas where help to diagnose neonata
increasing in ,ecent-yelrs, and more
first in the lungs and then diagnose neonatal seizur
sanitation is poor. iuUercutosis occurs months' The symptoms same time recording any
6
spreads to the rn"ning"t, which.may-take The presenting in newborns is usually w'
are insidious ano maii"[e months to apDear'
be a high degree of suspicion the newborn is clinically
symptoms .r" u"ri"i'bJi'tt"t" trt""ro
othLr patientsvrrith TB or
in children wno nail ;;;i; contact with The seizure patterns in n
areas. Most often low-grade
who have travetteit niir,iv'""o"mic irritability' which is
include:
followei by listlessness and
fever is present,
commonest feature; it is e Apnoea with tonic s:
mainly due to r,""0."Ju. n,j.a..r," is the
o Focal clonic movem(
progressive nliijilui;i;J bt analsesia. vomiting and abdominal o
vomiting Multifocalclonic lim
pain may o""no,."o"]ril;ffii;thiJ condftion' Headache andlater' o Myoclonic jerks
increase in trequenlv irt" signs of meningitis.appear
"no in 3o-40o/o. of cases. Seizures may appear o Paroxysmal laughirr
i"i"Ui"f intarciions occur is established' The level of o Tonic deviation of :.
early, but usuallv ;";;i;; utt"i *eningism s Tonic stiffening of ;ts
focal neu rological deficit'
consciousnes, oeJr-i nJs-irogi;s.irerv,-*ith
and tremiparesis may follow' and The differential diagnosi
Papilloedem",
death occu* it "runi,inJ'iJparnies
noi'il""t"o fn" cause of hydrocephalus is the for treatable causes is r:
"Iifv. the choroid plexus due to
;;;;ili"" ot ausoipiion of CS'F bv r,ijn of protein in the cSF'
viral), electrolyte imbaiar
deposits ot intectJi"m"il,iii""al "ont"nt
inborn errors of metaco.
and predominant
The csF witt stroil wcc or 100-250 cells/m2 protein levels' AFB may abnormalities, familiar ^
lymphocytes witn'tlw'gluctse and very
high
'd" tubercular Stopping the seizures
d"t".t"O in tkre CSriut a culture will diagnose
s

tubercular meningitis' The administration c+ :-


meningitis. A pci li'av *iil ;r"rp in diagnoiing at presentation' will help to stop seizure:
Recovery is made [v tilirot"n *L'o a'"
iot comatose
regions is as high asZoY* therapy by administen ":
The mortality rat; in levels to achieve an effe
"o*" phenobarbitone or phe.
clonazepam infusion w.'i
Case 36
can be stopped most of
1. c Half correction of acidosis with bicarbonate continue and subsequer
Answers 31-40 77

and
;oon h.v. 10 ml/kg of 0.9% NaCl over 30 min
? i h COr.high from acidosis
lnd
r. LereDrat function monitorinocorrectio;
(CFM)
or blocked ETT
4. Abnormal. There is evidencj of seizure activity
5. b Reload with phenobarbitone
c Maintenance with phenobarbitone
g Repeat cranial US
6. MAC|
Bright basalganglia
lnternal capsule involvement
7. 10-15% chance of normal development
e Speech problem
Neonata! seizures
Neonatal seizures are poorry organised and difficurt to recognise
and
distinguish from normar activityl Thev may arise from the blainstem
or hemispheres. The possibirity of sei:urei increases witrr uriin
a'th in a.bnormality. Seizures are usually acccmpanied by a cfrange
in Hn,
.'een desaturation and an increase in systolic UttoO pressure. fEC
may
'e help to diagnose neonatar seizures. Video tere,.netry is the
blst way to
diagnose neonatal seizures: marking the evenis on video and
at the
)lOmS i n q.a nv E E G a b n oim a ities. The
:_. T: -19 :-r-"9-ord I epi ieptif o rm activity
rn newDorns is usually widespread and can be detected
even when
cicion ihe newborn is clinically asymptomatic.
rB or
The seizure patterns in newborns appear in different forms,
rade which
include:
rls
it is . Apnoea with tonic stiffening of the body
Jcminal o Focal clonic movements of one or both rin-,bs on both sides
nniting o Multifocal clonic limb movernents
o Myoclonic jerks
rDpear o Paroxysmal laughing
r level of o Tonic deviation of the eyes, upward or .ne side
I Ceficit. e Tonic stiffening of the body 'o
ow, and
is the The differential diagnosis of neonatal seizures is varied, but looking
for treatable causes is very important. Rule out infection (bacterial
I or
thE CSF. viral), electrolyte imbalance, lVH, cerebral haemorrhage, drujs,
tnt inborn errors of metaborism, HrE, cerebrar dysgenesiJand otlrer
brain
IFB may abnormalities, familial neonatal seizures, and TS.

eningitis.
llopping the seizures is important. but this depends on the cause.
The adrninistration of phenobarbitone or phenytoin as an infusion
)ntation. will help.to stop seizures. lt is very important to give maintenance
therapy by administering these diugs'via i.v. can-nura and measuring
levels to achieve an effeitive dose" [rewborns who do not respono
phenobarbitone or phenytoin will require ventilation, to
and a
clonazepam infusion will help to stop seizure activity. Muscle twitches
can be stopped most of the tinne, uui abnormal Urain activity wiit
continue and subsequentry wilr either stop or. evorve into another
78 100 Grey Cases in Paediatrics for MRCPCH

type of seizure with a different type of brain activity. Finding the pain is usually periur-
cluse is very important, as treatment with anticonvulsants v'/ill child will have a coale
;;;;rJ on finding the cause and treating this if it is treatable. The not be able to walk ur
prognosis also d"epends on the cause' onto the table for exa.
and rebound phenom
iliac fossa. No singie c
Case 37
appendicitis; they can
1. c Glandular fever (infectious mononucieosis) There should always i
2. d lgM for EBV not be delayed as the
perforated appendix. .
Glandular fever {infectious mononuc!eosis} appendix with surrou.
appendicular mass ca
Thisisadiseaseofolderchildrenandyoungadults.andanalmost disease and terminal
iOenti"at clinical picture can be caused by CMV and Toxoplasma include mesenteric ac
iindii.The initial presentation is usually not much different from ovarian cyst, Meckel's
if,-ui oiu["tv other URTI. lt is characterised by fever, sore throat.and pneumonia, Henoch-S
;;l;rS"J iervicat tyrnphadenopathy..lt is associated with exudative with cell crisis.
tonsil'iitis. There are many cornplications that may be associated
ifri. rir.i i"tection, I ncl udin g aseptic meningitis, tra nsverse myelitis,
g"ff't palsy and Guillain*Ba116 syndrome' Myocarditis' Case 39
"na
;;[;;;d"ly lnd a ruptured spleen are other complications' and 1. c Benign myo:
oltients s[ould be warned against doing any contact sports if there 2. b Repeat slee;
i. .pf .no*"galy. Frolonged illnesses may cause fatigue, lethargy 3. d Nothing
und fr"qr*niabsence from sehool. The Paul*Bunnelltest is a
,uui".tir" test and the antiboclies rnay tske 2-3 weeks to develop. Benign myoclonic epil
inl.p".iti. EBV lgrvl antibody test is more readily available and is
used io indicate current infeciion. The blood film will show atypical This condition is ve-,
,no"o.G.. There is no specific treatment and general advice for rnonths and 3 years :
pain and temperature should be given. children with chronic fatigue generalised fliyoc,-- - .
and lethargy should be helped at school and home' A repeatedseveralt -=.
multidiscifli nary tea* inctuding a psychologist, physiotherapist and wake up. The jerks ,..
p""Ji"tii.irn should ,r.,cili tigeiher to help the chiicj as rveli as the and can be felt. Tn: l:
family involved' spikes or polyspik:s :
usually idiopathic.'. :
cases, unless it ca _: ,
Case 38 = -
case drugs may he l
1. e Appendicular mass for this condition t: ;:
b Crohn's disease affect the developr ='
d lleocaecal abscess
2. d l-listopathology of the surgical mass Case 40
c Mantoux test
1. d lron-deficj=-:
Appendicitis h Crohn's d s==,
Thisisaverycommonconditioninwhichdiagnosiscanbevery
g Coeliac dis=:..
Oi*icrft, if in doubt surgery is advised. An emergency surgical
2. b Microcytos s
as it lift untreateci, the appendix mav perforate
i Hypochrcr- :
;;;;;* "ndis needed, of an d Anisocytos .
and cause many problems, or may cause thc developrnent
rnay resernble crol'ln's disease- This is still a
appendicular mass that lron-deficiency anaer
with a significant rate of.mortality and morbidity' lt usually
.iriiOition
pi"u"nt, with abdJminal pain, fever and vomiting. The abdominal This rs one of the r: - -

.at
I
B
x
q
.1

:
Answers 3'l-40 79

0,a1,n.is usually periumbiricarand radiates to


the right iriac fossa. The
child wilt have a coated tongue. ue i"rii;r;
uecauie ot irr" i"i", *irr
!?l b:theabte. to walk qpright ino wirr rinJli r"ry difficult io
onto table for examination. There *iiiU" tenderness, "il_U ,p
and rebound phenomena wi, be po"iti"" guarding,
examination iGe rigr,t
iliac fossa..No.single blood or irai.rf "t wiil diagnose
appendicitis; thev can herp but "gi".itest
There shourd arways be a'nijrr "r".v
irr" can present differentry.
i"o-"i oi.r.picion, and surgery shourd
not-be delayed as the.rate oimoruiaity
perfora.ted appendix. US is helpfui
ii f,i'gf, in prt;enii uiitf"i,
in-i""ogniring a thickened
appendix with surround.ing oedema. rtrlEtrer"iti.i
r ost appendicu lar mass can i nc-r ude i teocaeca t
Jirgnois
-iron of an
iu bercu rosis, n;r-
disease and terminar ireitis. conoltions
associated with appendicitis
lm i ncl ude mesenteric
aden itis, g".ir;"nl";iiL, constipati on, torsion
and ova rian cyst, Meckel,s- d ivertiiu I iti s,
sa lpi n g itis, UTi, ri g h; b;;;i
etive pneumonia, Henoch-Schonlein priprri,
dlabetes mellitus and sickle-
:d with cell crisis.
re litis,

Case 39
and
:here 1. c Benign myoclonic epilepsy of infancy
,l'gY 2. b Repeat sleep EEG
3. d Nothing
llop.
nd is Benign rnyoclonic epilepsy of infancy
yprcal This condition is very-rare and occurs in
for infants between the ages of 4
rnonths and 3 years. As. the child falls asteep
f atigue generalised myoclonic jerks of tne timoi
an,i within f O_SO"min,
wiii r,-cur, *r-,i.r,-.n-.v'u"
repeated severar times. rt wiil not affect
rrst and sreeo ;. cause the chird to
wake up' The jerks wi, not stop if ror."o"L rr
s the rrds the chird,s hand
and can be felt. The EEG is usually normal,
i:.
spikes o.r.polyspikes during tfre iniericrai r-,u.
.,.,uy ,f,o* g"nJratised
f This condition is
usually idiopathic. No further tests or ,r*"-t,,,,
are required in most
cases, unless it causes problems or anxiety
case drugs may help. The prognosis is vei, g
rn ,e tamiiv, anO in this
.rd and it'i. u"rv r.r"
for this condition to deverop tJ gL;"."rilud
affect the development of the cfita.- -"-'- tonic-cronic seizures or

Case 40
. d
'l
lron-deficiencyanaemia
h Crohn's disease
r€ry g Coeliac disease
r:gical 2. b Microcytosis
rcrforate i Hypochromic red cells
an d Anisocytosis
is still a
usually lron-deficiency anaemia
n'rinal This rs one of the commonest types of anaemia,
affecting chirdren in
r-
t
80 100 Grey Cases in Paediatrics for MRCPCH

inner cities more than in rural areas. Poor nutrition and diet play a OUESTIONS 41-50
large part in causing this in children. Poverty and social problems are
cofactors, when healthy food is not affordable. other causes of iron- Case 41
deficiency anadmia are impaired iron absorption and excessive loss A mother brought her 4
of iron due to bleeding. lnfestation with worms is one of the febrile. He was seen 2 vr
commonest causes of iron-deficiency anaemia worldwide. Chronic was diagnosed as sebor
blood loss from the gut due to inflammation secondary to delivery (FTND) and has
inflammatory bowel disease or chronic infection will lead to iron- He is the first child born
deficiency anaemia. Any chronic blood loss from any part of the body history of illness. He wa
will lead to iron-deficiency anaemia. The affected child will look pale carried out. Two hours i
and tired, and this will affect their performance in school. Looking for seizures in his left arm.
a cause is important. Dietary intake and nutritional history for all
three times. He was giv,
family members is very important. When iron supplementation is continued to have focal
started for treatment of iron-deficiency anaemia secondary to dose of phenobarbitone
insufficient dietary intake, allfamily members should receive the tonic-clonic seizures an
supplement, and support from the dietitian, health visitor and if further m6nagement. Hr
necessary a social worker is needed. Oral treatment is the starting
seizures. The test result
point of treatment, and if there is poor compliance and no other
cause can be found, i.v. treatment should be considered, although Na 134 mr
this is rarely needed. Recovery is usually very good, especially for K 4.3 mn
children attending school. U 4.1 mn
Ca LM m
Mg 0.99 m
Glu 5.8 mn
Bilirubin 42 ymt
Urine pH 6
RAS Negati
ALT 24lU1
Arb 38 g,4
CRP <5
ESR <2mn
MSU Negati
Stool No grc
CXR Norrn a
Cranial CT No abr
Hb 14.2 g
wcc 17.2 x
PLT 459 >r ''l

PTT 39s
TT 1s
lNR 1.3
csF
wcc 100 (7:
RBC 15
Protein 0.99 g'
Glu 2.4 mn
No organism
Serum glu 4.5 mn
NH. 28 rn rr
OUESTIONS 41-50

Case 41

l.tg*'gt brought her 4-week-oti uoy to A&E, as he was irritabre


febrile' He was seen 2 weeks previousty, *itn'" rristory orrisrr, and
wrricrr
wa.s diagnosed as seborrhoeic dermatiiis. He was
a futt+erm normar
delivery (FrND)and has had no difficurties in the first 2 weeks
of rife.
He is the first child born to healthy parents and there is
no family
history of illness. He was admitted inO a tutt septic ,.r""n *r,
carried out. Two hours after admission he started having tonic-cronic
seizures in his left arm, which lasted for 2 min; this happ"eneJ .Uort
three times. He was given a roading dose of pnenouar'Liton"
ne
contrnued to have focal seizures, which required another half"no loading
dose of phenobarbitone. His seizures became more generalised
tonic-clonic seizures and he was transferred to a spe-cialist cenire for
further management. He was intubated and paralyseO to
contiof
seizures. The test results are:
Na 134 mmol/l
K 4.3 mmol/l
U 4.1 mmol/l
Ca 2.M mmotll
Mg 0.99 mmolil
Glu 5.8 mmol/l
Bilirubin 42 pmolll
Urine pH 6
RAS Negative
ALT 24tufl
Atb 38 g/l
CRP <5
ESR < 2 mm/hour
MSU Negative
Stool No growth
cxR Normal
CranialCT No abnormalities
Hb 14.2 gldl
wcc 17.2x10s11(c.7.4, N 6.1)
PLT 459 x 10s/l
PTT 39s
TT 1s
INR 1.3 ,

csF
WCC 100 (75% lymphocytes)
RBC 15
Protein 0.99 g/l
Glu 2.4 mmolil
No organism
Serum glu q.5 mmoi/l
NH, 28 mmolll
82 100 Grey Cases in Paediatrics for MRCPCH

Lactate 1.8 mmolll


pH
becoming aggressive an
7.38
he is taking lamotrigine i
tuo, 5.2 kpa..
him. ln the last 2 months
Po2 8.3 kpa
seizures in the morning.
Be 4.2 these occasions but he s
Urine pH 6
something requiring con
1. What is the most likely diagnosis? sometimes in the mornir
a Viral meningitis with medication.
b Bacterial meningitis 1, What is the most likr
c Viral encephalitis a Atypical absencr
d Herpes encephalitis b Absence epileps
e Tubercularmeningitis c Juvenile myoclc
f tt4etabolicencephalopathy d Generalised ton
g Non-accidentalinjury e Absence with e'y
h HIV f Benign Rolandic
i Toxic encephalopathy g Juvenile absenc
2. Which first three lines of drugs should be used in this case? 2. What abnormalities
a Cefotaxime or ceftriaxone
b Gentamicin
c Benzylpenicillin
d Erythromycin FP2-F4

e Aciclovir F4-C4
c4-P4 *e
f Metronidazole PLOa aal**
g Rifampicin fP2'F8 .-
h Streptomycin F&TIO *-,-
++
i Ganciclovir TIGPS
Pt-oz #
3. Which other two tests should the laboratory carry out on the FPI-F3 €
sample of CSF? F3-C3 +
a PCR for viruses (HSV, EBV, adenovirus) c3-P3

b PCR for bacteria (Neissena, Pneumococcus, and P3-O:


Fpl-F7
r
-*-
H aem o p h i I us i nfl ue nzael r'-
f7-T9
c Culture T9-P7 *i+.
d Mycoplasma titres P7-Az ,r
e Oligoclonal bands .EcG -1'-
f Lactate 2y. l Sa.rded€rf
g Pyruvate
h Glycine
i Zeil Gram stain
a Generalised spi
b Generalised soi
Case 42 photosenshiviq
A 13-year-old boy presented with a history of absence attacks since
c Polyspikes anC
the age of 9 years; these start as staring with his eyes fluttering in
d Generalised pot
episodes of 5-10 s duration, and then he returns to normal. He may
3. Which other anticor
controlling his seiz.
have upto 100 of these attacks per day. He was an FTI\D and there
are no concerns about his development. He is doing well at school.
a Carbamazepinr
The results of the general and systemic examinations were normal.
b Topiramate
The awake EEG was reported as normal on two occasions. He was
c Levatricetam
treated with sodium valproate but developed a behavioural problem,
d Oxycarbamazei
e Sodium valp'1
Ouestions 41-50 83

becoming aggressive and violent with other children. At the moment


he is taking lamotrlgine and ethosuximide, which are not helping
him. ln the last 2 mon;hs, he has had three generalised tonic-clonic
seizures in the morning. His mother said that he had no .ierks on
these occasions but he said that sometimes when he is doing
something requiring concentration he does have jerks (and also
sometimes in the morning). There is a problem with his complying
with medication.
1. What is the most likely diagnosis?
a Atypical absence seizures
b Absence epilepsy
c Juvenile myoclonic epilepsy
d Generalised tonic-clonic epilepsy on awakening
e Absence with eye rnyoclonia
f Benign Rolandic epilepsy of childhood
g Juvenile absence epilepsy
2. What abnormalities appear on this EEG?

Fp2-F4
F444
c4-P4
P4-Oz
Fp2-F8
FE.TIO
TIGPS
Pr-o:
he Fpl-F3 &,".-,"*^e.-^^.,
\ *&+)-
c3-P3
P3.o:
vv"-..
"-rw-_y-r,4wL lr --'x-*.-
fpl-F7
r7-T9
T9.F7
v,oz
.ECG

2y. 't cpaso& otdaring Ayake: Nilen

a Generalised spike and wave of 3 Hzls


b Generalised spike and wave of 4 Hz/s with
photosensitivity
s srnce
c Polyspikes and waves with 4-5 Hzls
ng in d Generalised polyspike and waves
ie may 3. Which other anticonvulsants, in order, should be tried for
: there
controlling his seizures?
;chool.
a Carbamazepine
rormal.
b Topiramate
e was
c Levatricetam
rroblem,
d Oxycarbamazepine
e Sodium valproate
84 100 Grey Cases in Paediatrics for MRCpCH

f Vigabatrin c Ventilate
g Clonazepam d Give antibiotic
h Clobazam e Give antiviral r
f Urine for toxic
g Transfer to ITL
Case 48 h Ask for help
i Send blood fo
A S-year-old child presented with a history of diarrhoea and and safe
vomiting in the last 3 days. His vomiting is less frequent now but his
diarrhoea is getting worse and his mother described nis stiot-- The girlwas resuscitat
form as'runny and smelly'. He had the same symptoms 3 months transferred to the liver
ago, which resolved within 3 days and no cause was found on stool list for a liver transplar
culture. He is not interested in any food other than sweets -
upper limbs. The test t
biscuits. He has been complaining of a sharp pain in ttre iislt-itiac
"na '
fossa, which lasts for 1-2 hours and then disappears. He w-as- Hb 7.2g,1
swimm_ing in the sea 2 weeks ago with his brot'her and father. Tne WWC 4.3 x
upppr Glr endoscopy and biopsy show partial villous utropr,r. tn" PLT 120 x
coeliac screen and test for lactose milk intolerance were n"g"tir". INR 6
Other test results are: PTT > 100
Hb 7s PT >30:
slat ALT 32011
WCC 15.6 x 10Yl (N 6.3, L 3.0, E
ALP 2250
PLT 425 x 10e/l
CRP 20
Atb 16 s/'
Na 126 r
ESR 2 mm/hour
LFT K 5.3 n
Normal
Ca 1.88
Abdominal US Normal
Mg 0.66
Stool A lot of mucus and blood
MSU 15 white cells only and C/S no growth
2. What are the diff€
1. What is the most likely diagnosis? a Renal failure
2. Which other three tests should be carried out? b Septicaemia
c lntoxication
d Hepatic failur
Case tl4 e Metabolic dis
f Hepatitis
on. her 13th birthday, a girl presented to A&E with a history of a
reduced level of consciousness and lethargy. she was ort uti diy with
g Encephalopa
her mother, and after she returned homg. ihe fert ,n*"rl-*unl to
h Heavy meta!
sleep and her mother found her difficuli to rouse. she has been
3. Which other thra
order?
unwell in the past few rnonths, with abdominal pain that wai a Blood cuhurt
diagnosed-as constipation. she rooks pare and skinny. Her aunt oied
of hepatic failure at the age of 44 years. There is no consang;inity.
b Urine toxicoi
c CRP
Her sister is 7 years old, fit and well apart from one vaginalLleed last
week. The girl responds to pain but does not open her-eyes.-HLr
d Urinary copp
serum glucose level is 2"4 mmolll and her Bp is 6s/s0 mmHq. Her
e Caeruloplasr
f Blood gas
temperature is 36.8"c, her HR is 110/min and capillary refiil [ime (cRT) g Viral serolog
is 4 s centrally. hLP
1. What should be the initial management of this case? i Abdomir:at I
a Resuscitation fluid j Cranial h'i
b Secure airway k Liver bic:s.'
Questions 41-50 85

c Ventilate
d Give antibiotics
e Give intiviral agent
f Urine for tgxicology
g Transfer to ITU
I h Ask for help
; i Send blood for FBC, U&E, LFT, clotting, culture, blood group
-.
and safe
lbut his
I The girt was reeuscitated and the blood results came back. She was
Fnths transferred to the liver unit, ventilated for 7 days and put on a waiting
pt stool list for a liver transplant. She developed abnormal movement in her
id upper limbs. The test results are:
I iliac
i Hb 7.2sldt
pThe wwc 4.3 x 10e/l
pt. The PLT 120 x 1Oe/l
Iive. INR 6
PTT >100s
PT >30s
ALT 320 tu/|
ALP 2250 rUtl
Alb 16 slt
Na 126 mmol/l
K 5.3 mmol/l
Ca 1.88 mmol/l
Mg 0.66 mmol/l

2. What are the differentialdiagnoses and in what order?


a Renalfailure
b Septicaemia
c lntoxication
d . Hepatic failure
e Metabolic disorders
f Hepatitis
g Encephalopathy
h Heavy metal poisoning
3. Which other thre€ tests should you carry out and what
order?
a Blood culture
died b Urine toxicology
ty. c CBP :

last d Urinary copper


e Caeruloplasmin
Her f Blood gas
(CRT) o Viral serology
h LP
i AbdominalUS
j Cranial MRI
k Liver biopsy
86 100 Grey Cases in Paediatrics for MRCPCH

4. What is/are the abnormalityiies on this photo?


c lntoxication
d Myasthenia gravi:
e Myotonic dystrop
f Dermatomyositis
g Myeloencephaliti:
h lnborn error of mr
i Midbrain aslrocyt
j Meningitis
k Guillain-Barrd syr
2. Which three invesliga

Case 46
A 7-year-old boy presente
gain and a large spleen. I
life to correct biliary atres
was discharged from foltt
has been well since, apar
hospital with jaundice at t
various blood tests and a
Case 45 apart from a raised ALT k
A father accompanied his 14-year-old daughter to a children's outpatient
was considered to be not
feeling unwell, with letha
departrnent and she was seen by a GP. She had a UTl, was feeling
pain and headaches. His,
teti-rargic and had double vision. She was referred to hospitalfor a
found that he had a large
seconl opinion. She always feels tired, especially at weekends and at the
referred for an abdomina
end of the day. she was well before and has no previous history of eye
problems. Her family has no history of eye problems. she plays hockey from a large spleen. Whe
lethargic, pale and said h
at scnool and is also a keen cyclist. Her family lives in a four-bedroomed
house, with one cat" Both parents are teachers and she has two brothers,
appetite is reduced and h
measures about 4 cm bel
who are healthy. She looks tired, has both eyes half-closed and she finds
edges and is mildly tendr
it difficutt to open her eyes; she has to tilt her head backward to be able
to look to someone. Her eye movement is intact her pupils react equally abdominal LIS showed a
measuring 3 cm below tl
and she has no problem with her visualfield. Her fundi are very easily
The test results are:
visualised and tirere is no evidence of papiltoedema' Her other cranial
nerves are all intact. She said she feels weak in her body but her reflexes Hb 8.2 gldt
are all present; she can lift her arms and tegs against gravity, with normal wcc 2.'l x 1G
tone. She feels stronger in her lower limbs than in her upper limbs. She PLT 100 x 10
was admitted to hos[ital for further testing, and in the evening she Na 135 mrn
choked on her food and found it difficult to breathe. she was given 10 K 4.2 mm<
umin oxygen via a facernask. which helped a little. By midnight she was Atb 26 gn
intubated ind transferred to the PICU. Her CXR was normal and blood Total protein 5s gn
gas testing showed respiratory acidosis._Her EEG and cranial MRI with ALT 166 tU i
Iontrast siowed no abnormalities. An LP carried out in the PICU was YGT 150 lu t
reported to be normal. Lactate, ammonia, AAs and organic acids were Bilirubin 20 pmol
reported to be normal. She continued on a ventilator for 7 days and INR 1.3
made a good recovery with treatment. PTT 55s
1. What is the most likely diagnosis?
PT 12s
HBsA Negativ
a Viralencephalitis
HBsAg Negativ
b ADEM
Questions 41-50 87 -
c lntoxication
d Myasthenia gravis
e Myotonic dystrophy
f Dermatdmyositis
g Myeloencephalitis
h lnborn error of metabolism
i Midbrain astrocytoma
j Meningitis
k Guillain-Barr6syndrome
2. Which three investigations are required to prove your diagnosis?

Case 46
A 7-year-old boy presented with a history of malaise, poor weight
gain and a large spleen. He had an operation in the first 5 weeks of
life to correct biliary atresia. The operation was successful and he
was discharged from follow-up at the age of 2 years, fit and well. He
has been well since, apart from viral infections and one admission to
hospital with jaundice at the age of 4, which resolved completely;
various blood tests and abdominal US showed no abnormalities
apart from a raised ALT level, which at that time was ignored as it
'e. s outpatient was considered to be not siEnificant. ln the past 6 weeks he has been
as'eeling feeling unwell, with lethargy, non-specific muscle pain, abdominal
::a for a pain and headaches. His doctor examined him 2 weeks ago and
re^is and atthe found that he had a large spleen and distended abdomen. He was
- s:cry of eye referred for an abdominal X-ray, whrch did not show anything apart
E E ays hockey from a large spleen. When he was seen in the clinic, he was
:-r-iedr6.med lethargic, pale and said his energy is gone. He sleeps a lot, his
€:s trvo brothers, appetite is reduced and he has started losing weight. His spleen
e: =rd she finds measures about 4 cm below the left costal margin, with smooth
herc to be able edges and is mildly tender, and there is no organomegaly. An
) s 'eact equally abdominal uS showed a spleen measuring 5-6 cm, a liver
i:e uery easily measuring 3 cm below the costal margin and polycystic kidneys.
' :l. er cranial The test results are:
i-r: her reflexes Hb 8.2 slOt
*':. with normal WCC 2.1x10sll (N 60%)
rce'iimbs. She PLT 100 x 10s/l
rE^ 19 She
.,,es given 10
Na 135 mmol/l
' :- ght she was K 4.2 mmol/l
Alb 26 g/l
-: and blood Total protein 55 g/l
:- : MRlwith ALT 166IU/I
:-: )!CU was TGT 1501U/l
- : a:ids rtrere Bilirubin 20 pmol/l
'- ::ys and rNR 1.3
PTT 5h s
PT '!2 s
HBsA t{egative
HBsAg Negative
88 100 Grey Cases in Paediatrics for MRCpCH

HBC Negative
h Acute porphyrir
What are the four rr
Blood film Target cells
out?
AIK. Ph 360 ru/r
ANA " Negative'
a Urinary copper
b Urinary coprop
1. What is the most likely diagnosis? c Lead level
2. Which other two tests should be carried out? d Midnight cortis
J. What is the prognosis? e Urinary hydrox
f Blood gas
Case 47
g Hepatitis B and
h Renal US
An 11-year-cid boy presented to A&E with a history of diarrhoea and i GFB
vomiting in the last 24 hours. He was at schoolthe day before and j Serum AAs
was sent horne, as he felt unwell and had abdominal pain. He said he k Lactate level
has been feeling unwell for the last 4 weeks, with generalised I Urine toxicolog
weakness and lethargy. His friends commented that his skin is m Short Synacthe
suntanned, even though he has not been on holiday for the last n Abdominal MR
2 years. He was a FTND. He had viral meningitis when he was 7 years What should the mi
old but recovered well. His father was diagnosed as having irrita'ble a l.v. antibiotics
bowel syndrome 3 years ago. His mother has chronic asthma. His b Daily glucocort
younger brother and sister are healthy. He likes dogs and was c Daily mineralo<
promised a dog if he passed his GCSE exams. d Haemodialysis
He looks darker than his brother, his BP is 9565 mmHg, and his HB gO
e Detoxication w
b,p.m.There is no evidence of skin lesions and his abdomen is
f Liver transplan
generally tender but soft, with active bowel sounds. The test results are:
g Renal transplar
h Full endocrine
Na '128 mmol/l i Cranial MRI
K 6.2 mmol/l
Ur 12.2 mmolll
Cr 77 mmol/l Case 48
ALT 38IU/ A mother brought her c
ArK. Ph l2s tufl hours to A&E, with abd
Alb 27 gll stool in the last 2 day,s.
Hb 9.8 g/At illness. She was sent h(
WCC 7.3 x 10s/l (N 10%, L 80%) not dehydrated and d3{
PLT 180 x 10s/l presented with simiiar
MSU Ketones positive abdominal pain, mala.s
I

BS 2.5 mmol/l temperature is going u;


Stool No blood and C/S is negative first time. She also dev,
Abdominal US Normal measure about 2 mrn '
Ca 2.90 mmol/l healthy and returned fr
HCO3 13 mmol/l :
There is a 2 cm splencr
1. What is the most likely diagnosis? 77 b.p.m., her RR20 rr
a Septicaemia Na 130 rn
b Chronic renal failure K 3.9 rr'
c Metabolic disorder Ur 8.3 rnr
d Addisonian crisis Cr 76 mn
e Wilson's disease ALT - 55 lU'l
f Lead poisoning AIK. Ph 290 ru
g Chronic liver disease
Ouestions 41-50 89

h Acute porphyries
2' what are the four most significant tests that shourd be carried
out?
a
lJ,rinarV copper
,
b Urinary coproporphyrin.
c Lead level
d Midnight cortisol level
e Urinary hydroxyprogesterone or steroid level
f Blood gas
g Hepatitis B and C serology
h Renal US
and i GFR
E j Serum AAi
Fh"
k Lactate level
I Urine toxicology
I
r
m Short Synacthen test
t
n Abdominal MRI
Fars
3. What should the management plan be in three steps?
t a l.v. antibiotics
i.: b Daily glucocorticoid
c Daily mineralocorticosteroid replacement therapy
d Haemodialysis
h
e Detoxication with DTA
0 f Liver transplant
I
g Renal transplant
)ie: h Full endocrine work-up
l i CranialMRl
lr
F Case 48
;.
F A mother-b^rought her daughter for the second time in the last
24
hours to A&E, with abdominal pain and frequent passing
E of a soft
lE, stool in the last 2 days. The 4-year-old was tit ano werr u"etore is
it
[* illness. she was sent home after she was assessed at hospitit;
'' stre is
F not dehydrated and does not rook septic. Two days ratei sie -
presented with similar symptoms: headache, tethargy,
h
F"
more
abdominalpain, malaise and a pain in her neck. Heimothersaid
her
lt- is going up every day, and today reacheA +O;C to, ine
lI PTqgratrlg
Trrsl trme. 5he also developed erythematous papular
1',
t lesions, which
measure about 2 mm in diameter. A{l members'of her
h*
healthy and returned from a Mediterranean cruise
family;;;
t, rrorioav o'oavs ago.
There is a cm splenomegaly, with no lymphadenop"tf,-y.
L
I
77 b.p.m., her RR/20 min, ind her CRT <'2 s.
i"ifrn L
"t
r Na 130 mmol/l
K 3.9 mmol/l
*. Ur 8.3 mmot/l
Cr 76 mmol/l
ALT " s5 tu/l
AIK. Ph 290 tu/t
90 100 Grey Cases in Paediatrics for MRCPCH

Bilirubin 35 pmol/l
Hb 11.39/dl
wcc 2.2 x l0sll (L 66%, N 22%l
PLT 9O x 10e/l
MSU Negative
Stool Negative
Abdominal US Large spleen only
1. What is the most likely diagnosis?
2. which two other tests are required to support the diagnosis?
3. What should the treatment be in this case, in three steps?

Case 49
A 1O-year-old boy presented with a history of right limb pain, which is
getting worse. He was playing football at schoolthis morning with
other children. He fell a few times but had no bruises or cuts on his
limbs. He had limb pain on and off in 7 of the last 12 hours but it was
not as extreme as he currentlyfeels. He had had no joint swelling or
rashes associated with this pain before. No one in his family suffers
from a joint or bone problem. He said he could not walk after he got
back from school. He has a low-grade temperature and is not able to
bear weight on his feet. His left lower limb is difficult to move. He can
bend his knees up to 90" on the right but only up to 70o on the left. He 3- What is tha rnost likelt
can do full hip flexion, abduction and internal rotation on the right a P',^;thes: diseaS€
side, but finds it very difficult to do it on the left side. He has a b JCA
generalised tenderness on his upper thigh and keeps his left lower c Septic arthritis
limb in a flexion position at the hip and knee joints. d Osteomyelitis
14.2 gldt
e Slipped femoral e
Hb
14.3 x 1Oe/l (N 75%)
f Neuroblastoma
WCC
350 x 10e/l
g Osteosarcoma
PLT
CRP 20
h TB arthritis
ESR 12 mm/hour
Case 50
1. Which other tests, in order, should be carried out immediately?
a X-ray of hips and knees A toddler, aged 2 years, Yr
b US of hips on her face. Her childmirrt
c Abdominal US morning her father had b+
d Blo6d culture business trip before luncl'
e Urine for VMA woke up at 3.00 p.m. witi"
f Bone scan said that the bruises were
g MRI of both lower limbs sleep, and that since the :
h CT scan of lower limbs her mother, refused to ha
i Hip aspiration She played outside with '
2. What two abnormalities appear on this hip X-ray? about 2 hours. Her mothe
a Dislodged left femoral epiphysis advertising in a family co
b Reduced joint space on left the childminder looked a'
c Beduced bone density on the left They visited a farmhouse
d Fragmentation of femoral head on the left animals and spent all da'
e Swollen soft tissues around the left hip ioints temperature is 37.5"C anr
0uestions 41-50 91

trr
{e 3. What is the rnost likely diagnosis?
a Fe+hes'ciiseasa
b JCA
c Septic arthritis
d Osteomyelitis
e Slipped femoral epiphysis
f Neuroblastoma
g Osteosarcoma
h TB arthritis

Case 50
A toddler, aged 2 years, was referred by her doctor for fresh bruises
on her face. Her childminder looked after her all day. Only that
morning her father had been working from home and left home for a
business trip before lunchtime. The mother said that her daughter
woke up at 3.00 p.m. with the bruises on her face. Her childminder
said that the bruises were not there before the child's afternoon
sleep, and that since the morning, she was clingy, wanted to go with
her mother, refused to have her breakfast, and felt hot at that time.
She played outside with her older brother, who is 3 years old, for
about 2 hours. Her mother is working full time as a director of
advertising in a family company. There were no other concerns and
the childminder looked after the mother's children for the last 3 years.
They visited a farmhouse 2 days ago and the children played with the
animals and spent all day at the farm. The girl looks tired, her
temperature is 37.5'C and her HR is 90 b.p.m.;there were no other
92 100 Grey Cases in Paediatrics for MRCPCH

abnornnalities found on examination apart from a red throat. The 3. a PCR for viruset
bruises are about 5 cm long and about three of them are located on d Mycoplasma i'
the right side of her face, anterior to her right ear. There is another
red mark, which is 1 cm long and is located on her right shin. There Viral encephalitis
are a few petebhial rashes'on her right fronto-parietal region, not
visible on the skin, but can be seen at the margin of the hair and skin. Acute presentation $ :'
The test results are: vomiting should aler: :
encephalitis as the fi's:
HB 13.2 gldt neurology associatec .''
WCC 8.2 x 10sll (L 60%) with herpes simplex e'
PLT 240 x 10sll usually complain of ':
INR 1 one minute they are 3
PTT 35s minute they will have :
PT 10s commonest focal ne;'
LFT & U&E Normal always focal, then be::
MSU Negative as those associatei r.'
CRP <10 of clinical suspicion it
Mycoplasma and ba::i
1. What is the diagnosis?
is important if the cn
a Trauma injuries not necessary. Cran.=
:

b Viral infection some changes. The ::


C Slapped-cheekdisease CSF and blood. lf her:
d Glandular fever minimum of 3 weers
g sStreptococcalsepticaemia is possible after stoE:
f Sunburn known which antivi':
g NAI important.
2. List, in ororer, four steps reGUired for her management?
a Admit the chiid anC inform the duty consul-tant
b Viral S,alciOgy Case 42
c Stirl i.v. antibiotics 1. c Juvenile rni r
C inform Social Services and the Child Protection Team 2. b Generalisec s
e Check child protection register ? e Sodium vaic'
f Discharge home b Topiramate
g Give prophylactic antibiotics to all family members in contact h Clobazam
with the child in the last 24 hours
h Arrange for photos and skeletal survey Juvenile myoclonic q
i lnform the microbiologist Clumsiness occurs -
j Arrange for photography
k Transfer to specialist unit wakes up. lt is usua "
I Tell parents about your concerns 18 years in the ma;:-
bilateral and cons s:s
manytimes. lf ther': =
consciousness vvi!' :=
occur as a result of s
ANSWERS 41-50 from nocturnal or c:
generalised tonic-: :
Case 41 associated with po ' .
1. c Viral encephalitis polyspike discha:ge:
2.d Erythromycin recording. Seizures :
a Cefotaxime or ceftriaxone or eye closure. The '
e Aciclovir phenytoin, and tb-s'
Answers 41-50 93

ai. The 3. a PCR for viruses (HSV, EBV, adenovirus)


ocated on d Mycoplasrna titres
;another
tin. There Viral encephalitiq,
cn, not
rir and skin. Acute presentation with fever, headache, lethargy, nausea, and
vomiting should alert the attending doctor to consider a diagnosis of
encephalitis as the first choice. There is a high incidence of flcal
neurology associated with the presentation of encephalitis, especially
with herpes simplex encephalitis. older children wlirr enceprratitis
usually complain of headache and will have behavioural changes:
one minute they are OK, talking and doing things, and the nexl
minute they will have bizarre behaviour and belonrused. one of the
commonest focal neurology signs is seizures, vuhich are almost
always focal, then become generalised. other neurological signs such
as those associated with meningitis will be very rare. A high Jegree
of clinical suspicion will prompt immediate cover for nerpEs siniplex,
Mycoplasma and bacterial infection. lnvestigation, which includes Lp,
is important if the child is stabte. EEG is het[tut when available, but
not necessary. cranial cr is important, with contrast, as it may show
some changes. The course of treatment depends on the findings from
CSF.and blood. lf herpes simplex encephalitis is present, tfren i
minimum of 3 weeks' i.v. treatment is iecommended and recurrence
is possible" after stopping the treatment. For other viruses it is not yet
known which antiviral is best, and consultation with a virologist is
important.

Case 42

1. cb
2.
Juvenile myoelonic epilepsy
Generalised spike and wave o'f 4 Hzls with photosensitivity
3. e Sodium valproate
's b Topiramate
n contact h Clobazam

Juvenile myoclonic epilepsy


clumsiness occurs in the morning, soon after the affected individual
w-akes up. lt is usuallyfamilial and the age of onset is between 12
and
]8 vears in the majority of patients. The myocronus is usualry brief,
bilateral and consists of flexor jerks of the arms, which may occur
many times. lf there are no generarised tonic-clonic seizures,
consciousness will be retained. Generalised tonic-clonic seizures may
occur as a result of sleep deprivation, alcohol intake and awakening
frorn nocturnal or day sreep. This co;rdition is associated with
generalised tonic-clonic and absence seizures. Seizures are
usually
associated with polyspike and slow waves on the awake EEG. The
polyspike discharges are also concurrent with seizures in ictal
recording. seizures are sometimes precipitated by photic stirnulation
or eye closure..They are exacerbated by carbamaiepine and
phenytoin, and thus these drugs should be avoided at all times. The
94 100 Grey Cases in Paediatrics for MRCPCH

drugsofchoiceshouldbesodiumvalproateandlamotrigine.Life- k Liver biopsy


l,onf, tt.rt*ent is required, should not be stopped and.
very rarely
nlof, ooses are needed. some children are not controlled on 4. Kayser-Fleischer rin
pv, pecial ly patienls with genera ised to n ic-clonic
rn;""-tn"r. es I
Wilson's disease (hepatc
seizures, and may require a second drug'
This disease is inheritec
caused by a disturbanse
Case 43 incorporation into caer,
1. Giardiasis chromosome 13 and sy:-'
Repeat stocl analysis (three warm samples) the brain, liver and cornr
2. is hepatic failure, usualrS
lmmunoglobulins
Neurological ma nifesta:
Giardiasis
minimal liver problems.
There or speech, and may ren:
Giardiasis is caused by parasitic infection lGiardia lamblial. will appear and the initia
which infests the
,'i" t*" ,i"ges: the first'is the atrophozoite, include dysarthria, dy*c
oroximal sriall intestine in children. Significant mucosal darnage tremor, and drooling. T:'
;;;;;;:;;;ity partiat villous atrophv, but sometimes to the flat symptoms prior to the a
Iir.o.l. lf.,e otirer form is an orai cysr, which can survive in soil as behavioural disturba.
andwaterforseveralmonths;itisthemostinfectiveform,even on. The corneal features
frorn person to person. lmmune deficiency, m-alnutritio.n and certain features of the c
nvp"i"ioliv are the predisposing factors to infection with diarrhoeal
the
copper in the Desceme:
G.iim,Atiacyst. There is either icarrier state or an acute
these patients. Wilson's
lf f diarrhoea and intestinal malabsorption. Most
r".i,-or chronic with dystonia and dysar
cnifO#ntaf f into the carrier category. Children with FTT and
chronic
disease. The caerulopra:
or intermittent diarrhoea should be investigated for this..pa.rasitic
mg/|. Cranial MRI will cje
int".iiun. Stool culture and microscopic examination will diagnose if other decreased size of the ca
lnt*itio,., with G. tambtia" Multiple stool samples are requ.ired. midbrain, and pons. Tre
and is not to be discon:
biopsy
Ilgh d"gr"u of suspicion of giardiasis. Occasionally intestinal
"",'''"rchronicdiarrhoeacannotbefound,andthereshouldbea of the family is imporra-
*iv drug of
r"re"r the parasitic infeition' Metronidazole is the
and should be administered for 10 days' For
i*.t,rn".ornpromised patients, this drug may need to be
"floit" Case 45
administered for uP to 6 weeks' 1. d Myasthenia gr;
2. Edrophonium tesr
Anti-acetylcholine -
Case 44 EMG
1.h Ask for helP
b Secure airwaY Myasthenia gravis
t Urine for toxicologY This is a disease of the .
d,e Give antibiotics and antiviral agent by proximal muscle v,::
o Transfer to ITU muscular exercise. li :"a'
i Send blood for FBC,U&E, LFT, clotting, culture, blood group neu romuscular bl ock: 3.
and save serum
cases, it usually staris a
2. e Metabolic disorders adolescent girls. The :-
d HePatic failure the extraocular musc,es
c lntoxication and/or ptosis. This w,: :
g EncePhaloPathY and bulbar muscles, w-
3. e CaeruloPlasmin some cases it may ta<e
d Urinary coPper muscles to other mus: ,
Answers 41-50 95

k Liver biopsy
4. Kayser-Fleischer ring

Wilson's disease (hepatolenficular degerieration)


This disease is inherited as an autosomal recessive disorder and
caused by a disturbance in the biliary excretion of copper and its
incorporation into caeruloplasmin. T'he defective gene'is located on
chromosome 13 and symptoms are caused by deiosition of copper in
the brain, liver and cornea. The commonest presentation in chiidren
is hepatic failure, usualry without neuroloqicar manifestation.
Neurological manifestations occur in the iecond decade, with
minimal.liver problems. T.he initial symptoms are disturbances in gait
or.speech, and.may remain unchanged'for years. Other symptoms
will appear and the initial symptomi will worsen. New symptoms
include dysarthria, dystonia, rigidity, gait and posturar abnormarities,
tremor, and drooling. This disease is ilso associated with psychiatric
symptoms prior to the appearance of neurological symptomi, such
as behavioural disturbance, paranoid psychosis, and dementia later
on. The corneal features in the form oi Kayser-Fieischer rings witt be
certain features of the disease; this is caused by the deposiiion of
copper in the Descemet's membrane and is present in almost all of
these patients. wilson's disease should be considered in children
with dyslgnia and dysarthria, and in those with any chronic liver
diseas,e. The caeruloplasmin rever wilr be reduced io ress than
20
mg/|. cranial MRI will demonstrate increased signar intensitt a;d
decreased size of the caudate, putamen, subcorticar white matter,
midbrain, and pons. The treatment of chsice is orar D-peniciilamine
and is not to be discontinued. Genetic counselling for other members
of the family is important.

Case 45
1. d Myasthenia gravis
2. Edrophonium test
Anti-acetylcholine receptor antibodies
EMG

Myasthenia gravis
This is a disease of the neuromuscurar junction, and is characterised
by proximal muscle weakness and increased faiiguability on
rnuscular exercise. lt can be a congenital, autoimmune disease
of
neuromuscular blockage due to toxins or drugs. ln autoimrnune
ca.sgs, it usually starts after 1 year of age andls more prevalent
in
adolescent girls. The onset is usuaily iisidious, with invorvement
of
the extraocular rnuscles with unilateral or biraterat opntrraimopiegia
and/or ptosis. This wiil.be.foilowed by involvem.nt Jr tne
froiimar
and bulbar muscres, which may read io difficurty in s*attoiiinj. tn
some.cases it may take weeks and months to spread from the-ocular
muscles to other rnuscres. This weakness is vaiiabre. Many prti"ntt
96 100 Grey Cases in Paediatrics for MRCPCH

may only complain of increasing fatigue' They feel norrna.l on .

,*j6rii-,g, but as the day passes, they {eel rnore fatigued and weak' will enuse mur:h i s::
;;;;i;iiyiot towing exercise. Examinaiion will show norrnal ref lexes spironr:iacto{\e ei.i I

*eufness in th-e proximal muscle group. There wiil be bileteral goal for *irrhosis c' : -
"ni
pr*i, *itn a histoiy of doubie Vision. The EMG will show abnormal oce ur in multipolv: . s:
I"["iitir" rtinrutation and will help in diagnosis. Abnarmalities will be cirrhosis of the live'
ii!"iiii"O ty the edrophonium chioride (Tensilon) test" ln 60-80% of
patients, the presence of antibodies aEainst aceiyicholin€ receptors $ase 47
iuiif U. *no*n. The creatinine kinase level is nr:rmal and muscle
oioprv wiil shov,,no specific features. The diagnosis can be. confirrned 1. Aridisonian cr s s

;;ih; Tensiton test" The test sho{-}ld bethe car:'ied oui in hospital with 2. A.bdominai i'rtr
ieruriitrtion equiprnent ready, in case ehild beeomes hypoxic eir Lsw-dose dexa --
develops respiratory arrest. Tire chi16 shoulcl have a testing dose and Midnight oori sl
t6e iuf f'dose should be given after 30 s. Vider: camera footage or .$hort Synact":-
pi''iotou shoulC be taken in order to obsenve the changes' 'An 3. Daily glucocoi': ::
anticholinesterase agent will be very helpf ul. This is life-long Saily minera c:: '
iieatment and in some patients other trealments such as Cranial MRi
corticosteroirJs or azathioprine can N:e useci. Thymectcimy ean be
.o"tiO"teO for patients when medication is not helpfLll' Aaute primary adrena
Fiur*apt',eresis can be performed in.ac.ute situations' when drugs Aeute adrenal ins;':
nave not had much effect and the child's generai coniition is chronie adrenal i,rs --
deteriorating, but its use is very limited' additional stress s.:-
suffer an acute ac -3 - !
and abdominal pe - '
Case 46 (mainly proxirriat, ::
1. Portal hypertension secon'Jary to hepatic cirrhosis hyperkalaen"ria, r. e: : :
2. CholangiograPhY and tachycardia, ',., -
ERCP death if not recog^ s

3. Poor without liver transplantation as well as an amc 3


term treatment v,':: -
Clrrhosis of the liver crisis may follovy '':
who has lost a io: :'
ln a cirrhotic iiver there is replacernent of nermal liver tissue with nrass and microsc::
nodules of liver cells that are growing in an unorganised fas.hion
with
Portosystemie shunts are present due to rnass and abdorn, ^ :
too much fibrous tissue. nneningococca{ s:::
abnormal anastomosis, resulting from abnormal derangement of liver syndrome). There .',
ano tissues. As a consequence there will be abnormal liver
"*lts also be associate: ,.,
function and portal hypertension will arise. There are many causes, pneurnococcus c' :
*rli.t,can be classified as genetic, biliary, postnecrotic and venous
congestion. Liver biopsy is very important in finding the cause. Primary adrenoc:: :
fVt,i"gr.n"nt should be directed to the cause. Before a biopsy is aclrenal cortex, iel - :
cairiui out, any clotting problern should be corrected, and a biopsy sex steroids. lt is .-s -
irom the affected tissue ior biochemical, ir-nmunological and DNA associated rryith ":*:
studies is important, children with cirrhosis of the liver will achieve anorexia, malaise :
,Lrn" gro*th in the first few years with structured management. The myalgia and artl^.: :
Oiet sflouiO contain sufficient protein, essential fatty acids, minerals, manifestations. i::
trace elements and vitamins, and the caloric intake should be at least hypotension, taci . ::
40% above the normal requirement. lf hepatic failure arises, then the mental status. B::-
of protein should be reduced or Etopped. Fortal hypertension and include hypc - :' "
"rnor"t
will cause splenomegaly, ascites and a growth problem. A large spleen hypoglycaemia,:.:=
ae idosis" Other f : :: - '
Answers 4X-80 g?

will enuse rnueh discornfort. Aseites, if rnild, ean he treated


with
spirorrolactone and, if severe, fnLlserni$e ean be added. Trle
goal for cirrhosis of the livqr is liver tr*,3upirntutiorr. {irtimate
Falpabie kidneys
occur in multiporycystic kirlney disease, which has ,.,o ,dlitl"ri],
cirrhosis of the liver.

Gase tl7
1. Addisonian crisis
2. AbdominatMRt
Low-dose dexamethasone_su ppression test
Midnight eortisol levetr
$hort Synacthen test
3. Daily glucocorticoid-replacement therapy
Da i ly minera locortico id_repl a ce ment
tfr'eia py
Cranial MRI

Acute primary adrena! insufficiency {Addisonian


erisis)
Acute adrenal insuffic-iency occurs in chirdren with
undiagnosed
chronic adrenal insufficiency. These children, when expoled
to
additional stress such as wiifrmajor illness. traurna or surgery,
may
suffer an acute adrenar crisis. They may present with
signs of shock
and.abdominal pain, fever, hypoglyca*rnia, seizures, wiakness
proximaI), apathy, uomttlnf , nrr"ur, hyponatraemii,
S"?i.ily
nyperkalaemia, metabolic aeidosis, hypochloraemia,
hypotension,
a.nd tachycardia, which nray lead to ciiOioputrnonary
.of rp.u unO
f
death.if not recognised and treated. Fruid repracement is iriportant.
as well as an ample dose of glucocorricoid with
the air-n Uei,ijiong-
term treatme.nt with grucocorticoids and minerarocorticoids.
cnsls may lollow from massive adrenal haemorrhage in
ioienat
a newborn
who has lost a lot of brood during binh. There wiil
5e a parpabre frank
mass and microscopic haematuri-a. US is very good
at ioeniity;ng ilre
mass and abdorninal cr is arso herpfur. Adrenar crises
can occur with
meni n gococca I septicaem ia as wel i (Wateiho
use*F ride richsen
syndrome). There will be a massive adrenar haemorrhagu,
alss be associated with septicaemia secondary to strepti"n".u.,
ino it rnay
pneumococcus or diphtheria.
Primary adrenocortical insufficiency is due to destruction
of the
adrenal cortex, reducing production or giuco-minerarocorticoid
and
sex steroids. lt is usually an autoimmrni diserse
and can be
associated with HLA-DB3 and HLA-Bg. Syrnptoms include
weakness,
anorexia, malaise, postural hypotension, abdominal
.y*pto*r.
arth ra g i a, hypeipi gmentarion, and ne u ropsych iatiic
TI:19]:^: l.d
manrtestations.
I

Addisonian crisis is characterised by the iapid onset of


hypotension, ta_chyca rdia, fever, hypoglycaem ia and
deteriorati ng
mental status. Biochemicarcnanges oicur in tn" *"i"riiv
oi'Jrti'untu
and include hyponatraemia, hyp6rkalaemia, raised ,r"" iur"i,-- -
hypoglycaemia,- hypercarcaemia, and rnetabor ic an J respi r"iorv
acidosis' other features include normocytic anaemia and reucopenia.

ilr*

ffi
[ffii
[['lii
ll'
I

I
t 98 100 Grey Cases in Paediatrics for MRCPCH

Case 49
Random measurement of cortisol is often unhelpful, but the short
Synacthen test can be helpful in diagnosing the disease. lt is also 1. b US of hips
associated with positive autoantibodies in 60-7Ao/o of affected a X-ray of hips ;
individuals. Replacement,therapy is with glucocorticoids and e Urine for VMA
mineralocorticoids, but looking for the cause is vital and the 2. c Reduced bone
progncsis is dependent on the cause of the Addison's disease. d Fragmentatior
Children in whom Adciisonian crisis is suspected on first presentation 3. a Perthes'disea
should be investigated fully for an underlying endocrine problem.
Acute painful hips
Case 48 Trauma is one of the c
confirm this condition
1. Typhoid f:ver
suspected if no cause
2. Repeat stool culture
Salmonetla typhiserology (O & H antibodies) ln the majority of case
3. l.v. fluids UTI that occurred in tf'
Anti biotics (ci proflucloxacilli n) refuse to walk and olo
Hygiene The examination wili s
extend. The affecteci c
Typhoid fever and any attempt to ex
Blood tests will be nei
S. typhi only infects humans, and causes typhoid fever when an
fluids; aspiration of tn
individual eats contaminated food. S. typhi can survive for a long time,
are negative. Rest ar:
even in frozen or dried food. After an incubation period of usually needed.
10-14 days, vague influenza-like illness with fever, malaise, pains and
headache develops. The fever persists for a week and ihe child will Osteornyelitis and sep
become illwith vomiting, abdominal pain, diarrhoea and cough. There paediatrician and an c
will sometimes be constipation in older children. The affected child move and the chllci r;
looks septic and confused, and has a high temperature. There will be markers will be high a
tachycardia and tachypnoea, but later on, as the disease progresses, provides guidance tc
bradycardia may evolve and chest signs will appear. There is diagnostic, and wher
generalised tenderness in the abdomen, which is also distended. helpful. Treatment is
There may be meningeal signs and hepatosplenomegaly in many untilthe inflammatsr
cases. Sometimes perforation of the gut may occur, especially in the asymptomatic; ther: o
second or third week of illness. This is usually associated with sudden antibiotics will last 6-
deterioration, hypotension, tachycardia, abdominal pain and rigidity. and improvement in t
The blood culture is positive in more than two-thirds of patients in the be treatedwith broad
first week, but only in hal{ of these patients will the stool sample be Staphylococcus infect
positive. The Widaltest may be helpful; an O antibody of more than
Other causes of painf
{our-fold will indicate infection with S. typhibut a high H antibody will Perthes' disease, sickl
indicate previous infection or vaccination. Anaemia, hyponatraemia neuroblastoma, leuka
and thrombocytopenia occur with this illness. Supportive treatment histocytosis, rheumat
with careful electrolytes and fluid balance is more important than
trying to treat the infection with antibiotics. Chloramphenicol,
co-trimoxazole and amoxiciliin are very effective in treating S. typhi.
Ceftriaxone and ciproflucloxacillin are used more frequently these Case 50
days as species of S. typhi become more resistant to other antibiotics. 1. g NAI
Discussion with a microbiologist regarding antibiotic use is very 2. a Admit the ctrlT
important after getting a positive culture, and sensitivity is very e Check child I
important. The duration of illness varies from 7 to 14 days with the d lnform Socia
introduction of antibiotics, as advised by the microbiologist.
100 100 Grey Cases in Paediatrics for MRCPCH

Non-accidental iniuries OUESTIONS 51{O


Children are becoming more and more exposed to abuse. lt is
important to look [ard for indications of NAI in order to save more Case 51
children. This diagnosis is one of the most difficult to face for doctors A lO-year-old boy pres
and staff as well as carers. There is a lot of emphasis on training which started at the ag
doctors to diagnose, recognise and deal with such cases. Children mother are as follows:
with physical injuries may not be walking yet and may have learning side, unable to speak, t
difficulties, an unexplainable or changing history, or changing sites of shaking. This reaction
injuries. Children with a poor social background with domestic he falls from his bed ar
violence shoulo be treated with a high degree of suspicion, and a talking'gibberish'. The
thorough history and checking is required. lnvolving a senior week but he never mis
colleague from the beginning is important. Documentation should be experienced the same
accurate, very descriptive, detailecj and have no gaps' Health age of 13 years. There
professionals should describe what is seen when examining the child disorders. His general
and try to avoid giving an opinion. They should always get someone from a hyperpigmente
to chaperone them on examination and write their names and who shoulder plate. His aw
else is present on the examination form. The notes should not be left results are:
with anyone until the statement or report is finished. The situation
should always be discussed with the consultant as well as the lead BS 4.5 m
child protection person in the treating hospital. Attending case Ca 2.36 t
conferences is very good experience; they can be hostile situations Mg 0.9'1 t

but the aim is to pioiect the child as well as the parents' Child LFT Norn
protection teams will ask health professionals for their opinions and if U&Es Norr
this cannot be given, the child should be referred to the child ECG Norn
protection team leader in the treating hospital. 1. What is the diagn
2. What is the single
3. What treatment s

Case 52
A boy aged 7 years n,a
diarrhoea, which las'te'
underwear and has be
bowelfive times a dai,
Fresh blood was pres€
his passing a large ar-
weight is 25 kg. There
is no family history ct
Africa and the family :
bananas and sweet c:
fullness. His bladder i:
clinic appointment- Tr
He is the only child ir'
abroad.
He does not want to I
not accept him until :
The following tests n
ESR, CRP. Other tesr
ouEsTtoNS 51-.60

Case 51
A.10-year-old. boy preseited with a history of nightmare episodes,
which started at the age of g years. The eiisodei describeb uy rris
mother are as follows: he wakes up with iris mouth deviated t'o on"
side,.unable to speak, with saliva pouring from his rnouth and his arrn
shaking. This reaction lasts 2-3 min and ihen he is alright. sometimes
he falls from his bed and when his mother arrives he ii confused and
talking 'gibberish'. These episodes become more frequent during the
week but he never misses school. He was a FTND. Hii father
experienced the same sort of episodes but grew out of them by the
a.ge o! 13 years. There is no family history of epilepsy or sleep
disorde.rs. His general and systemic examinatibn wai nor-af ,part
from.a. hyperpigmented patch measuring 3 x 4 cm on his left
shoulder plate. His awake EEG was repo-rted as normal. other test
results are:
BS 4.5 mmol/l
Ca 2.36 mmol/l
Mg 0.91 mmol/l
LFT Normal
U&Es Normal
ECG Normal
1. What is the diagnosis?
2. What is the single most useful test?
3. What treatment should be given?

Case 52
A. boy aged 7 years was referred by his doctor with a history of
diarrhoea, which lasted fqr t!19 preceding 4 months. xe atwivi has dirty
underwear and has been builied at scho6rfor his smeil. He oblns nis
bowel.five times a day, with a runny stoor and has to rush to the toiret.
rresn Dlooct was present on three occasions, which was associated with
his.passing a large amount of faeces. He has not vomited and his
yeiglt is25tg' There has been no previous medicar probrem ,r,d th"r"
is.no family history of a similar illness. Both parents are from west
Africa and the family eats an African diet when possibre. He rikes
bananas and sweet corn. His abdomen is soft, with a generatised
fullness. His bladder is full, even if he passes urine 2 h-ours before the
clinic appointment. There are no peri-anal lesions and no fissures.
He is the only child in the famiry and there is no history of traver
abroad.
He does notwant to go back to school and feels that his teacher
will
not accept him untilthis problem is sorted out.

]!gto-[o_wt!g tests were a]l normal: FBC, U&8, TFT, coeliac screen,
ESR, CRP. Other test results are:
102 100 Grey Cases in Paediatrics for MRCPCH

MSU White cells 15


RBC 2 stroke-like event. He i
Protein Positive years old and healthy
No organism Rugby player.
Stool l$o evidence of virus, bacterial or protozoan infection
The optic disc of the I
1. What is'the most likely diagnosis? and the right is norrni
a Crohn's disease und he can move his
b Coeliac disease lateral rectus muscle
c C:,stic fibrosis reveals central scaton
d Anal fistulae delayed.
e Constipation with overflow
f Ulcerative colitis The following tests w
g Hirschsprung'sdisease titres for adenovirus,
h Toddier diarrhoea disease, and all result
reported as normal, fr
i Chronic protozoan infection pressure of 11 mmHg
2. Which other investigation/s may help the diagnosis?
a Upper GIT endoscoPY 0.66 g/1.
b Lower GIT endoscoPY 1. What are the two
c Abdominal X-ray and in what orde
d Sweat test a CSF for lgG c
e Barium swallow and follow through b Cranial MRI
f Barium enema c Cranial and s
g Abdominal US d Lumbar punc
h Repeat fresh, warm stool culture e Bone marro\f
i Repeat abdominal examination f Evoked visua
How can this patient be managed in three steps?
3.
a Start antibiotics 2.
s ERG
What are the mq
b Staft oral prednisolone make these?
c Give phosphate enema a Preseptal orb
d Start laxative (lactulose and senna) b Retrobulbar r
e Give advice about diet cMS
f Refer to surgeon for manual evacuation d Septalorbital
g Refer to surgeon for surgical treatmeni e Ethmoiditis
h Refer to psychologist f Retinoblastor
i Reassure and give no treatment g Neuroblastor
j Refer for genetic counselling h Optic glioma
k Repeat tests after 6 months i Optic neuritis
I Picolax for 2 days j Encephaloml
k Miller-Fisher
3. Which two are ap
Case 53 a l.v. methylpft
A schoolboy aged 5 years presented with painful and reduced eye b Optic nerve d
movement, blurred vision in his left eye and headache. His vision has c Oralprednisr
been deteriorating over the last 12 hours, he can only see bright d Aciclovir for i
colours and any attempt to move his left eye is accompanied by e i.v. antibioti:
severe pain. He is not able to move his left eye at all. His right eye is f Sinus draina(
less painful and he can still see with it, but not as well as before. He is g Chemotherag
a keen cricket player and likes this sport. There are no other illnesses h Badiotherapy
in the family and his maternal grandmother died in her 60s after a i Physiotherap
j l.v.immunoE
Ouestions 51-60 103

stroke-like event. He is the second child in the family; his sister


is 10
years old and healthy, as is his mother, and his father
is a nationat
fiugDy ptayer.
n infection
The optic disc of the reft eye is swoilen and pare, with retinat
exudate,
and the right is normar. There is painfur eye movement
on the reft
and he can move his right eye uuilt is painfurwrren r,e ii using
the
lateral rectus muscle on the right. The visuar fierd examinati;;
reveals central scatomas and the visualry evoked r".ponru-it
delayed.
The fo-llowing tests were carried out: FBC, ESR, ANA, AAs,
viral
titres for adenovirus, coxsackievirus, Mycopla"-r, unOf_V-" -'
disease, and all resurts were normar. A iraniar cr with
reported as normal, foilowed by lumbar puncture with "ontrurt
*r,
an op"ning
pr^e-ssu.ie of 11 mmHg. There were no cells and the protein fErei is
0.66 s/1.
1. What are the two most usefur tests that may herp the diagnosis,
and in what order would you carry these
a CSF for lgG oligoclonal antibodies out?
b CranialMRt
c Cranialand spinal MRI
d Lumbar puncture for viral pCRs
e Bone marrow biopsy
f Evoked visual poteniial
s ERG
2. What are the most likely diagnoses, and in what order would you
make these?
a Preseptal orbitalcellulitis
b Retrobulbar neuritis
cMS
d Septal orbital cellulitis with abscess
e Ethmoiditis
f Hetinoblastoma
g Neuroblastoma
h Optic
i Optic aliomas
neuritis
j Encephalomyelitis
k Miller-Fishersyndrome
3. Which two are appropriate methods of treatment?
g Lr. methylprednisolone for 3 days
rd eye b Optic nerve decompressircn
vision has c Oral prednisolone for 6 weeks
right d Aciclovir for 3 weeks
ldbv e i.v. antibiotics for 1 week
tht eye is f Sinus drainage
afore. He is g Chemotherapy
r illnesses h Radiotherapy
; after a i physiotherapy
j l.v. immunoglobulin
104 100 Grey Cases in Paediatrics for MRCPCH

a Urinary org a -
Case 54 b Calcium/cree.
A boy aged 10 years presented to his doctor with loin pain' He had c Water dep:i, a

naa tfrijbefore 6nd it was resolved with analgesia. His GP asked him d Urine toxicc :
to drink a lot of fluids and he has been drinking 2'5 l/d' He has to e Renal US
wat<e up three times every night to go to the toilet. His father suffers f IVP
from high blood pressure, caused by nephropathy. He is very skinny g Kidney biccs.
and in iain. He was given codeine phosphate, which helped a little h 24-hour ur -r
bit. Aftbr a short period he started shouting from the toilet that he i ,Abdomina - :
was bleeding frorn his penis. This stopped straightaway and he was
rushed for an abdominal X-ray and admitted for further management.
His test results were: Case 55

Na 155 mmol/l A 14-year-old girl u,:s


K 4.2 mmol/l of weight loss, asso: :
Cl 101 mmol/l diarrhoea. She repone
Ur 7.1 mmol/l had a 1-month histo-,
preceding the referra
Serum osmolalitY 302 mosmol/kg .

Urine osmolalitY 110 mosmol/kg has had no previous -


Ca 2.50 history of ulcerative ::
Poo t.b with a weight of 4;-.5 <
PTH Normal level is no evidence of fi:-.gr
Phosphaturia Negative tenderness. There r., e'
Arnino acid urea Negative and general exarnire:
BS 4.5 Na
'1. What is the diagnosis? K
2. Which abnormJiity is visible on the abdominal X-ray? U
LT
Bilirubin
ALT
Atk" Ph
Atb
Total protein
Ca
Phos
M9
CBP
ESR
Hb
wcc
PLT
Ferritin
lron
TGT
TIBC
INR
Anti-endomyseal bc:
Anti-gliadin bodies
Anti-reticulin type '
ln which order should three otirer investigations be Mantoux test
requested?
Ouestions 51-60 105

o Urinary organic and AAs


b Calcium/creati nine urinary clearance
e had c Water deprivation test wiih ADDVP
<ed him d Urine toxidology
;to e Renal US
suffers f IVP
ski n ny g Kidney biopsy
l little h 24-hour urine collection for oxalate
,t he i Abdominal ultrasound
he was
rgement.
Case 55
A.14-year-old girl was referred by her doctor with a 2-month history
of weight loss, associated with vomiting, abdominal pain and -
diarrhoea. she reported no blood or m,iius in her stools. she atso
had a 1-month history of mouth urcers, and rethargy tor r monirr
preceding the referral. There is no history of recenliraret auioio.
sne
has had no previous hospitarisation. Theie is no positive f;;irt
history of ulcerative colitis in the family. She looked pale anJ sii-,
with a weight of 47.5 kg, a height of 16'3 cm.and a eMt of t2.6. ihere
is no evidence of finger clubbing. There is slight epigasiric'
'-' '
tenderness' There were no othei abnormariti6s toun-o ufon systernic
and general examination. The test results are:
Na 138 mmol/l
K 4.0 mmol/l
U 2.3 mmol/l
Cr 74 mmol/l
Bilirubin 6 pmol/l
ALT 8 ru/l
Atk. Ph 80 tu/l
Atb 31 g/
Total protein 79 gll
Ca 2.38 mmol/l
Phos 1.3 mmol/l
Mg 0.79 mmol/]
CRP 50
ESR 80 mm/hour
Hb 12.6 g/dl
wcc 6.3 x 10s/l
PLT 451 x 10s/l
Ferritin 5
lron 2
YGT 118
TIBC 39
INR 1.2
Anti-endomyseal bodies Negative
Anti-gliadin bodies Nelative
Arti-reticulin type 1 We-akly positive
Mantoux test ruegative
106 100 Grey Cases in Paediatrics for MRCPCH
Case Sfi
lgs Normal A 3-year-old girl pres
Stool Negative for bacteria, parasites and viruses off for the last 3 rnc-.
ANA Negative was admitted previ; _

following an episoc:
1. What is the ne>t test thai should be carried out?
admission she was -
a Abdominal US
overnight, ihe situa:
b AbdominalCT
The rnultisticks texl s
:

c AbdominalMRl
the test for C/S was -
d Upper GIT endoscoPY with biopsY
centile, and she has :
e Lower GIT endoscoPY with bioPsY
active as she useo tc
f Barium swallow and follow through
the time in winter, r:
g RePeat stool cultures
Bangladesh and are :
2. tist two abnormalities that appear on her barium swailow and
evidence of cataract :
follow-through X-raY?
normal, and anothe: :
abnormalities apa r-i .-
Na 136 .
K 3.5 -
Ur 5.3 ^
Cr 70--
ALT 12-
Atk. Ph '15C
Bilirubin
Ca
Mg 0.7:
Phosphate l.J -
CRP <5
ESR <2'
TSH 30.6
T4 10r
Hb
wcc 6.C ,
PLT 250 .
FSH 30r
Urine
WCC 10c
RBC 30
Cast Fec :
3. in what order should management steps be carried out Nitrite Neg:
imrnediate!y? Protein a--
a Oral prednisolone of 2 rng/kg for 2-3 weeks AA Pcs :
b Polynneric diet pH 5.3
c AzathioPrine
1. Which three nc.- -
d Mesalazine
a Slit lamp ei,:
e Metronidazole for 'l week
b
f Gluten-free diet
c
Renat US
g LaparoscoPY
d
i.v. ureterog-a
I Surgical removal of affected intestine Measuring -:
i ftefer to psYchologist e Cranial l,4F
j !nvestigate othei nne*rbei"s of family
Ouestions 51-60 107

Case 56

.ftix:_ti:,?,;,fl ff ,"*,"s#,lll,f i,"i,"i;H?[1,i".,.:"fJ,:.f"3..1#


was admitted previorLcty on
two ;;L;;;. with dehydration
f o owi n g a n e p isode'ot'ro,
itin g-r-iti";t i u rrr, o"u. Du ri n g p
I I

admission she was mitdty;.i;;il.;;;i*]in i


revi o u s
overnight, ,.r. luids siven
ihe situation corrected itserf.
she was arso treated for UTr.
The muttisticks text
the test for crs was "no*s
u-wil'.t rioi."o RBC was positive, but
negativg weight are on the 10th
ry"r1"rinilno
centiie, and she has coirse taciat
feitires-and thin hair. she is not as
active as she used to ue ano
sh" pr-#r"i, sit next to a radiator a, of
the time in winter. Her order iiuLl
Bangtadesh and are oistani
ir'*"ttlo*r parents are from
cousl;;: i;;;;", took red, with no
evidence of cataract or cornear
opacity. The fundi examination is
normal, and another systemic
rrd s;;;;;l examination reveals
abnormalities apart from rough
stii on i-,e, hands and legs. no
Na 136 mmot/l
K 3.5 mmol/l
Ur 5.0 mmol/l
Cr 70 mmol/l
ALT 12 tu/l
Atk. Ph 150 tu/t
Bilirubin 10 pmot/t
Ca 2.30 mmot/l
Mg 0.78 mmot/l
Phosphate 1.3 mmot/l
CRP <5
ESR < 2 mm/hour
TSH 30.6 mU/l
T4 10 mU/l
Hb 12.5 gldt
wcc 6.0 x 10s/l
PLT 250 x 1Os/l
FSH 30 mU/l
Urine
wcc 100
RBC 30
Cast Bed cast
Nitrite Negative
Protein +++
AA Positive
pH 5.3
1' Which three non-haematorogicar
tests wi,
a Slit lamp eye examinatijn herp the diagnosis?
b Renal US
c. i.y ureterography
d Measuring'intraocularpressure
e Cranial MRt
108 100 Grey Cases in Paediatrics for MRCPCH

f DMSA scan
g Urinary GAG ECHO i
h Urinary organic acids and AAs DMSA It
i Urine for reducing substances Renal Doppler tt
2. What is the-most likely diagnosis? Midnight cortisol
a Hypotliyroidism Ca 2
b Lowe syndrome 1. What are the thn
c Renal vein thrombosis order would you
d Lead poisoning a Neurofibron
e Nephrotic syndrome b Neuroblasto
f Galactosaemia c Conn's dise:
g Cystinosis d Chronic reru
h Tyrosinaemia e Phaeochrorr
i Wilson's disease f Pituitary ade
3. Which othert\^rotests may help the diagnosis? g Congenitala
a Radionuclide scan for thyroid gtand h Raised intra
b Ur.inary organiqacid i Essential hyl
c Serum AAs j Renal artery
d White cellenzyme 2. What are.the mo
e Serum lead level
t Bone marrow aspirate
o Serum copper and caeruloplasmin Case 58
h Fibroblast cell culture
A 7-year-old girlwas
seizures. She was an
Case 57 of 10 months she wa
delay with hypotonia
A child presented with a history of pallor, headache, blurred vision squint, and ocular mr
and sweating. His parents mentioned thathiS heart races from time to family history of ptos
time, specifically when he sweats. He feels tired and has a problem age of 6 years she de
getting to sleep. His mother said that he was dribbling for 2 days, sodium valproate wa
i montf, ago. His is now 7 years old and said that he has a headache spikes and waves to r
that is generalised and lasts all day, but never'wakes him at night or age of 7 years she st;
is worsi during the morning. At school, there are some occasions caused a dystonic pa
when he needs to leave the classroom for a rest. His blood pressure her eyes rolled up an
is 130/80 mmHg, HR is 90 b'p.m., RB 20/min and temperature is
37.2C. He has two hyperpigmented patches measuring 4 mm each Two months later her
on his torso but has no other skin lesions. There is no organomegaly not able to feed herst
and no heart murmur. All pulses are intact and his visual field and with swinging of trer
acuity are compatible with his age. The fundi are difficult to visualise She is developmental
as he complains that the light is hurting his eyes. His father works as age of 5 years. Her m
a postman and his mother is a housewife. He is the youngest of three her hearing and visiq
children; the older two are healthy. His grandfather died from
ischaemic heart disease' The test results are: She looks well, with e
right eye (this was co
Na 134 mmot/l tremor, with bradytin
K 4.2 mmot/l with generalised hypr
U 2.3 mmol/l centile and her OFC it
Cr 45 mmol/l mass on the left side.
MSU Negative
KUB US Normal Urea 9.1 n
Cr 87m
Questions 51-60 i09

ECHO Normal
DMSA Normat
Renal Doppler Normal
Midnight corriFol < 200 mU/l
Ca 2.48 mmot/l
1' what are the three most rikery differentiar diagnoses, and in what
order would you make them?
a Neurofibromatosis type 1
b Neuroblastoma
c Conn's disease
d Chronic renal failure
e Phaeochromocytoma
f Pituitary adenoma
g Congenital adrenal hyperplasia
h Raised intracranial piessure
i Essential hypertension
j Renal artery stenosis
2. What are the most useful investigations?

Case 58
A 7-year-old girlwas referred to a specialist centre
with ataxia and
seizures. She was an FTND ano rraJno-nuon"t"t proutems.
of 10 months she was seen by rrer cF, wno oiagnos"J Ai in"
"g"
o"i"iop*"ntut
delay with hypotonia. she was ,eieir#to
ed vision an eye doctor for her
frcm time to squint, and ocurar motor apraxia was diagnosed.
There is strong
r oroblem
family history of ptosis (mother unJ rnltu.rnat grandmother).
age of 6 years she developed absence i"lrrr"., At the
2 days. and treatment with
sodium vatproate was stahed. an e iG pe.tor*eo
:headache
spikes and waves to one side, and was iescribeo
;;il';i;;e snows
a: night or as auno-r-m"at. ay tr,"
:casions age of 7 years she started to have a aifreient
type of seizure, which
c pressure caused a dystonic Dosture of her arms that tasiiO
tor, up to ZO ,in,
her eyes rolled up and she *""not ,Of
i-rre is io communicate.
:nm each "
Two months rater her abirity to wark started to deteriorate, she was
;aromegaly not able to feed herserf, and myocronic seizures
emerged, associated
'e d and with swinging of her head.
:: visualise She is- developmentally delayed, and first walked
:' ivorks as and spoke at the
age of 5 years. Her memory is describeJty hern,otr,"i-J';sloo,,
;est of three her hearing and vision are normal. ano
'-:11
she looks well, with a convergent squint on the reft
right eye {this was corrected it trr" Jg" of s years). eye and a normar
tre.mor, with bradykinesia and a widi base gait.
she has marked
Her tonu ir reJuced,
with.generalised hyperrefrexia. Her *eighi
and height are on the 3rd
centile and her oFC is on the 1Oth centiie. There
ir in JJo-i"rr
mass on the teft side, not crossing the midrine.
rne test res;ii;;r",
Urea 9.1 mmot/l
Cr 87 mmol/l
110 100 Grey Cases in Paediatrics for MRCPCH

'141 mmol/l and was described as


Na
4.1 mmol/l illness.
K
Ca 2.61 mmol/l He had other probtem
Poo 1.01"mmol/l months and became v
wcc 6.1 x 10e/l oesophageal reflux wi
Hb 13.2 gldl year. He smiled at 6 w
PLT 231 x 10s/l himself with his finger
ESR 40 mm/hour was able to perform tl
CRP 6 can perform five-word
Abdominal US bilaterat multiple cysts affecting kidneys his parents as 'able to
Cranial MRI lett f."*i"pheric dysplasia, left cerebellar dysplasia' tested and is normal.
Ltt-int"riui vermis'absent with evidence of gliosis'
There has been no suc
Normal right hemisPhere
Normal want other children, as
ERG, EVP geneticist. The parents
1. Name three differential diagnoses convergent squint, the
a Aicardi sYndrome present. There is no d1
b Joubert sYndrome areflexia and weaknes
c ldioPathic cerebral dYsPlasia and height are on the I
d FamilialcerebraldYsPlasia results are:
e lntoxication Serum AA, urine orgar
f SPinal dYsraPhism MRI
g MYotonic dYstroPhY
h Ataxiatelangiectasia EEG
2. Wni"f, three invistigations should be performed?
a AlPha-fetoProtein ERG
b ChromosomalstudY NCS, EMG
c Vitamin B.,r level
d Biotin level 1. What are the th ree
e EMG 2. Which three other
f lmmunoglobulin level 3. What advice shouk
g CXR
I stit tamP eYe examination
i DNA linkage studY Case 60
A 10-year-old girl prese
smalland short wasted
Case 59 upper limb, with left su
hypotonia is the
The presence of tiemor, developmental delaY and mainstream school. Th<
i".tbn why a s-year-otO boy wis reJerred to the outpatient
. during the night if she I
to, a secJni oplnion He.was'an FTND' and had mild day at the age of 2year
department
ir,IJi* ., birth. Aithe igl of 5 weeks he was admitted with aBv the weakness in her leg wa
i.,-ili"rv Liiriitabitity and crying; no reason for this was found. has been investigated_ I
he was described as
;;;;f t*;exs, ouiinf ,o'iini""t"ssrnent' possible.investigation'
further investigation ha
'floppy' and referred fir further opinion and weakness in the right ut
and bilateral optic
He was found to b;-de;;;tuntlltv d.elaved' upper limbs, clumsines
neurophvsiological
atrophy was diagno;;i' A ilii;etabolic and weakness with markeJ
was detected' An
investigation was *ri.re*i o't; no abnorrnality She has a full range ;' ;
optic atrophy, and pigmentery
;;;th;i;"logist co;tirmed the
of 7 months he had examination are nor-ra
retinopathy *"t o"i""i"Jit *titJt"ge' nitrrb
age
and dystonia, she can walk dow-s:= -
a febrile illness associated with extrapyramidalligns
Ouestions 51-60 111

and was described as having a ,mask face,


appearance during this
illness.
He had other problems: he suffered from eczema
at the age of 6-
months and'became wheezy by the age of g months.
oesophageal reflux was correcied by Tundoplication
Casiro_
year' He smiled at 6 weeks of age. Ai tg months atthe ige of l
he was abre to feed
himself with his- fingers. At 3 ye-ars of aLe he sat *ltrrort
was able to perform three-word sentenles. rrro*
rrppJrt ano
at s ve"o 5i"'gu, n"
can perform five-word sentences. His vision has
been describeJ by
his parents as'able to see and look at lighis,. His
Cysplasia, hearing-f,r, L""n
tested and is normal.
f gliosis.
There has been no such iilness in the famiry and
the parents do
- - not
want other children, as they are waiting for the opinion
of a
geneticist. The parents are not related.ihe
boy has a mild,
converge{t squint, there are no dysmorphic features,
present. There is no dystonia, but there is generarised
inJiremo, is
hypotonii with
areflexia and weakness. His oFC is on th; isth
centire, anJrris weight
and height are on the S0th and 25th centile, respectively.
The test
results are:
Serum AA, urine organic acid and AA Normal
MRI High signal in white matter,
not significant
EEG
Fast activity with polyspike
and slow background activity
ERG
Abnormal
NCS. EMG Normal
1. What are the three most likely diagnoses?
2. Which three other investigation, ."t,out,l be undertaken?
3. What advice should be giien to the parents?

Case 60
A 1O.-year-old girl presented with a history of severe
scoliosis, and
small and short wasted rower limbs. There is weaknesson
upper limb, with left subtle facial weakness. She
tn!lignt
l.'a is the
rnainstream school. There is no bowet probrem,
attends a
ni uut st',e *ets herserf
during the night if she has been stressed or irt, urtrrr"
rad mild
day at the age of 2 years and by night by tn"
*r. ;t;y
rrr'rh a 5 v"rir. ffl"
,,r.no. BY the
weakness.in her leg was noticed wlren sne waig "g"ofmonih, of age ano
has.bee.n investigated. Spinal dysraphism was
'ibeC as diagno"eO unO"no
further investigation has been carried out. Now;h; p;;;;;l"i,n
estigation. weakness.in the right upper rimb, severe scotiosis,l!v**"rrv'or ,
? optic upper timbs, ctumsiness with her right hand, tn"
oiogical .uUif" i;g;ii*i.f
weakness with marked lower limb iasting. ""A
leC An
t.'rtery she has a full range of eye movement and the results of
the fundi
onths he had examination are normal. The power in her upper limbs
is gooJ anO
: Cvstonia. she can walk downstairs whire hording onto ir,"
rairing. H.",. to"" r,
r
112 100 Grey Cases in Paediatrics for MRCPCH
ANSWERS 51-60
normal, as are the reflexes in her upper limbs. Power is zero, the tone
ir trvpJi""i. and reflexes are absent in the lower limbs' Sensation is Case 51
i;i;tii;iil rpp"r timbs. The tactile sensation is absent in the lower
t 1. BRECH {benion F
limbs. 2. Sleep EEG
A nerve conduction study \/as carried out on the lower limbs and
the 3. None
i"rrtt, rt" as follows: the sensory pa( is normal bnd the motor part
i" .U"ti*"t up to T3 level. There are no abnormalities in the upper Benign childhood epi
limbs, with normal overaltEMG' This is a common syr
1. What are three possible diagnoses? childhood; the onset ,

2. What diagnosis is visible on the spinal MRI? mean of 7 years. lt oc


with epilepsy. lt is ger
typical seizure occurs
during the daytime. S
in their mouth, which
seizure" These are sin
consciousness; the wt
or labial paraesthesia.
and/or speech arrest.
generalised seizures c
syndrome. The EEG is
and waves during slet
reported as normal. N
normal. Treatment car
frequentiy, on a daily
school and social life r
respond to medicatior
multi-drug treatment.
lead to CSS during ste
prognosis in these pat
development should n
may evolve into anoth
generalised tonic-clon

Case 52
1. e Constipation r
.)
i Bepeat abdon
2 I Picotax tor 2 d
d Start laxativeI

e Give advice at

Constipation with over


This is one of the comr
any time and usually o
rs unknown but possibr
hypothyroidism, anat s
detailed history and ex
3. What other two tests should be done?
Answers 5'l-60 t X3

A,NSWERS 51-60

Case 5l
BRECH {bl:nign Rotandic epitepsy of chitdhood)
I
2" Sleep EEG
3. None

Benign childhood epilepsy with centro-temporat


spikes
syndrome of idiopathic
llj,r,l-::TTon
cnrronood; the onset is between the
parriat epitepsy in
ages of 2 and iS y"ar", with a
mean o'f 7 years" lt occurs in about l*zsy, or i.r,ooi_ig;;;;li;r",
with epile.psy. rt is geneticaily determined and it can
be AS or AD. The
typicai seizure occurs during the night but sometimes
during the daytirne. sometiftes cniloren w;it oesciioe
*uy o..u..
in their mouth, which is followed by a generalised
. tu'nnv L"rine
tonic_clonic
seizure. These are simple partial without i*puir*"ni ot
consciousness; the warning signs""irrrli
are mainly motor uut can ue nuccrt
or labial paraesthesia._They mainty involve the face,
witl.r iativation
and/or speech arrest. Each seizure usuaily rasts
up to 60 s. i*conuary
generalised seizures.occ ur in.2}oh oip"ii"ntu
with this epif epsy
syndrome" The EEG is very characteristi, witn
and waves during sleep recording, and
c"ntrolt"ilpJiJr spites
fuite often the awake EEG is
reported as normal. No neuroimjging is'required,
as it will be
normal' Treatment can be given t6 piti*nt" who
are having .*iuun",
frequently, on a da-iJy basis] *rll,n-tl'ere is parent"r ,nii"ivto tn*
"no are
school and sociar rife of the chird
seizures shouli
respond to medication, and there is no "*".t"0.
need to give . i.,igf, J*" o,.
multi-drug treatment. carbamazepine should
oiavoidei, u. ii."v
lead to CSS during sleep and afebt s"f,ool performinc;. "'
prognosis in these patients is exceilent Th"
and schoor
development shourd not be affected. rn some casesf"rtor*r-n"" unc
these episodes
may evolve into another type of epirepsy in
adurt life, in the form of
generalised tonic-rlonrc setzures.

Case 52
1. e Constipation with overflow
?. iI
3.
Repeat abdominal examination
Picolax for 2 days
d Start laxative fiactulose and sennal
e Give advice about diet

Constipation with overllow


This is one of the commonest causes of abdominar pain.
rt can start at
and usuaily.one member of the flmiry rras'rrao it.
]ny_tjme
rs unknown but possible causes should
ir," lurr"
-
be ruled *t. ir.f rii"g
hypothyroidism, anal stricture, anO iirscf,sprrJng,s
disease. A
detailed history and examination will be r"if, ptrf . fn"i" I, no
n
"-f
114 100 Grey Cases in Paediatrics for MRCPCH

need to carry out an abdominal X-ray to diagnose constipation; the 3 weeks, which hi:s :
clinical diagnosis can be made fronr history and exarnination. optic neuritis is ve:"
Examination of the perineal region is necessary to rule out fissures or evelive. Most patier:
other abnormalities. A rectal examination should not be undertaken persistence of op:,: i

unless the historry indicates the need for it. Treatment involves remain irnpairecl.
prescribing a stool softener and starting with a simple stimulant such
as senna and lactulose. ln children who are soiling their clothes, an #ase 54
aggressive approach with a stronger laxative is probably needed to
clean them up in the first instance, then a softener and stimulant 1. N,Jephrogenic c
should be added. Difficult cases may need frequent enemas. which ?. ffiilateral r€na
=
-
are not tolerated very well by some children. Children with a heart 3" Abdomina
condition should not be prescribed strong laxatives. lnvolvement with a Urinary cr;:
a psychologist and nurse specialist is needed in difficult cases. b Calciurn,'c-=

FJephrocalcinosis
Case 53 Causes
1. b Cranial MRI 0xaiosis is an autcs:
a CSF for oiigoclonal bands metabolism, caus -l
2. i Optic neuritis thr;rs are three pa:::
b Retrobulbar neuritis wltli nephrocalcincs
d Septal orbital cellulitis with abscess iuv*nile type vrith '=
3. a i.v. methylprednisolone for 3 days depr:sition, which -
c Oral prednisolone 2 mg/kg for 6 weeks less oxaiate secrei -
excessive giycolate ,
Optic neuritis fixalate arrd L-gly::"
of this *unditicn ,s :
This is involvement of the optic nerve by inflammation, degeneration rryhe"lher high or r: :
or demyelination, resulting in impaired vision and pain around the nephrocalcincsis, --
involved eye/s. Both MS and optic neuritis have the same aetiology. leukccytes in type I
Some children with optic neuritis may develop MS later in life. Some intakes of oxalate a-
viral infections may cause optic neuritis, including measles, varicella, excretlon in type : :
and mumps. lt can be associated with Miller Fischer syndrome. lt is inhibit calcium oxa :
characterised by a sudden reduction in visual activity, either
unilaterally or bilaterally. lt is usually preceded by headache and AnothercBUSB, nrr:
painful eye movement. lt usually starts by causing blurred vision and asyrnptomatic. li s s
*alculi and eolic,
*-.
progresses to a complete loss of vision within a few days. Biiateral
involvernent has been reported in75% of cases. A swolien disc can ievels of phr:spi-::=
be seen in 75% of cases (neuropapillitis) and retrobulbar nes.i!"itis in absorp'iion of ca :
?5o/o in a normal fundi examination. Central scatomas wili be vlsible renal iubuiar def ::
in a visuai field examination. with delayed visually evoked response. sauses but none :,
Other causes sl-rou!d be excluded, such as malingering and hysteria, caieium and ox: ::
and other caLlses of optic nerve compression such as optic Sliomas' Other causes inc ,
pituitary adenoma (eraniopharyngiorna), and an AV maiforrnation D intake, sarcoic: s
pressing on the optic ne!-ve. Retrobulbar abscess follows septal hrypothyroidism :
ceNluiitii. An lylRl scan will show demyelination of one optic nerve or kidney.
both. Tlre risk of develop!ng MS in urtilateral optic neuritis is high and
the CSF may show pleocytosis and increased intrathecal lgG ease 55
production of oligoclonal antibodies" [!o treatment has been proven
io be of value bui corticosteroids have been recommended in the 1. d, e, Upper e-
form of rnethylprednisolone for 3 days and oral prednisolone for Cobblestone ::
Answers 51-60 11S

rg
3 weeks, which has been very herpfur. The foilow-up
of patients with
optic neuritis is very important a$ there is a high rlsr tr,at rrns
sor evolve. Most pati'nts recover normaror usefulvision oespite tr.,"
*av
persistence of optic atrophy, but.corour vision
and stereo-Jcopie v;sion
remain impaired.
uch
n
lo Case 54

]. Nephrogenic diaberes insipidus


lr 2' Bilateral renal multipte high densities shadow {nephrocalcinosisl
: 3. Abdominat US
Ll-:- a Urinary organic and AAs
b Calcium/creatinineurinaryclearance
Nephrocalcinosis
Causes
oxalosis is an autosomar recessive inherited inborn error of
metabolism, causing excessive endogenous oxalate synthesis, and
there are.three patterns of clinical disiase: {i) rnalignant infaniiL form
with neph rocalcinosis resurti n g i n ea rry end-stage r"ena t ta i u rej u t
t
juvenile type with recurrent stones, renal infecti6ns
and oxalate
deposition, which may cause arrhythmias; (3) benign adult form with
less oxalate secretion and longer suruival. Type 1 dxalosis results
frorn
excessive glycolate and oxalate excretion and type 2 causes excessive
oxalate and L-glyceric acid excretion with normj grycorate. niagnc,srs
of this co.n_dition is by demonstrating high oxarate-excretion and'
o whether high or no glycolate is found in-patients with renal stones or
nephrocalcinosis. The metaboiic enzyme defect can be detected in
iY. ieukocyte-s in type 2 but not in type r. Treatment is difficult, with fewer
,i'ne
lr la
intakes of oxalate and carcium. pyridoxine wirl reduce oxarate
is excrelion.in type 1, and magnesium and phosphate shourd bL given to
inhibit calcium oxalate crystal formation in the urine.
Another cause, hypercalciuria, is usually idiopathic and
asymptomatic. lt is sometimes present with haematuria or renal
calculi and colic. The plasma calciurn level is usually normat, u..r"
;" levels of phosphate and parathyroid hormone. lncreaseO gui -
ri absorption of calcium or increased bone consumption of ialciurn or
a
renal tubular defect of calcium reabsorption were suggested
as
causes but none could be proved. Avoiding an excessioe intake of
- calcium and oxalate is the treatment in these cases.
l other causes include distal renal tubular acidosis, excessive vitamin
D intake, sarcoidosis, renal tubular necrosis, hyperparathvrLlJirrn,
I Or'
hypothyroidism, proronged imrnobirisation, and meduilaiy ipo"g"
kidney.
anc

€n Case 55

1.
2.
d, e, Upper and lower GIT endoscopy with biopsy
Cobblestone appearance, and string sign

l
115 100 Grey Cases in Paediatrics for MRCPCH

diagnosis can be o.ir


Caseating granulomata by measuring the Ie,
3. a Oral prednisolone 2 mg/tg tor 2-3 weeks usually stored intr.a:
d Mesalazine cystinosis is the inia
b Polymeric"diet bone marrow and c:
decade of life, is less
Crohn's disease The infantile form ;s
The aetiology of this disease is unknown and its multifactorial symptoms that sta:
polyuria, polydipsi:
complex aetiology and pathology make it the most difficult
inflammatory bowel disease to manage. lt is a T-cell-mediated chronic rickets. End-stage r:
inflammatory disorder that may affect any part of the GlT. Non- life if the patient is .
caseating granulomata are characteristic but not always present. lt is kidneys and corneas
immunoiogical in origin but there have been many suggestions of Fanconisyndrome :
absorption is assoc :
other causes, such as bacteria, viruses or allergies. The disease is also
electrolytes and nu:-
characterised by skip areas with thick intestinal walls. lt is transmural
with submucosalthickening and with infiltration of lymphocytes and difficultto manage. I
lysosomes but, co:::
macrophages. The granulomata are a form of aggregated transformed
in patients with en:.
macrophages that may contain giant cells. The commonest presenting
cystinosis in transc :
features are growth failure, abdominal pain, diarrhoea and weight
Other complications
loss. The abdominal pain is characteristic, periumbilical, colicky and
relieved by defaecation. Other features may include erythema
mellitus, cerebral a::
hypercholestero la e -
nodosum, arthritis, uveitis and stomatitis. Perianal ulcers orfistulae
may also be presenting features. Anaemia, finger clubbing. with poor
weight gain and shofi stature, are other findings on examination'
Case 57
Upp--er dlT endoscopy as well as colonoscopy are indicated to confirm
tfie'diagnosis. The histological appearance will consist of non- 1. e Phaeochrc-
caseating granulomata, fissuring ulceration, focal inflammation, b Neu robla s::
submucosil or transmural inflammation, mucus retention and a Neurofib.: -
aggregation of lymphocytes. Barium swallow and follow-up should be 2. Cranial MRi
pliformeO after-biopsy, but a barium enema is not needed. ESR and Abdominal N4F
bRp are usually high and a way to assess patients during follow-up' Urinary VMA
Radiology can show a cobblestone appearance, aphthoid ulceration, Urinary dopan- -
granulariiy and mucosal oedema. S-Aminosalicylic acid derivatives,
Il"*"ntrl'nrtrition and surgery are the main ways of treating Crohn's Phaeochromocyton :
disease. Steroids are used to induce remission and azathioprine can
This is an adrena -.
be used for its steroid-sparing effect. The prognosis is much better in
paediatric patients and the risk of cancer is ill defined' neuroblastoma a": ,
the main causes :' ^
1A% of affectei i::
Case 56 but occur in the a:: _
Hippel-Lindau s\,- : -
1. h Urinary organic acid and AAs
Hypertension, s\.,:::
a Slit lamP eYe examination
rnanifestations c. : -
b Renal US
catechoiamines :-:
2. g Cystinosis
Suppression or s: - -
3. c Serum AAs
procedure. Abdc* -
h Fibroblast cell culture
tumour. Whole-l: : .

abdorninal diseas= !
Cystinosis out a biopsy or i-: '-
This is a metabolic disorder that is inherited as an autosomal cxybenzamine a- I :
recessive condition; the gerre is located on chromosorne '!7p. Prenatal

.!'ii
Answers 51-60 117

diagnosis can be offered in famiries who have


bv m.easuring the rever ot cvsteine i"i't," iioi"ur"Jii. with cystinosis,
a chird
usually stored intraceilurarry in rysosomes. Trre crll",""'"'i"
cystinosis is t[e infantile type, *nicf, ,*".t, the
. ort
=irlr"'iorm ot
kidneys and eyes,
bone marrow and other tissues. The adutt form
i*h"" ,L."ro
are";;;;J
decade of life, is ress benign ano trre r,ion.v.
usuary not invorved.
The infantire form is chara-cterisiJ uv , nor.nar
baby at birth, with
symptoms that starr t!1nqe9r from ihe age of
6 .;"ih; ;;.lrling
I chronic !?lydipsia, FTT, be hyd rati o n, a n? u nex p ta i n eJ rever-witn
t:ly^yll?rnd-stage
renar fairure wiil occur within the first to yeais
'cKers.
life if the patient is nottreated. crystars of cysteine of
can be seen in the
:nt. lt is kidneys and corneas, and the.v"tLin"l*erin
reukocytes wiriue nigr,.
1S Of Fanconi syndrome due to geneiarised fairure
of proximar tuburar
€ is also absorption is associated with cystinoiis. Carerut-b;i;;;il;"
rsm ural electrolytes and nutrition is.very important, as
these patients are very
ies and Cystemi.ne *;rr ["rp to mobitize cysteine from
rsJormed l:*:,lt^"^lalaSe.
out, compliance is poor. Renaltransplantation
'esenting :I"_r_r:-9r
rn patients with end-stage renar fairure. is efflctive
There is no recurrence'of
:ight cystinosis in transpranted kidneys anJ renar function
ky and oth.er complications associated *iir, .v.linosis
i. .rii.i*iory.
include diabetes
I mellitus, cerebral atrophy, corneal jegl;eration
anO
stulae hyperchotesterotaemia, *f,t.f, Uii iJter in life.
ith poor "rii"
ion.
r confirm base 5/
1. e Phaeochromocytoma
3n,
b Neuroblastoma
d
;hould be
a Neurofibromatosistypel
SR and
2. Craniat MRI
Abdominat MRt or CT
ow-up.
Urinary VMA
Bration,
Urinary dopamine
atives,
l Crohn's Phaeochromocytoma
ine can
better in This is an adrenal medutary tumour that
occurs more frequentry than
neuroblastoma and oanglioneuro*r. *t,i.n
u"ryi"."]ft ir'"n"
the main causes of h"vpenension in "r" occurs bilaterarry inot
and
fi% of affected individuals; rrrli or f,"ru't-un.,orr.
"'hirdr"n are extra-adrenar
but occur in the abdomen. lt is atso associated
with Von
syndrome and neurofibromatosis type 1.
iltJ::Lf:,lau
Hypenensron, sweating, headache, and
visual btuiiing are common
manifestations of phaebchromocytorn..
catecholamines thei r metabol ites'a re
iign urine levels of
Oia jnostic ma rkers.
suppression or stimurarr'on t"-{ri" rlqrir"o
procedure' Abdornina-r for the diagnostic
-c,Tor MRt scans?e needed to iocarise the
tumour. Whote-body Ml?.g r*qrireJlo rut" ort th;";i;;;;;r"
.is
abdominal disease. Stabil isi n g .r-r i ro i"n-l"io.re
removi n g o r ca r ryin g
out a biopsy of the rumour is Essentiar. an
lt ,rpr.,. nrr.il"i irrg'rr.t, ,*
oxybenzamine, and a beta-b!ock*, . ,,"h'ra proprancior
:. Prenatal ars intrrcr,i;,,rrl
118 100 Grey Cases in Paediatrics for MRCPCH

to controltachycardia and cardiac dysarrhythmias. Fluid


management is important before removing the tumour. The stored in a normal brain (l
operation for phaeochromocytoma should be carefully planned, and accumulates over the year
invasive monitoring of BP and HR during the operation is very normal phenomenon. Late
important. The outCome of surgery is usually good. linked to chromosome I 1.
developmental delay follo
of age. Seizures are usuall
Case 58 Dementia will follow; affer
1. b Joubert syndrome of 3*6 years and cjie befor,
g Myotonic dystrophy oc ilrs late with macular. r
h Ataxiatelangiectasia Et : is characteristic with
2. a Alpha-fetoprotein with production of spikes
e EMG response to photic stimulz
f lmmunoglobulin level and somatosensory-evoke
biopsy will show curviline
Joubert syndrome vascular and smooth mus
biopsy.
This syndrome consists of familial agenesis of the cerebellar vermis,
episodic hyperpnoea, abnormal eye movement, ataxia and
retardation. Other features include facial asymmetry, retinal Case 60
anomalies, disc coloboma and renal abnormalities. The commonest
presentation will be with respiratory problems, which include panting 1. Syringomyelia(syrnix
respiration interspersed with long pauses that are not associated with Spinal dysraphism
any changes in blood gases. Respiratory problems usually start Denervation
during the neonatal period, with cyanosis. The vermis is dysplastic 2. Syringomyelia (syrinx
and cerebellar peduncles are small. The electroretinogram is flat or 3. MRI of spine and brair
markedly depressed. Ataxia and retardation develop later. This Nerve conduction stur
syndrome can be inherited recessively but sporadic cases occur. The and upper limbs
MRI and cranial CT will show the abnormalities. The CT appearance
will show a brainstem that looks like a molar tooth. The MRI scan is Syringomyelia
more definitive and will show an umbrella-shaped sign of the lower This is characterised by c:
fourth ventricle and atrophic vermis with a horizontal superior in length and may extenc
cerebellar peduncle. There are other syndromes associated with is located in the grey ma:
vermal agenesis, such as Dandy-Walker syndrome, Aicardi region. lt may follow trau:
syndrome, Smith-Lemli-Opitz syndrome, Goldenhar syndrome and congenital malformatior :
Meckel-G ruber syndrome. depend on the cyst's loca:
spinal cord, pain and te;-rl
Case 59 first. Symptoms may be u
before the shoulders wher
1. Mitochondrial disorders (recessive) pressure sensations wili g
carbohydrate-deficient glycoprotein (cDG) syndrome disturbs the posterior col:
Non-metabolic genetic disorder (Batten disease) sensation in the hands it.
? Rectal biopsy Scoliosis is common ano I
White cell enzymes A large cavity may produ:
Transferrin level limbs. Sphincter contro -
3. No further pregnancy until they have genetic counselling progress very slowly arc
children presenting witr :
Batten disease limbs is vital. Bulbar s;tr-s
This disease is characterised by storage of ceroid-lipofuscinoses diagnostic and can dif{!.e
lipopigments (ceroid) in the brain, which is different from pigment syringornyelia. A syr-ir: : :
decompress the cav::, -
=
OUESTIONS 61-70

Gase 61 p .

A girl with a known history of chronic renar fairure has been on


haemodialysis three times per week for the past 1g months and is
waiting for a kidney transplant. Her original problem followed
meningococcal septicaemia. she also needed to have her left foot
amputated at that time. she. is suffering from right hip pain, lethargy
and muscle weakness. She is t2 years6ld. sheiays ir,it bones
"riher
ache, and she finds it very difficuit to go upstairs and spends ail her
time in her bedroom on the first floor. she cannot do exercise or walk
a long distance. The hip pain was associated with difficurty in frexing
hdr hips. Her BP is stabre and her BM is 4.5 mmor/r. Her hip US was
reported as normal and an X-ray of the long bones shows osteopenia
and reduced mineralisation.
1. What abnormality is visible on this X-ray?

z. What are the two causes of these bone abnormalities?


a Rickets
b Secondaryhyperparathyroidism
c Chronic renal failure
d Metabolic acidosis

ry
i
122 100 Grey Cases in Paediatrics for MRCpCH

e Aluminium toxicity abnormalities were fou


f Renal dialysis class parents, who botf
g 1,25(OH)rD.deficiency history of headaches or
h lnfection systemic examination,
i Heactive arthritis central nervous system
j Malignancy mother insists that her
K JCA admitted for video teler
3. What is the diagnosis? have been described as
a Chronic renal failure horizontally and she lie
bewveen 30 and 60 s. T
b Renal osteodystrophy, secondary to chronic renal failure that is associated with'
c Hyperparathyroidism
d Distal renal tubular acidosis 1. What is the most ii
e Fanconi syndrome a Vasovagal atta
f Familial hypophosphataemic rickets b Seizures
g JCA c Vertigo
h Psoriatic arthritis d Benign paroxr-
i SLE e Ataxia
f Dystonia
g Migraine
Case 62 h Transient iscni
A 2-year-old girl was born at term with no neonatal problems until i Dancing eye s'
the age of 1 month, when she was seen by a Gp because she was 2. What is the approp
floppy, not moving her legs and not interested in feeding. She was
admined to the local hospital and treated with antibiotic; for 2 days, Case 63
then discharged home on day 4, with the diagnosis by US of GOH.
Anti-reflux medication was started and she was well until the age of A 4-year-old child pres
5 months, when this started to happen again. She became ftopiy, is not eating and his Bl
could not bear weight on her feet and lost her head control. sile was lethargic and compla;^
also having a problem with feeding as she would choke on her food. centrally. He passes be
These episodes of floppiness happen once every week. The anti- drinks a lot but does rr
reflux medication was stopped and she was onty given Gaviscon. At disease 1 year ago. Hs
the age of 6 months all medication was stopped and she seemed to time he was unable ic
be all right untilthe age of t1 months. when she started crawling, her sisters, who are healr-
legs were weak and gave way. She was not able to sit property. An but is still active anc ci
EEG was carried out during one of these episodes and'was reported organomegaly. His brr
to be normal. She was referred to a specialised unit and a full normal, as is the res;':
metabolic screen, including a fasting test, was carried out. All results normal, but his abdom
came back normal. masses in his kidner.,s
Other test results a.e
At the age of 18 months she started to fall over, and with these
episodes she was blinking her eyes quite frequently throughout. An Na
EEG was carried out ictally and post-ictally, and *as reporied to be K
normal. The result of an ophthalmological examination of VEp and U
ERG was normal. A brain MRr was found to be normal. Now these LT
episodes happen three to four times per week. They last up to 5 min Ct
HCO3
each time and following these her gait is affected. Some oithese
epis.odes.can last up to I hours. Recently she starled yawning, Ca
pH
blinking he1 ey_es quite often and turning her head to one sid6'during
these attacks. Following an attack she feels tired and goes to sleep f6r BM
up to t hour. She had five UTls investigated fully and no Osmolality
Ouestions 61-70 123

abnormalities were found. She is the only child of healthy middle-


class parents, who both work from home. There is no such illness or
history of headaches or epilepsy in the family. Upon a general or
systemic examin4tion, no abnormalities were found. The results of a
central nervous system (CNS) examination were normal but her
mother insists that her gait is not as it should be. The girlwas
admitted for video telemetry for 2 days. she had three attacks, which
have been described as follows: she goes pale, her eyes move
horizontally and she lies motionless during each attack, which lasts
bewveen 30 and 60 s. There is no evidenciof abnormriity on th; EEG
rl failure that is associated with these attacks.
1. What is the most likely diagnosis?
a Vasovagal attacks
b Seizures
c Vertigo
d Benign paroxysmal vertigo
e Ataxia
f Dystonia
g Migraine
h Transient ischaemic attack
ems until
i Dancing eye syndrome
she was
2. What is the appropriate treatment?
. She was
for 2 days, Case 63
i of GOR. A 4-year-old child presented with weight and height < 3rd centile, He
rhe age of is not eating and his BMI is 17. He vomits at leasifourtimes/week, is
e floppy, lethargic and complains of abdominal pain that is not located
ol. She was
centrally. He passes between 0.5 and 'l L of urine each day. He also
,n her food.
drinks a lot but does not eat much. His father died from a bowel
're anti-
disease 1 year ago. He suffers from cramps in his legs and for some
aviscon. At
time he was unable to walk (as if he is paralysed). He has two older
seemed to
sisters, who are healthy and doing well at school. He looks very thin
:rawling, her but is still active and cheerful. He has no skin marks or
rperly. An
organomegaly. His blood pressure is 80/60 mmHg, his fundi are
s reported normal, as is the result of a cNS examination. Hia cxR is described as
r full normal, but his abdominal US is abnormal, with multiple echogenic
- All results masses in his kidneys, although his liver and spleen are.normal.
Other test results are;
these Na 138 mmol/l
rghout. An t\
rted to be 2.3 mmol/l
I VEP and
U 4.2 mmol/l
rw these
LT 60 mmol/l
cr i02 mmol/l
rp to 5 min
HCO3 14 mmol/l
rf these
Ca 2.12 mmol/l
ning, pH
side during 1.35 mmol/l
BM 4.5 mmol/l
; to sleep for
Osmolality 297 mosmol/l

i
124 100 Grey Cases in Paediatrics for MRCPCH

MSU
pH 6.1 febrile and miserable, nr
b.p.m., Sat level of 899.
wcc 10
There is marked interco
BCs " 5, lungs. The liver is 1 cn:
Calciurn/creatinine ratio High
200 mosmol/l is no heart murmur an3
Osrnolality
't. What are the most likelY causes? CXR, which was descrl=
(pneumonitis) with mur:
a Fanconi sYndrome right costal margin. The
b Chronic renalfailure
c Nephrogenic diabetes insiPidus CRP 250
d Distal renaltubuiar aeidosis {TA) Hb 9.49:
e Proximal renalTA WCC 24 x',i
f Hypercalciuria PLT 350 x '
g PrimaryhYPerParathYroidism ESR 10
h PrimaryhYPothYroidism lg Norri a
2. What is the diagnosis? CD4/CD8 ratio Norma
3. Which four other investigations may help the diagnosis? CH50 Norma
a Amrnoniurn chloride loading test Heaf test No ree
b Serum AAs Sweat test Na < 4
c Water deprivation test
d PTH
1. What other invest';
e TSH/r4
a Bronchoscop.r'
t Calcium/Cr ratio
b Chest CT
Chromosomes
c Measles lgl\4 a'
s d Rubella lgtv'i a-
h IVU
DMSA scan
e NPA
i
Urine organic acid and AAs
f Echocardiogra:
i S, Mantoux test 1

, h HIV antibody
Case 84 i Mycoplasme.'
with her mother and had a history of having
j Gastric wash f :
A toddler presented 2. What is
past 7 days. This started with a high temperature. the most 1,.:
been unwell for the a HIV infection
She wouldn't eat and was irritable. This lasted for 2 days, then she b Giant-cell pn:-
developed a rash all over her body, which started at her neck and c Cystic fibros
then spread. The GP saw her at this time and diagnosed a viral rash.
s

Her temperature stayed down for another 5 days but then she had a
d Diamond-Scr,
very hig'h ternperature agairr, with rigors and irritability' Her mother
e Combined ir-
gave her paracetamol overnight, but the girl was still not able to sleep
f Foreign bocy
g Bronchiectes s
Ind was described by her mother as'jurnpy ail night" She is one of h lnterstitial c-=
seven children, of a travelling family that now lives in a caravan.
Three of her elder sisters and one brother have been unwell before
her, with a similar illness and rash, but recovered within 5 days and Case 65
only have the remains of the rashes on their bodies. The rnother also A 7-year-old boy r,,as ':
said that all of them had watery eyes and were drooling. There is no tonic-+lonic seizu re : -
peeling of the skin and there are no ioint problems. The toddler is 18 with an altered ler.e :-=

month; old and her mother is not sure which of her children have before, he had bee" --
been vaccinated and which have not, but she thinks her daughter has drowsy and sudde- ;
had the vaccine (BPT + polio and Hib) only once. There are a dog and level of 1.1 mmol
a cat living wiih the family. The parents want to move the family to test shows +-+- kei- - : :
another site as soon as their daughter starts to get better. The girl is of rnuscle parns. -: -:
Questrons 61-70 125

febrile and miserable, has a fading rash, and a RR 45/min, a. HR of 140


b.p.m., Sat level of 89% in air and g5% on 4 L O2via a facemask.
There is marked'intercostal recession, with coarse crepitation on both
lungs. The liver is 1 cm enlarged and there is no splenomegaly"There
is no heart murmur and she is alert. She was admittbd.anO hab a
CXR, which was described as showing bronchopneumonia
(pneumonitis) with multiple small lesions in both lungs below the
right costal margin. The test results are:
CRP 250
Hb 9.4 gldl
wcc 24x10ell (N 40%, L 58%)
PLT 350 x 10e/l
ESR 10
I lg Normal
l CD4/CD8 ratio Normal
Pnosis? cH50 Normal
Heaf test No reaction
Sweat test Na < 40 mmol/l
1. What other investigation/s may help the diagnosis?
a Bronchoscopy with lavage
b Chest CT
c Measles lgM antibodies
d Rubella lgM antibodies
eNPAj
f Echocardiography
S, Mantoux test 1:1000
t , "ti h ' HIV antibody
i Mycoplasrna titres
f having
j Gastric wash for Mycobacterium
2. What is the most likely diagnosis from the CXR?
tfien she a HIV infection
and b Giant-cellpneumonitis
viral rash. c Cystic fibrosis
she had a d Diamond-Schwachmansyndrome
mother e Combined immune deficiency syndrome
able to sleep
f Foreign body
h is one of g Bronchiectasis
haravan.
h lnterstitialpneumonia
rellbefore
-5 days and Case 55
lmother slso A 7-year-old boy was found in his bed, having a generalised
|.There is no tonic-clonic seizure early one morning. He presented twice to A & E
)bddler is 18 with an altered level of consciousnesJfollowing a UTl" The night
ben have
before, he had been unwell. The next morning,-he vomited, w-as
r'daughter has drowsy and suddenly started fitting. A BM tei showed a glucose
re a dog and levelof 1.1 mmol/|, which was coniirmed by laboratory reluh- A urine
h famity to test shows +r+ ketones, ++. proteins and no organisms. He complains
tr- The girl is of muscle pains. He had had anothq acimissro-n from the outpaiient
126 100 Grey Cases in Paediatrics for MRCPCH

department, w,ith a blood glucose level of 2.2 mmol/l in the past. A


Flffi anO F* test showed left VUR, which was diagnosed when_he ANA ..
*"i flnn"ths old. He had had two other admissions in Seoul, South
ReF
K;;;, for similar episodes, and also had had a febrile convulsion
ESR 3
when he was 3 Years old. ' CRP 2

Plasma AA, lactate, NHo, very-long-chain fatty acids, GH, cortisol' and 1. What are the '
TFT levels were all within normal values. urine organic acid and AAs a ECG
were normal. b MRI of r:
1. What is the most likely cause of his seizures? c Cathete- s
a Lack of sleeP d Echoca':
b HyPoglYcaemia e CXR
c High ketones in urine f Abdorr ':
d High blood Pressure g X-ray oI q
e Uraemia h 24-hou'i
2. What is the most likely diagnosis? 2. Which three :
3. How could the diagnosis be confirmed? a ASO tji,::
b Throat s,'"
c Anti-D\a:
Case 65 d Blood f -
joint pain e Blood :- '
At the age of 11 years, a young girl presented to A&E with f DNA Cc,:
unO *ru"not feeling well' She was treated for an episode of.
tonsiilitis 6 weeks igo, which she recovered from completely but has
SCK
h Chron':s
fett tired ever since Ihen. She has a rash on her back and lower limbs i AAs
that looks like bruises' She finds it very difficult to sleep j Swea:::s
wiinout using three pillows under her head, as her breathing is kls
years
muetr Uetterir;tn ther". Her {ather died from a brain tumour 3 I Thyro c '.
ugo. goth her sister and brother are well. Her mother is a full-time
m Viral t:'=
tJacher, and two cats and three goldfish live in the house with them. EBV
She does not go to school anY more as she is too tired' Upon
e*piration, a gitlop rhythm whh-early diastolic rnurmur at the mid- s.n ECHO was::-
itJrnat edge ian be heard. A soft systolic murmur can also be heard ,aflet was thl:-.: -
,i1fr" upui. An ejectlon systolic click can be heard at the lower sternal 'aradoxicalll,. --:
eOge. fhe liver measures 3 cm below the costal margin, and has a uicuspid with :: '
smooth surface. The test results are: gradient of 36 - -
Na 141 mmol/l 3. What lesi: - s
K 3.9 mmol/l a l\4itra ,:
U 6.2 mmol/l b Aorti: s-=
Cr 55 mmol/l c Bicus::
Ca 2.12 mmol/l d Mitra .:
Alb 36 gil e Mitra '':
Total protein 70 gil f ASD
ALT 35IU/I g VSD
Ark. Ph 280 lu/l h Pulrr: ^,
Hb 11.2gidl i Tricls: :
WCC 6.9 x 10s/l 4. What is t^= :
PLT 450 x 10s/l a Acute --;
tNR 1.1 b Endo::" :
PTT 50 S c Cong=- -
d [4yo:=':
Questions 61-70 127

ANA Negative
ReF Negative
ESR 60 mm/hour
CRP 25
1. What are the'most urgent investigations?
a ECG
b MRI of heart
c Catheterisation
d Echocardiography
e CXR
f Abdorninal US
g X-ray of knees
h 24-hour ECG
2. Which three other investigations are appropriate?
a ASO titres
b Throat swab
c Anti-DNase
d Blood film
parn e Blood culture
f DNA double strand
has SCK
limbs h Chromosomes
i AAs
j Sweat test
years klg
I Thyroid function
them. m Viral titres for: coxsackievirus, adenovirus, parvovirus and
EBV
mid- An ECHO was performed and showed that the anterior mitral valve
heard
sternal 'eaflet was thickened and the posterior leaflet was moving
paradoxically. There is mild mitral regurgitatlon. The aortic valve is
a
bicuspid with AR flow velocity. The Ah now velocity was 3 mls with a
gradient of 36 mm. The peak systolic flow across the valve was 2 m/s.
3. What lesions are associated with ECHO and clinicalfindings?
a Mitral valve stenosis
b Aortic stenosis
c Bicuspid aortic valve stenosis
d Mitralvalve prolapse
e Mitral regurgitation
f ASD
g VSD
h Pulmonary stenosis
i Tricuspidregurgitation
4. What is the diagnosis?
a Acute rheumatic fever
b Endocarditis
c Congenital heart disease
d Myocarditis
128 100 Grey Cases in Paediatrics for MRCPCH

e JCA a Rehydration
f SLE b Regular sedation with
2 c Regular sedation with
67
' d Maximise L-dopa
Case e Ventilate and sedate
A 13-year-old girl was admitted to hospital with a history of f Daily CK and U&Es
deterioration in her dystonic movernents in the last 2 days. She is g Transfer to PICU
known to have dystonic cerebral palsy. Co-careldopa helped slightly h Regular analgesia
and she was given 10 mg oral diazepam when needed. She had had
diarrhoea, which stopped 24 hours before admission. She was born Case 68
at term without pre- or postnatal problems. Her problem started at
the age of 2 years, when she couldn't walk and developed abnormal A male infant was seen in the r

movement, and since then her global development has been behind. tachypnoea and no feeding fo'
She is conscious and able to communicate with her mother, but the normal vaginal delivery. His rn
dystonic movements continue and she finds it very distressing. The he has wvo older siblings, ager
dose of Co-careldopa was increased to 125 mgltwice a day and she healthy. The father has obtainr
was given one dose of chloral hydrate and 10 mg of oral diazepam. 7 months. The boy is now 4 n-
She was still having violent dystonic movements t hour later, and the fed while they live in a refuge-
results of the blood tests are: OFC is 42 cm, HR 120 b.p.m., i
intercostal recession, and tne'
Na 130 mmol/l reduced in both bases, and tii
V 4.7 mmol/l measures 3 cm and there is c:
Ur 6 mmol/l his neck. He is sleepy. O, was
Cr 120 mmol/l with a reservoir, and the Sat D
ALT 55 rull tonsils and there are two birtt
Atk. Fh 330 rufl admitted for investigation arc
Hb 1a slot
PI.T 250 x'l0s/l Hb 9.6 g/dl
CRP <5 WCC 15.9 x 10P,1

ESR < 2 mm/hour PLT 250 x 1Sr1


Urine Positive for blood ALT 25 ru/l
Stool Negative Atk. Ph 125lufl
TSH 1.56 mU,4
1. What is the most likely diagnosis? T4 15 mUfl
a Status epilepticus CRP 145
b Choreoathetoticcrisis lgG 22.2
c Sydenham chorea lgA 5.4
d Parkinsonian-likecrises lgE <2
e Oculojerk crisis Abdominal US Large splee-
2. What are three other important tests? MSU Negative
a EEG Mantoux test 1:10000 & r:'
b ASO titres
c Daily CK 1. Which two abnormalities
d Urine myoglobulin a Hyperinflation
e Muscle biopsy b Consolidation of the
f Urine toxicology c Bilateral promine:r: -
g Co-careldopa level d Bilateral nodular in''
h Cranial MRI e Pulmonary oederna
i EMG f lncreased brorrchiat r
j Daily U&E's g Right upper lobe ooa
3. List the correct steps of her management l'l lnterstitial granuia:r:
Ouestions 61-70 129

a Rehydration
b Regular sedation with diazepam
c Regular sedation with chloromethiazole
d Maximise L-dopa
e Ventilate and sedate
f Daily CK and U&Es
he is g Transfer to PICU
slightiy h Regular analgesia
rad had
rs born
Case 68
ted at
,normal A male infant was seen in the rapid access clinic with a history of
behind. tachypnoea and no feeding for the last u18 hours. He was a full-term
but the normal vaginal delivery. His mother is a refugee from Zimbabwe and
rg. The he has two older siblings, aged 4 and 6 years, who are well and
rd she healthy. The father has obtained a visa to enter Britain in the last
epam. 7 months. The boy is now 4 months old and has been totally breast-
. and the fed while they live in a refuge-detention centre. His weight is 4.600 kg,
OFC is 42 cm, HR 120 b.p.m., RR 30/min, Sat level in air 91% with
intercostal recession, and there is subcostal recession. Air entry is
reduced in both bases, and there are no added sounds' His spleen
measures 3 cm and there is one large cervical gland on the left side of
his neck. He is sleepy. O, was given at a rate of 5 l/min via a facemask
with a reservoir, and the Sat picked up to 95-97o/o. He has large
tonsils and there are two birthmarks on his abdomen. He was
admitted for investigation and treatment. His test results are:
Hb 9.6 g/dl
WCC 15.9 x 1Os/l
PLT 250 x 10s/l
ALT 25 tu/l
Ark. Ph 125 tU/
TSH 1.56 mU/l
T4 15 mU/l
CRP 145
lgG 22.2
lsA 5.4
lsE <z
Abdominal US Large spleen measuring 3 cm
MSU Negative
Mantoux test 1:10000 & 1:1000 Negative
1. Which two abnormalities appear on the CXR?
a Hyperinflation
b Consolidation of the lower lobes
c Bilateral prominent hilars
d Bilateral nodular infiltration
e Pulmonary oedema
f lncreasedbronchialwallthickening
g Right upper lobe opacity
l'r lnterstitiaI granulation in both ]ung fields
130 100 Grey Cases in Paediatrics for MRCpCH

chickenpox and recc . ='t


8 months was beicr," :- -.
and she started to \.,: :
language and was r,='-
hearing is normal. T-: -
s0rnetimes can be ve-. :
sonretinles there is a : =
The skin rash starlec : -
is eczema-like ras" - .
noticed at the age c' '
recently. She was ie'."-
history sf FTT, was I ::
receiving treatmeni': -
below the 3rd centil: - i
never noticed any te:'.
family history, and :- = :
Examination revea':: .
and a small head. S'= ,
support of a cha:' a' : :
support. She has 'rc ).
t. What is the most likely cliagnosis? lower limbs and rec - : =
a Bronchiolitis Tremor is marked ir - ='
b Aspiration pneumonia A fundoscopy shc'i s : :
c Congestive hea( failure described by a nax -'.
d Pseudomonascariniipneumonia (pCp) papillae'.
e Pulmonarytuberculosis
f Cystic fibrosis The following invesi ;::
g Allergic alveolitis chromosomes, AAs :":
h Bronchopneumonia pyruvate, brain MF :' '
3. Which three other tests should be carried out? and axonal neuropa:-.
a HIV antibody 1. What are thres : -=
b pH study 2. List four f u rt h: - -
c Sweat test 3. What three ste:: r -
d PG24
e NPA
f Lymph node biopsy
g Heaf test Case 70
h CD4/CDB ratio A 9-year-old girl p'e
i Bone marrow biopsy headache. She ha:
j Bronchial lavage for PCP lethargic and haC :
k Lateral CXR measured by he. 3 tr
I ECHO mrnHg; at this ti.:'=
was described as .
advice frorn a pae:
Case 69 cn the basis of c a -
pressure was ne\ 3-
AZli-year-old presented.with a history of developmentaldelay, FTT,
photosensitivity, scoliosis, tremor, no tears, and rashes from ihe age and is severe eic,
Faracetarnol he p=:
of 1 year. she was an FTND and there were no neonatal problems.
Her development was normal for the first 6 months, when she had Soes but she ne,:'
henigr:r and wilr : -
Ouestions 61-70 131

chickenpox and recovered without a problem. The OFC at the age of


8 months was below the 3rd centile. She sat at age of 14 month!
and she started to,walk a few steps. At this time she had no
language and was wearing spectacles for shr:rt-sightedness. Her
hearing is normal. Tremors started at the age of 1 year and
sometimes can be very bad. The tremors are mainly proximal and
sometimes there is a period of up to 1 month without any tremors.
The skin rash started on the upper limbs and has now disappeared. lt
is eczema-like rash in a lineal pattern on exposed areas. Scoliosis was
noticed at the age of 1 year and has become more prominent
recently. She was referred to a hospital outpatient department with a
history of FTT, was diagnosed as having GOR, and is currently
receiving treatment for this. Her weight, height and OFC are still
below the 3rd centile. Her mother describes her as a'dry baby' as she
never noticed any tears during episodes of crying. There is no such
family history, and the parents are Caucasian and not related.
Examination revealed a small child with severe scoliosis, sunken eyes
€ and a smali head. She is constantly drooling. She stands with the
support of a chair and can only walk one or two steps without
support. She has no skin problems. all reflexes are absent in the
lower limbs and reduced in upper limbs, and she has stiff legs.
Tremor is marked in her hands.
A fundoscopy shows pigmentary retinopathy. Her tongue is
described by a maxillofacial surgeon as'smooth without fungiform
papillae'.
The following investigations were carried out and are all normal:
chromosomes, AAs, organic acids, lactate. NHo, CSF lactate and
pyruvate, brain MRl, EMG and a skin biopsy. An NCV was carried out
and axonal neuropathy was suggested.
1. What are three differential diagnoses?
2. List four further investigations.
3. What three steps should form the management plan?

Case 70
A 9-year-old girl presented with a history of joint pain, fever and
headache. She had lost 6 kg in the past 2 months, was increasingly
lethargic and had poor concentration. Her blood pressure was
measured by her GP when she was 8 years old and was 110/75
mmHg; at this time she presented with a rash all over her body that
was described as 'a small pinpoint rash'. This rash disappeared after
advice from a paediatrician to give a S-day course of oral prednisolone
on the basis of diagnosis of Henoch-schonlein purpura. i{er blood
pressure was never checked again. A headache started 2 days ago
Jelay, FTT, and is severe enough for her to find it difficult even to chew.
om the age Paracetamol helped for 2 hours only. The pinpoint rash comes and
,roblems. goes bl,t she never bothered to ask for help as she has been totd it is
r she had benign and will go in time. $-{er BF was 130/80 mmHc on three
132 100 Grey Cases in Paediatrics for MRCPCH

occasions. The rash is mainly on her arms and legs and is present to
a lesser degree on her face and chest; it does not blanch when ANSWERS 61_70
pressed. She is afebrile at the moment but hertemperature can be
high on some days* The longest period she has had without a Case 61
temperature was 7 days, 2 weeks ago, after a course of antibiotics 1. Renal osteodys'i'cr
and non-steroidal anti-inflammatory drugs (NSAlDs). No other 2. c Chronic renal '
abnormalities were found on systemic examination, apart from a a Rickets
swollen left ankle joint. An X-ray of the joint showed soft-tissue 3. b Renal osteoC;
swelling. Other test results are:
Renal osteodystrophy
LFT and U&Es Normal
ESR 70 mm/hour This is due to disturba
CRP 25 secondary to chronic :
Ferritin 1600 hyperparathyroidi s rn,
PLT 345 x 10e/l hypocalcaemia due to
DNA double strand Negative vitamin D metabolisrn
Renal US Normal biochemical changes;
Cranial CT Normal with contrast is mainly related to tin
Hb 12.5 gldl is worse with congeni':
wcc 6.8 x 10s/l ml/min/l.73m2, renal s
ANA Weakly positive skeletal symptoms are
(short stature), bone p
hydroxych o eca cif ero
I I
1. What other tests may help the diagnosis? start to appear in patie
a Henal artery angiography hyperphosphataemia,
b Renal biopsy hyperparathyroidism i
c Pin-point lesion biopsy )letary phosphate res
d HBsAg ,icarbonate) are effec
e Lyme disease serology i.voided in children. R
f DMSA scan therapy in patients w'i:
g Cranial diffuse weight (DW) MRI complications.
h Muscle biopsy
i Joint biopsy
j Mycoplasma titres
k Skin biopsy Case 62
I Renal US
2. What are the most likely diagnoses in order? d Benign parox
a JCA Good and no trea
b SLE
c Kawasaki disease Benign paroxysmal vr
d Henoch-Schonleinpurpura
e Behget's syndrome This is a disorder cr
f Lyme disease by a sudden onse: :'
g Hepatitis B The child will lie ma:
h Polyarteritis nodosa held by a parent. Aie)
i Phaeochromocytoma events. The child '*
j Renal TB These episodes mar
k Ehlers-Danlossyndrome replaced by headacrr
I Scleroderma of migraine is alwa
''s
m Mixed connective-tissue disorders antimigraine trea'll-.e
associated with hea:
Answers 51-70 '133

fesent to
EN ANSWERS 61_70
:can be
ta Case 61 t
biotics 1. Renal osteodystrophy
her 2. c Chronic renal failure
iom a
a Rickets
Fre 3. b Renal osteodystrophy secondary to chronic renal failure

Renal osteodystrophy
This is due to disturbances in bone and mineral metabolism
secondary to chronic renal failure. Phosphate retention, secondary
hyperparathyroidism, skeletal resistance to parathyroid hormone,
hypocalcaemia due to malabsorption of calcium, and changes to
vitamin D metabolism in patients with chronic renalfailure are
biochemical changes associated with renal osteodystrophy. Severity
is mainly related to time of onset, of renal failure and to the cause. lt
is worse with congenital causes. lf the GFR is more than 25
ml/min/l.73m2, renal dystrophy is very unlikely tb occur. Many
skeletalsymptoms are associated with this; including poor growth
(short stature), bone pain, and skeletal abnormalities. 1-Alpha-
hydroxycholecalciferol should be given when biochemical changes
start to appear in patients with chronic renal failure (hypocalcaemia,
hyperphosphataemia, a rise in alkaline phosphatase or
hyperparathyroidism) or if the GFR falls below 25 ml/minl1.73m2.
llietary phosphate restriction and phosphate binders (calcium
:ricarbonate) are effective, and aluminium hydroxide use should be
avoided in children. Renal transplantation is the best replacement
therapy in patients with chronic renal failure and associated
complications.

Case 62

1. d Benign paroxysmalvertigo
2, Good and no treatment is required
Benign paroxysmal vertigo
This is a disorder of infants and preschool.children. tt is characterised
by a sudden onset of vertigo, with difficulty in maintaining posture.
The child will lie motionless on the floor, or indicate the need to be
held by a parent. Ataxia and headaches are not associated with these
events. The child will be pale, exhibit nystagmus and be frightened.
These episodes may last a minute or two. The attacks may be
replaced by headache and vomiting as time passes. A family hixory
of migraine is always present. No treatment is required, but
antimigraine treatment can be given, especially if attacks are
associated with headache.
134 100 Grey Cases in Paediatrics for MRCPCH

Case 63 cell-mediated imm une'=


1. g Primary hyperparathyroidisrn lungs, leading to giant-:e
h Primaryhypothyroidism the epithelia of the nasc;
a Fanconi syndrome sites may then be susce:
b Chronic renal failure media and bacterial pne,
f Hypercalciuria affected by the state of nr
2. Distal renalTA medical care. Measles
's
3. a Ammonium chloride loading test countries. lnterstitial pre
d PTH deaths from measles. Or
e TSH/T. days after the rash disa::
b Serum AAs there may be complica:i:
j Urine lrganic acid and AAs disorders.

Distal renalTA Subacute sclerosing pa nr

ln the classic type of distal renal TA, there is an inability to lower Very rarely (seven in 1 0t
urinary pH below 5.8 and an inability to excrete appropriate amounts sclerosing panencephai i:
of acid, even when a loading acid dose is given. lt can be diagnosed initial infection. lt is a pr':
by an ammonium chloride loading dose that fails to lower urinary pH will present with freque.
below 5.8; urinary acid excretion will be reduced. There are many touch - progressive den
causes, which can be idiopathic or familial, including deafness, or is very characteristic, w':
secondary to nephrocalcinosis, hypercalcaemia, renal parenchymal high-voltage slow waves
lesions, and vitamin D intoxication. lt is transient in infants. clinical jerks. This may ::
severely symptom atic. a
It is sporadic, and children can present with anorexia, vomiting, and measles at an early age
FTT. Chronic acidosis may lead to chronic bone disease, as there will the advent of vaccina:' : -
be excessive removal of calcium from bones to be secreted in the the virus in the brain, a.:
urine; this may also lead to renal calculi and polyuria. There will be a virus from sucfr patien:s
low level of potassium as a result of increased distal sodium. expressed; the M prc:: -
Potassium exchange can also cause polyuria. The potassium level cells and the protein ie',,:
can be very low, which may cause cardiac arrhythmias, flaccid lgG, are always high. u, :
paralysis, respiratory difficulties and coma. Sodium bicarbonate will for measles are presg::
correct acidosis completely and growth will return to normal. ln levels, as well as in tne s,
nephrocalcinosis the changes will take time and may not return to duration of the disease s
normal. A potassium supplement is required in cases where there is treatment yet but sup;:i
symptomatic hypokalaemia. important.
Measles is one of the p-e
Case 64 almost abolished since :'
1. c Measles lgM antibodies many countries. Manv -.
2. Giant cell pneumonitis whose parents decicje: -
vaccine for fear of au: s-
Measles has yet been proven b::,
and inflammatory bov.e
Presentation usua ly accompanies fever, respiratory tract symptoms,
I

a runny nose (coryza), cough, and conjunctivitis. Koplik's spots on


mucosal membranes - small (1-3 mm), irregular, bright red spots, Case 65
with a bluish-white speck at centre - may be enormous in number; 1. b Hypoglycae-
red areas may become confluent. A maculopapular rash extends 2. Ketotic hypoglycae- =

from the face to the lower body and mainly limbs. Recovery is 3. Fasting and tesr D c:
usually rapid, but it is important that the patient has an unimpaired cortisol level, Gu a-
Answers 61-70 135

cell-mediated immune response. There is continued growth in the


lungs,.leading-to giant-cell pneumonia. Since the virtis grows in
the epithelia of the nasopharynx, middle ear, and lungslall of these
sites may then be susceptfble to secondary bacterial hfection. otitis
media and bacterial pneumonia are quite common. The outcome is
affected by the state of nourishment of the patient and access to
medical care. Measles is still a major killer in underdeveloped
countrles. lnterstitial pneumonia and pneumonia accounts for 60% of
deaths from measles. one in 1000 cases may get encephalitis a few
days after the rash disappears. Most patienti Jurvive encephalitis but
there may be complications * deafness, seizures, and nnenial
disorders.

Subacute sclerosin g panencephalitis


Very rarely (seven in 1 000 000 cases), the patient may get subacute
panencephalitis (SSpE). This devetops 3-10
9c]9r99i10 iears after
initial infection. lt is a progressive, fatal disease. Affected children
will present with frequent falls, myoclonic jerks - especially on
touch - progressive dementia, and an encepharopat'hic piciure. EEG
is v_ery characteristic, with paroxysmal complexes in the for:m of
high-voltage slow waves recruiting periodicalry at the time of the
clinical jerks. This may be seen even before the chird becomes
severely symptomatic. Risk factors include a history of primary
measles at an early age. The incidence of sspE has decreased since
the advent of vaccination. sspE is associated with defective forms of
the virus in the brain, and so it is difficult to isolate the infectious
virus from such patients. Certain viral proteins are often not
expressed; the M protein is frequently absent. The CSF contains no
cells and the protein level is normal. The immunoglobulins. mainly
lgG, are always high, with an oligoclonal pattern. Specific antibodies
for measles are present in CSF ai well as'blood and very nijn
levels, as well as in the serum. This is a fatal condition "t and tni
duration of the disease is usually less than 12 months. There is no
lleatTenl yet but supportive care for the patients and famity is very
tmportant.
Measles is one of the preventable infectious diseases and has been
almost abolished since the introduction of the measles vaccine in
many countries. Many new cases have been reported in babies
whose parents decided not to give the measles, mumps and rubelra
vaccine for fear of autism and inflammatory bowel diseases. Nlo link
has yet been proven between the triple vaccine (MMR) and autism
and inflammatory bowel disease.

Case 65
1. b Hypoglycaemia
2. Ketotic hypoglycaemia
Fasting and test blood for: ketones, g!ucose, acetylcarnitine,
cortisol level, GH, and very-long-chain fatty acids
[r
I 138 100 Grey Cases in Paediatrics for MRCPCH

increased very slowly after initial administration. Looking for a cause Hereditary sensc
is important for genetic counselling and prognosis. Detailed syndrome)
investigations including muscle and skin biopsies are worthwhile to This syndrome is
help the parents and chiJd. common in Ash<
characterised b1,
and intermedioia
Case ffi myelinated fibre:
is lacking. The
1. a Hyperinflation with hypotonia, s
cr-
h lnterstitial granulation in both lung fields child grows up, c
2. d PCP
tears, skin blotch
3. h CD4/CD8 ratio
BP, cyclic vomiiir
a HIV antibody is due to apnoea
j Bronchial lavage for PCP and impaired gas
condition. Absen
lnterstitial pneumonitis lPneu mocystis carinii pneumonia) diagnostic criterli
Pneumocystis carinii has many features of a fungus. lmmune- on instillation of I

compromised children are most vulnerable to infection with this features in helpir
organism, Cystic fibrosis is another disease that will affect children in supportive, and c
th; first 4 years of life. Children with T-cell defects are more acute crises and r
vulnerable to infection, and this can lead to PCP. The clinical features prognosis is poor
of PCP develop very slowly and consist of dyspnoea on exertion or
tachypnoea at rest, a dry cough, breathlessness and cyanosis. There Case 70
are few auscultatory signs found and the CXR will show diffuse
infiltrates that can resemble a ground glass appearance. The blood 1. c Pin-poinr
gases will show a low oxygenation rate and low COr. Sputum, if a, Renal ar:
found, will contain a characteristic cyst, with special staining (silver I Renal US
methenamine). This ean also be found on the nasopharyngeal g Cranial !
aspirate, bronchial lavage or lung biopsy. Monoclonal antibodies will 2.h Polyarte:
l
help to identify P. carinii. A high dose of co-trimoxazole for a period m Mixed cc
of 14-21days should be given, and if there is no response, i.v. b SLE
pentamidine can be used. Steroids in HIV infeition and PCP show a
l reduction in the mortality rate for these conditions. Children with PCP Polyarteritis nodc
secondary to P. cariniishould be given prophylaxis (co-trimoxazole) This is a necrotis
either daily or three times per week. Other causative factors for PCP arteries. Multior.:r
are Mycoplasma pneumoniae, CMV, and Legionella' the form of angil
varied but usual...
weight loss and :
Case 59 of the disease. Ti-
1. Riley-Day syndrorne will produce syrn:
pathology in sma,
Rett syndrome
Cockayne syndrome
characterised by r
nuclear antiboCies
2. Spinaland brain MRI
be high but the cir
Awake and sleeP EEG
lesions, kidney, c.
ERG, EVP
Repeat skin biopsy
complications s-:
hypertensive re: -
3" Genetic counselling
be helpful, anc ::.
Refer to orthopaedic surgeon
considered.
Review by oPhthalmologist
Answers 6l-70 'l39

Hereditary sensory and autonomic neuropathy


syndrome)
type lll {Riley_Day
This syndrome is transmittpd as an autosomar
recessive - " anci
common in Ashkenazi Jews (chromosome gp31_q33). trait - is
characterised by a loss of neurons in ii-,u port"rior
k i;
root. Lissauer tract
and intermediolaterar grey corumn", ,no
ior. of unmyerinaied and
myelinated fibres in perip'rrerat n"ru". *n"re
a catecrroramine ending
is lacking' The onset of symptoms o""ur.
troo., tr'.,"-tirrt t"*'d"ais of rife
with hypotonia, sucking difficuities, a poo,.
and vomiting. As ihe
child grows up, other fiatures i;;;;; gi*fin "rVrerardation, absence
tears, skin blotchino motor incoordinJtion, of
unstable temperature and
BP, cvclic vomitino]and drooring. D;.;i;;;
infancy and earry chirdhood
is du.e to apnoea #d .hest inteJtiort
s'.oiiosis, posturar hyporension
and impaired gastric motirity rr"
-ajor probiems associated with this
condition' Absence of fungiform pupittr6 i, the tongue i, on"-ot tr..,u
diagnostic criteria. Absent-or oi*T"i!r-,eJ iendon
refrexes and meiosis
on instillation of 2.Syo metacholine nisirminu
in the eyes
in hetping the diagnoiir syndrome. "r"-oif,", is
lgaturel.
"rii"v-oay are etectiveTreatment
supportive, and diazepem and chlorpromazine
in
cri9e9 and hypertension. prenatar JLgnosis possibre ireating
19y1e
prognosis is poor. is and the

Case 7O
1. c Pin-point lesion biopsy
Renal artery angiography
?,
I Renat US
g CranialDW MRt
2. h Polyarteritis nodosa
m Mixed connective_tissue disorders
b SLE

Polyarteritis nodosa

Ilb !t a nec.rotising vascuritis of smail and medium-sized muscurar


arteries. Murtiorgan invorvement ir .i.o.i.ted with this condition in
the.form of angiographic features ii"n"r.v"*s.
varied but usuailv are associated with Symptoms are
tere, on and off, irritabirity,
weight loss and lethargy. Various ur,ln r""ir",
arise in different stages
of..the disease. The attEiitis irror"i"g
ini-tir*r, kidneys, heart and Grr
will produce symptoms and signs ,"i"i"O
pathology in small- and medirim_.ir"O-*ur"ular io the affected organ. The
characterise.d by polymorphonuclear infiltratlon
arteries is
nuclear antibodies are h.igh in the and necrosis. Anti_
malority of patient.. inu-e-sn'*itt
but the diagnosis can be mad'e by biopsy from
l.-,li_gh
lesrons, the skin
kidney, or liver. The prognosis is'usually poor,
complications such as rena.l iailu"re, nypertension, with-many
h ypertensive reri noparhy,
strokes,
iver t"i ir r"' J,ii'c rr probrems. ste roids ca n
r

be helpful, and other opiions fo, immunotherapy


considered. should be
t', i E=36H#EcgS?3q5a
+o O.o tro,
€*t
JJ
O-o qo € .,g 3 E: :'[Fg
' t r;i
flsggggigrqq*fg
6-9 - O o o X = *=.f _a ag
o
1+{g11ls.Ed'* #+ dE €a
=E3Aor_bi_0.9.=aE 6'
i

l
gB5'$9aa".
io
a-
i
o
9
o
o
d IIliiiiiliEliE
As
ot
tJ
f,o 3e266r i qq6Ii666 *;ii:tF frt#E iliE
q
o
PBB
ooo
r;;3ti3a
q aasgi$3aie[;q3i
o
o, ==
i.o I
o p.
o
c*
'\,
:,
(o 4eiaEil?[i*i*sE
i1*ffifrltlffiiggE
5
N)
5(O
o
>r o
6)
EX
J
o
6
AJ
.<i o
o,
tt
a+ o
o
6q
:5

[E iEEEJ
"
do ='
1l
o)
o

i1 E,"
.5
o-
'frg or'

C)

flfigigfH o

ii
tl
is
rg*gigili
o

:D
C)
T
C)
-
P_ c"
=€ *g
*e
*= ggt
;E r li i l
!C
E{
:1
1
[l ffi lig [i;*igrg

r. Er, rl. +... -,: i+Hrri=r6- iil iE*fi [+$iiif iSi


i1.ii*'iu#*a1;grgi*ri,i,
::=-x:d.2
a-= - = - d 1,
=-=:aro(or
::i<=a9l{i
=.,
a H3-,,iE
i:1 1:1*[ ;.aEu$g'3= $ * Ei-;!lq Eili$;iiii$iii
ii';l**'rr*=salEli[gi;[ilitigili
i
g=
:
* rg [*ia{[i;ii;e5gAi
:, d

s I Hi'r$]ii[Ei$$}E[ o
o
o
a.

*[-egili;$iEi*i$
o
f
<n
\J
I
@
o
e #63.'*i Pil o) 5
(^)
i
=i?cn* xqsEB36
e<:
cto,
(D f.f
4s 3
i =
+'=i[[ffiEillgregIgElf[
[ ;;;;;;;;;; 5
5
o
c)
c,
o
egE o
(I)
{n
o:r- o
a
lP.
"iBtfE6E*i;
o *c f'
T
o,

uiiieti*gil1ii[*ii$lilliflE*iiiiEg;iE*
o
o
6'
o
=.
a
o
3:-'l AS:q 6:,6' j 9r'o3:'i,:-u^ *l=11 ioI{i a
o
t; -0
o
-
Hq
{i
;:
g:
8;
il1
lg[Eill*iliriiiEliEiEii;Ellii
6s
o(I
c-@a

I,
zE tr,>Ttpo>D:(C Q>=
!=-?S?PgEt&5 S' r'r
a-9-]E[=ri:i].5.t,r
: i1 ^13E lE--;gf !,. r(o +., oo cro € r(o +o o-o o'€ (- H_ g.a T
J o tsc
=;ed
oO
o
o+
U'u
c
g
:,

:iliEili[ii ilglaflisli[,*tEagi1'i
iiiiiac$tal a#; ;F EE t
?1ftH#iBHgE BSil * =a=

;:=!i;ae*Es q fr:)a tt
B 3.3'33=as=s
:iisi[EflAs; [i 'sn 4 i
6i
E

;:r;l*[;ire E
ita$erH+:ga g
5

o
c
o
(n

H + :r.
o
& f

=nil;;Eg*6ei
iqeqgr**:g}
ial?+Ya'ia;a
?_*=o.Bilg;+itu$
a
Jc
l
E
g
o
{
@
I

1-tr"g'*i= \'
5
(r!
C:':' C)>O
i.9 9 aE-
F+-l grg;rBHT 5
O)
o,
=+ 6'- ir6='gq=.!,- o
le_
o o, o-ortrj'.o
-'{-.s:o u)
J
a
o
o = ^. =.o)
-'f o o
.. ,a u ) J e. ) c)
L -*o
-.
o
Ut
o d 3e
= =ila o
o
o iIgarli
=orO(/}=_u,

--i.:!la o 9
o)
o
o
Ut
. q jo, i3, 5'
T'
: I2*:+$- o,
o
so,
JeSi
Io?orEAo, =54 o
=.

;iE i[]a
@

o
7ti=Nror5 1*A
- JOi ='u (D +J
LJ
6 6) I P i.J a o =f ^. (, =.i !9. 1
=
3J: a []3 i;3i o
il1EiEaif$Er**[[aisgg,gE
gEi=4;3saBq+;ne?i* E t s .
or C F<f

o,
:,
=)
= ['s=l;Egg
3 j
e-: E
T
o
-
o- ==
)
:l
a- Filti**
a
=
6'
h a$i i +++O_7i

iiiiiiiger;geiE*fli=r
5
e g
N
s
o
=

sssis:fii#iiii$
s==o;r:tYt f; gg3
t:q=oPjci
, -'=(o +o Q.o Ito, +o cL6 Ero
3"
==I= E3;
F
t= O < ! €-@ €
^ -r.Y fJ
t5=r.8.9.19
gr i
E<-!rrr--ico>cr-r-r6
qgodo6 ! a8+
-9{d;?r^9f;
EEa tq O:!.
!. rU 6:.o
-P3:€A:+l ? 9q - :
I EFsg$ggg*is'F***;a s
€ oro9
3iI
iii;5iii; flriar;g"t3=3=
A q3d.Eafi -or 6rirEt
oci o d
fQo
JJ
Bqe
o(o,

1. HHg s-E,g # 8-stsi EgrE


=V

'iv 88.
iiiiFiHn$gigEfarxs 9.s'
l!-l=e{6J-
t,
9s
=-
IF
:t
o J)a
.(.(
E';
O I-
n

q,r _
gs, _.(D
E

iiiiirirEggiiiiffii ;q
'o7'lt, a
coo
==a
(r-5
i; 8. o,
='
;n
I
id
!r,
TO
ei
a '\,
SE+o
6'H o
L

I b;'F
feE
o
o
*.
o

iiiiirF$iiiiiiiifli
s ujisil
ld3
(Dli
oaa
IJ
-org,
E:E
o
=
{
&
o
ag-F$ 5eF c<g
3jv, {5
s
@

J
o
o
6)
o
(.)
q,
(t
o
o

T
o)

3 aXi3a
o-

g 6n-'+"4:;3t; 6'

E g$ig ,6rq;s*il
=.
c)
Q
o

u
o

liIiriigiirigraggirumi
-E
r)
I

iia-*ririg i;i
f +; *r qi;383a
fg13f*g;i4
oo6 6' -' o rdL =* I
E

1.,

J:(e +O e.r) Oo +o
* €J 3 -r-.-'f,(o o-c,

'; r*aiii$$ ggdrsEesir r


J

[4Fif$a58$g
* $Fi iiigi${iig*f dl.la9A6+o;
EiBP9+fr*fH
E;i Ifgg$r ?fl;f,FIffiiis
o ai :
: o , (pr, = es o;
=c-d ;; E dE
g
d
E HTE
d 6-i?
lg
r'-'a*?
ooon,
€ *
F 1-H
orYl/,
o6:i
to€66'
='qYf
';iiii#i'iirFiiiiii$ ai
o
o
c.
o
co
'\,
.io
o
5

ii$i$gi$Ei$igiiIii E
tr
cr
o
o
-J.
a
o
c
o
an
:-+.
:l 5 o
f
o ai
!

figliifis{lfgiif[g*
-e;s:n fg"d ff.A :
L
I
@
o
I s
(o
5U)N) j (rl
(,
=Eit F3el*$ti ie*fi: iEa*: i I €€€
J==
o, ='o)
_.J

J
6'
J
o
o
6)
+;aa;HgliEi$qi;3i=.7 ca
+3
+Y^
6 {
o
o
asa*: olu
rOO
=+O
o
(,
f,
o
0)
a
ci 79. o o
a
QXo-
!EEi[liIBifllillliilii J*O
oo*
@oa
g6e
o,
ae
(n
3
9r
6'
o
o,
j'
-u
o,
o
o
or'
'" aa o -.
a
grE r r;
E*lgg.[a+ai?
*
-(: o-
E,g s.
o
o

i
9.
gf [f
=3eimfi = =?E H #f
a(D
3.9
(,
6'
o

o :t. o
:, f,
a(D
ii o- J
o
-u
o
lIe*itiB[[EIl*P+ss -.L
3 '\.'
@
x:D
C)
I

o
g
o)

f
o
Eil+1Eii[ggEi[IgEI U)
6',
.\,

[EIl '.fr *B#;*'=;g


ilp*-

iE3l5 e ,o t g r(o+oo.o.',€-+oo...,,o, i53*.6p.8*$trsfi$5s


@N .,;
*Ff EE q
q
iiii$iin:rFf{eniisi,$*flf$Hui
:1iii g ;
=li3iI S
;;=;.: +H $Ai q zzz f :::!rJ
O o o -N(d.rN
vzz^z
Nro o Jo
{
e

fig$e'l-' lE $e !
f w o(o(o cr(o (p
=(o --(,
,a oo, j' oro
=aA'tul='o H$aEE d 4
[-{? o o(D (D

=ir-r:E
:igri + -i*
Hq'E g ;E= iil
H
5+:t;:
fgdoo 6' I q e-
?
a
:ie.,'+ E A X: ?

i;Efl[
:1qii
33T?=
E:
g+
o '\,
gF
:s
o s a
o
i1i:*3
-c" l-.or
3
6' O-
=.
8.
O
{
o
F
?" €r c ah
:jfx:
;;3 e e- A
e
\l

S.co()=.i
:=d X
I
6
o
=5ae (,
L

t
: #af : B;:
ggigg*t a
rilis ; C

Eal*a[E[.{i*Ei
;iiE r3e
x*xa5gg
e
:;
Frg i
:
a

sE
jfiE 1 i

$iferB*
:EcrEis
iee *ia e

ieEESir goffai-a
3:qsa.6
5*6S.?' o i65 ., __
I

t$J

o ro-o q
x ;BfF:lg[;rra=;i
o36)c s++ig
o Uro,
ii- o i: =. i$g$i;f[1 3
$s; r c EgrigF$3iniiFi
='o,
-oa=' ()
ed
+A
oc
acr
to9
o,
ia
ro
i$ ;E$ig;$Fgii iii
;en*=igrfr;ilii
d3
f.
a
=
o,
flE$lies*igrii
ii
f
U'
(D

o,
a
o
6 [*ggi3B.}iE1;E
C
J E:FgBqi;.q-qe;r; :
;;;r3i-i[[I;=l :
I
I
T
I
T
I
I
I
I
I
I
li"):"n
l0J SeJrqqsEt
l(a ul
It'\ arorcr o
| {r o-e}
<f:r
{<o
tl, (o 1:l
gigiriiilil$iiirgiiiiigrurigigggigrig-* O
tJ)

a=a
=4Qa (f
N:c .^
j, -1
-:*. .. o
;-.
tftO a
ryo =
^(o -u
tJ +
O

; =
o-
r) gggiirliFiirgirrui*iuirriiirr[grli a
c

-if$iEgiiiiiiiiiiiii#iiii a)
a
r

$?f3iiF$i
FF
d";
s aI'*.,:!
3f 5i ifi i
a; 3E E
;5=e
=s
::=?i:s:r: =
= = =C -=o'.t$efl s
___2.J:y.OJ,
-
E:j39hqEl
I

=--!-:ioi(
::a-==rX(
: l: i- i'r Y co:.j: F
X i;rii
;f ,---<5 ' EgigIf,iiiiF$iFffi$$siiEr
i+=6;'9or^-:a-
='--'Oo;
f ;;
.i,iiiE +iigi =
-l
'u;$3
g
Fii$f$$E[s$$$$gif,$f,$ii[ F=
=.iii3fglifis
i:.iHr;rc$sii e EEgifi$i,iiii;i5gErflEiii gqiE1r*659;sig#a3f,fi$g'f*
Eqi;pq3fr9€i€$*HFf+$ir*
='riF3r$;BBfi [giinEFeiH$tEiflgEFi5ii
:iiisi*5+figF
ItisE i!iE:i:? fri$3frrifrFiit;gfr{i
ii=issi;is;s $g*Ef
f$ig$$rFa+r,=aia;'a.;i$in
ro,sEi3EIsa='ErS+9B5 39 t
:,
6

=;iliifi3:53i
j .i i$fsE f5H
- ,;*-o_sfi33
=r fi 'E
igisgFli;;*F$iFi
=13593:3E6i.F;335.fi
$s
*l
o
o
!

&
o
(rl
(,I
o-oEf c) o{Eo-Jo-o ? 1., :'
[ 3€E€iflE?TsHfissigS
==n=sa
:=;iCsEqB;9 aB.F=Ef$ (rr
o,

1i iEi,E rg tfg Egi-"€ iE


H
tD o_(o fJr -o-+o o
cr $:HjstsEiiit3$EEaf a
€:iO e die= H s=fld fg; E
t OCt(.)r6l:lCO:I o
o
a
=
i€{*EHiri
- =.!? L0
{=o- rlEiiiI d=i iqE;r
o
o
iiEiiiEFliH $81il$iE V4@l--l^
E agrgllq==E
(D

*qE$qiiilgFBI[}Eii
$E (Il
a
o
o
i-i;iFEiEssE 33E:i=I s rfr+r$aes :]
EdES e.; 6 rilSsi; q
!
siliFgarggiiE;is:a :-g€ fl E rFB f +€ di
o
o
o-

EE
Or'
a-ft B
:,
-[ E
=
=[,Eg o

,iiiiiiiiiiiiiiii
,ld
:J;iHFiid-s giltE3+s
d
s
fr,H;EgiEsqigs't*EtB an

o
OrO
6- a
o
gHit$*} o 1'
ilifliiqEe c,
-
liE:EgsEfiq i1fiiliq
i3E'Ir=s!gI
!;; jfs sSiiE FSEJIfl.",
qE q q*
=
=iBr='f$'.;IE;$gE!
=;
n =;',' i;nlE3riiHFgisEt
!3*'
;: Ei
6'
E se ; d*I g*46

€- 3 o
Or
v,
+o o.o c'o, o
6
o
{'=g=?F
-:o
a;fi-rq
or U= o (tra
E

=?
o :: iii
sE;e!E
='
X. 5'e<
o- a
B. qB
P
=o,
3 gQ:+ -U

:-.
:: o o-oo
our-
D
E"AH
xo
:< *
i-
(o
;.J
>n 9
f
o
.C 5'
:,
an
-,n
-a
=.
=
o
{o
5 o
it o,
f
q {
:- U
@
I

=9 o
o
o
- a
Eiiliili*la[*,iEilg;g;a
(o
gt
{
(r|
@

J
o
o
o
a
o
o
o,
an
o
o
-o
G'
o
s
o,
:,
()
o
o

a)
c,
1'
o
.I

--
ii+i'##iffiryTii-g'5$ffiffi
=: :i ag
g5 $5
Do

5 r
E <€
i3
.
-= ;E d 5$
g-E a
; ''j €*5 e
@<- 5
3 iq d
E

grgiifif$Ffi
8g g :i*$gi$$6
fgi$E.i;ri$1giE{€*,

ig.ruf*igii##i
"Ei
i*ff,igf;*E3"gi*$$f,
o,
o
oz
rs6;.pqJ
H A.q
a
o
tC)
(Dl
6 o
o

o
(l
o

=
r NJO O)
i--i,o- -J o
I =
-)1) JTA
3= :
Ie-g=
I
uro
oo
.I
C
l
33
NS
JG)
=f
i3
C)0

"EEFlS',eF"r fs395i9+gi$$gE,Fgffl
+o
F'
o'sr(o +o
i-
oi
I

& o'€
I

5(o oo
q; +; a SiEi { r3ii; 6
6)
o.o
$ a. o,
:, :,
o @
E
F-JOG)J(,t(JtAs qa
:ssSeTyyE o,
o
o,- 1 l33i=
d dssed rrg#gi*girgf#ffi
ls
c d =x mgi#rriryrruis-i
i.s*qqa3q'* 3d.I
? q33: g€
igdH
* o
::i.
o
ai 9: =d.*I i *o.a"^-.
=e
9,6 3 3o=9!.5g' ^ rc E
HAgguri:*=s=iiie++ iSgSB EE9qg E*3; E
A5
N)o

;q as
o
iEis=P'dq:53d.9q-id 5 + o

$;l:iest5ry#[i+*l
R
8;e-=-i**d$3$-1q!? 3
3E 3.,6 ss.
n
F i
I n
E

o
o
o!

Eii#ifuTf#E$ifl
-3
o
="
f
ID
@

gf 3gii+5 *s.5,$s a I
t0
o
(6,, q 5 r
C'}
J
o,

* - r(o *o o.o,ro,:' -., .,': g$


N

EE; g:1:f i1{ii [" :,(o +o o.o


": StFu o
o
6)
o
o
Cu
q
o
o

igiagEIiaIEii
*i[?t*i[[ta*a Etiffia[g*iigggggE i,g =
!

i+Eglri[ii]ffi 'E i i:
E
gelliiil;a*iii['[[1E$EliEI

?3 ri?sii; ii
iE [g--rirEg
g
a
c

ra[fllggsliEiia6**i-
sif[ rgrii]r
U'JA (D
'
*[ *

t*

& :'- +- - - -';


J o o_o' o_@ o co a)5i9
=.f)
- =c +-. - ('.,;
f,gi5$$3E.iil?f if 3ilrHgil1Ii:

Eilflflii3E[E
g E-.* i s*iigli5iiiEE
,ggriiryiiil
ggliE1$Eii$i
o

* g
5

o
6i i;;,;^;__._;i;;;;;;;;-':'tifl,= $ [8ffiflfif$!
i= r€83,*g$$rr;siei'ri*gi;ggif**$iii$$$i o
o

: E ilglg
o
o
o
qoE,* o
o

H? it3=
!lI) J*'
= 5'
1'
o,
o
=i
:i
-
? g
(u
:.

vrf $g$ffiiii
'-'= e o

::i e B;
o

f
o

ii 5 I Ez zzza o
3
f,
SifrlEiii o

ii
T
ir i o
-
1;
i *i$fis+
*EiE i[I3ilti
iE $_ga'iri[;
ii1; €
r$
q$ *['[3fi
E

!,
-. c.- 7o €E
JTN J

-=oa-@96.9 =
-.:ed )
vi
===
.1
=a;Pid
=^i "
--l f
-:'==v=:
-.'7.
_=^>;^-aF
o,

.,-^>u
5
*JS
: *J<- <o i
'- = Ii o)

' 3 iH J.
arr='o
3 or
)) =J
iJv

^.9

ad9 O'-
rj
ilo
(Da
+ \,
cX
-a
oco
t4= o
6o
'v{ o
o
o 6'
*:, tn
= \,
=
o &
o
x
o,
(,l
o
G

cc
c
o
co)
o
(D
(,

-t

l€€lE $
o)
o

;tr-p;i Io

E 3fl ;: +i$E fr;


riq-ufirrili$ligiifgfiiE$lig
c)
a
o

6
g je.
too o,
a
a
o
T
(-)
r

EFg
oo irirIErii+*Eiiiiiig$
er"q*$;g*
;il
ts ijgi-5[fl *

o o-o Ero,
u
9Ln999
s* o o o iuiffiii[$figig$$flii
-a:d66
: d o-o_o-
O-+d-
--o
dFile
f q, : [s+?ia+8sr;;
g d.PE Ei5i F
3 sE.z3i.riq9 aQ-i=

a! 6I6
-9--J

e **r*a E ig[riiBi$gi;i$$f}
****i;iarn
i if i5f6ii
{ii*grulrffiiiii o
C
o
o
o'
-
U'
@

ra=a '3
d-
q A€ t5E..
iH=srf=i fr$f];
=';'8; o
I
(o

O)
(o
='-:

--

; f- i$firis$fl$$$$;iF$EinE$ s (o +o oo go
xJ '(o +o qo (to,
o €rto +

imflflrgrm*#gii;*
iru** F*ig#.'fggsgF$$e$EF*
ci
€- E;;
or or or i:t

e O- -
a;iii€<o
dg:i-"H{ o

o, O O 2 a'6
I
i
I

4
mi
(Ll-> !

'u=< F--9.-.r:'
gg; -
',3 t=il :

s ryi*#Tffdryiggigrir
o:$c
-U(,)
E"
a
*a
o)J
f 5f Ir;sq$i33i:s sj {3ss
C

-. T'
JO
6-p.
;rffiilrirrgag*;$i3$e
r$ H s n;":igFl?f
o=
o:l
=i.
Jc)
-J
O

?;
u;..3t=F -<6;*;tg3g;gpu
il s;a

(,N

-
$>1.,
DiO -o o_(o g:rc) o) r(o +o o.o (rr€r(cr +.oo-oor€
--'Z>OX>c)@@Tr
&
{mf 2
q; FIgg B
":
=:.=
d or:i e-o- ssse*3ig$i$,=ft}*i
--!-+
4.^
^ -
:=-lC:r=)
.:
_
AJ
v 4
V

-J
J:
I q3;
o<<
= 1i =.H
r-:':5='O
: !+. o, J
O-a Q-
---Ui
-
-A
U
,6-d
gfi;E fi"qqEgs Hrg
<_.J-J

:--^ p
-G,^
r.-(D
;'o fi 3 *3*g sgu r
-0)
JU)
;3
L O C Or(O l--
;dEr
3affq i
&
()

:=' '{-oo
Z -c t
'\'
: 6-
*
= o. ji, ,:i
^r
B ,rl.;.,;ri
l,t; i';.
:c "
!;.:-!! f
ah

a o
a
?
o
@
= I
@
o
{
N
f,E lfi$geF€t#
zo<a s
s=
+=o-o, o !
N
o
ti$+ii*rg i9 g :/ c ='o 9I i6- o
o
; #ifi lig l3Fg s 3=:is s iri lf,
i#ii#$i$figig$gfliFFgiiEiff E=
o
ET
6)
@

='.>=6)
o(/,o o
n$iiigr $fir; $ E{ ) gi giF
o
o O.t
o

E sg r$erg g
iir
oJ
6
,Eo
qo.
Fgf, iii#j iFliru fl$ ff # i
o
o
c
,i
o
o
o
o

='
-D
o,
o
: 5 o
$Ei6fg*siig
o,
o ' giif $ii$f$ o 6'
3
6' $.fi#'$ flgr5Fffl _-.
o
o

sfiig$;Fgi; o
1
3
iHiig#fg-8" girg '€$ :o

::li-q;r*;gf ii # fl# ig#rg #gi# lg


C)
1l
C,
:E

$$ isl riF$#5
BsE n.Ess3s$
3 gJ--o=.t:-
;=3 + =r
fo='|h
o.(6
19,
#fg#Fiiryffi#ffi$i$#iff
ll
-)- o
=d=

!, !.) j o
0t
tn
-'o o- (r(o o
s
iiEgiiiriiiiii
(^)
@??a@
o: d a c.6-
.<:r5c x
V*dNA

: iimE
{x sEE
fa=Xs,
5'9 ! 5 (D

^:t>o o-
v.( Ol
g 6 d'.2
rri
rq 5€
q

iiiiiiiieligr gI (DD
; g rB
po o
i9
€d
t
3.

'iiiEiiilgiii
JO)
I
o,
o
o f
at
6-
o
{o
a
o ?n
@
I
(o
o
!
(,
FJJ o
0r {5
J

ur
+ or 0, (I*.o" o o
6
\ o
o
@|u)x<r>t
u*ot g;i
Ho+E:3 6)
I
o
o +J9 f,
or O I+.J C)

3 gg;B
o,
at
o)+o=* o
an
J q qo c f
q b9;' 1'
o)(u(rtl-
= 9b9 o,
@_ o
s
go) HA
e
o)
a.
a,
(].s o
(DT
o
J6;
o,
C)
a
C)
c. -o
:, CI
(D
-

i.--9d

-i-*liEi[iiiilliii
1=lE rilI1g
gqlii$;i;**fiil
ffiffiij

liigii[,Iliiae'*'';iElliggrggtlglillag
illlale'-Bue'*€i'rE1llEigraiigE a
{

ali}lgii'*1[g'1ii*EElI[i1i1
o
CD

!
=1rA'
UJ IQJ

:'s0,
{
J

g)
3iE55gs$s$f$$ Fr$r$ H o qto o o.c,
sg$iig$ni$Ei$gF[ii J

3r3,E:s$q*F$ds156;a g o-rro+zmFC)rf o
o
q< J o < q o.*H O ? .i ro Q = *d o- < :grFF=Y^g o
O*'o vo"' 5 a
o
a';n*$frrs*a;riirtfrg SEdeo
* s66Eq B
q o
E; a89
iiiiiE
o,
2
+.r: (D,D-E
o
itg*i;.fE-EE*3HfHg€ 5 = o"t'<
d
9-.
Frf;e+trtis$ o
a
5'
3- or 4 .r,
1'
+f,
oi
o,
o
;3EiFrEr$*i[${$fiEr sd
6'
-1.
c)
o
iiifsilflc$Eriff5B$r
i; =7' o-o s f, r ri 58fi C3.{.g< ilirrlfimiaiiigi o

7
r)
l,
o
i i igr5rg$rjffln rss# I

"-i: ir Esr #f {H ** *a frg


;ilFrtiit$rasg$iis
3$.gEliff*"
iigryrrgggirg
g;-H 3 B flrfgf
=
o =a:-r, -<
trir
il Ea 5i E =

nF

i= [ ci [F+i 3rsEif;E3i6Fr #
o (o
7rc)1
+:r Is itqFr€ri+EitqsfsfiI
s B <P
i :1i[E 33 =E
16
E_
i#[aEEi*{aE r}i[$g
=s '"'='.? : $$gii$g1riE$$ r og
:
=-

:
:=
?
1: i rAqgqiilFii;E a[r
aqfiiElr;ai*:gg1$t{
$5
fo
o(D
(Do
='o
O=

i.E*gflliBgIii;g1gI
?:- =a 3
it e:rr*EBfE$+i{3P<:ra

irriif'ss+*+iEsiit
i'EgfliEEgBig$ilEiii 3ilgxgr: iigqI+[EEF

:l

*fl4i*g$E3aIa5+i;E
q r 3i 3; iiH 3iie a?s.t
EgIEiii$iBgaiEr o

o
o
@

t
*E
f1{=f+;iriE
m; isiaili*Ti*ilig5l
) @ 6qg iqd g.g=;
o
I
(o

[ !a = ;*fq*e;
^r o_ j g[ {
\,1
S, l'r:. p
Uo JO +O-=tr
6'p
I
l'
G 14F=ElEg$iii$s;g#f
u.*;1Hq39;
FFqS?388sero .^!

rltrlno>+oI O
6'a'6'5Dfdo o
o
f,E 8E X
:l.o=1"
o9oJ
$ o aO:iJrX o
e,o qE. t Eo='
- J a,n
o
a.=.L'a f *
gle
o
o
3 o'iiJ
= Ul

d;Crr qSoi
r il
5'

:i;
; AE = E; =
AE
-o
o,
o
g

i;
o,
f,qJLAf,
J-av) a.

fi:
*)?
x:!o g
i igrE 3 ir$
r)
!_; df sB EIs; a
o_ a
I.< 3
,,I. o, 5E o
o
:
'fiig3is$ffiqfiif .ffrlffig
S$sgE;;ir€r
o a
o
o c)
3 1l
a c)
(cI T
J

G
o-
o ag$ffiiiiifigiii

;i;ii+gigigri;igliig
#i;s#*iEis$$Erifl:Fi
:Hl:E+€Io=i;5Hig;;3=
5s *$iai;c;$Hffi*igri

,
at
{o
o
iigiiiii$fiEi$i5iEF @
(o
I

o
E+uFf
o $=* o-g iia
9Fg ": * g" ; \,
(o
-
r $E q.- is rypFlEsF 3as
*o o-- or
5 ;i i*s,i
E-gj:iF*ss; Fi3;
u
i

:: c trri$
iry,{$i f ' #'E ir#ruT#ii$ig{*g'rii$if
+c r
,
ia* E-iE * F
n$5iflfifi i+i$i'iii9*i*$g f
=uf:ESg
u;*;.d
036
'6 a E a
f,Jf,
o $g
a,6-
"
aissliga#fii:iii;pesiiHEr
EqE'+fiigagigdisiailras;
gxi$9=*$r,'s***ls.[ia#. i35
3E=B:::#e;e;XSf,,*1iid?3;s;.i
? 'flfg$fFfi *iiiifl$$fiii#$i
F =.='.*_i.; ?
:-= ;5ie-$g 3$.ig
5fgf ii l;ii iiir$+;r;;sa;=
r
? ;
i==u $ga$$iFi $$$iiFfFg$i$ii
fi' SqiEflii
$ #
d;
r I i.d;r+:X==.
sgflf
9**d
+-3}
Ei::gn*5[;:rjl
.;6di9;i'il{98:oo,
3iii3s3tgEf,=g 3
6. =€;.a'6,88.-=5'
!, @
N)

,(o +(D o.o cro €(o +o o't' C''s(o o


o
o
o
E$su'?*ii1flgiflE$i$ ()
O)
v,

lEair qgi
o
an

=
a
*3:e* '$E o,

a;gq i o
o
6'
=.
6-il o-d g 6p-
C)
a,

Eg s3 d. &i o

trfi Eo'
i.3 a
a)
-{o 1'
o
8e.
O'E
-
) .\,
q
6-
E

3 EE*
odg.
d
o-

;
f
9.
]
d'
g
E
ligigiiliiila
e.
g: BiIa.Hs 3 qilfti fl o
t
€: - *o o-o cro
=@
9;'por{c,oc E
c

rIiggi$$gB$gEE I G

i i: l;aE
o
f+ c
ase:Bag3slil
=r$fl
.,3fl ag o
o
o
o

i$i*gi$Fi*iE *+ *E
$3-.6'6',
vt ?'-- F; i' o
o
=
T

#:a ?3
o,

H; e
o,

Srdiii l$i$s i S.il 6'13


3r =i6 cq .t,=,
d,'@delF3.7:i't
rsl{s 1g[re]; :-r i a
o

a
6 6..19 g'd>:3 ilP it
=- (D

)rao,
(Da o
T
o)n C)
ug I
Fg
eaiili+;rir* Jr
or{
x:, o)

ii{=#-t
$$lffuiflifl is
F rfti

sEr533'
$Br f

s=H,tldrs
ql 5 *')J

e3
-S
*E
iai
o
o
o
=
o
at
c
o
o
:,
-r,
(o
o
G
= @
3.
o,
3 o
o. =
CD
E (o
o,
o
) I

o
:, o
(cI
:t
(,
J
@
o)
iF*'-'J(o +o o-o (].o or*'-'!:t(o +o o-o cro,
c*f €
o
J A(n(n <a t;) -r, u, >- < c/, >- I o
6)
o
o
ii i i
o,
tt
o
o
5'

iilEiifigiiiiir,i iii
't,
o,
o
q
r
iig'iiilii'rirrgi,,
o,
f.
o
lgg a
+ d+ Pciie.
o

;i IflEH'i=;'F
*(o
q9 a
e o

a
| - ='o
:a o, r)
*
1 d
fr i;"' =
1'
o
-
iiii$iiiiiiiiiiiiiii E.E9
o='

fiq
='

iiir=rilBiBgBB*ig' e
o,

; E;EOi? a
o.

I i;.
i od
'Ht*
A

=o
;g
5i
3
[ 999
=s=
6'9a,'
B
i.'
3=---
:-
3{3
Y(DJ
oE
3-= o9o
q-l,g
:1
4-a
E3f
g=<
e5 rEl
si
E
ila's1s$iie$$iaE[iEijFs
i
j= iiiEi igi;
rTco fff3 ig?sE +ii.u
a

ii
fi giiiii$tlrEilliilrE
e3 n
o
=uEgE 4iBa[iifl c
ffiri,* *gEEEffia*:ga;
o
o
d.
o
)
o
(o
I

<)
o
i" !=-=[fi; *$A*' filE'il {
@
@
@

J
o
o
6)

}3lBllBlFEIifilE$iisiE*a1 o

iii ffii}[[[IiEE[EiiiIli o
o,
UI
o
a
5'
-o
o)
o

lE*EfliIFIlliFiilillliiiEi[Ig[r}i
o-
6'
l.
o
o
o

v
r)
1'
o
-

o
c
..
o
g:
(g

o
o
@
t,
i! i'
cE
(o or(o o-t o(o +.eo-.,cro,€=.+ooocr o,< 839.PI
[{ ?
m :t 5 8dd<o J

PfaJ]_qlr d,
(o *5,-SqJgR<-oaao169 a3a:l cC
ii$gr}3E}[g$$ fo(ti'I-<l+ o-oi r e EFl'x*g-<5q 9*q qq' At* dg 6
ii i.-=EBi i E ;q$ g3 f$ 6fi -d'o'oi'r3 ,sq''o'
o
3=g i
o,<5r:
.= 6 s.q
E c
o,
xd3
iii$ii
a,
:.8 f o L=! o

i+giggagisggg €g:P:E + *EEI $r1F*


*i?s3=dil srgI tn
5'
"i 5 6'9
(' :,
6"
9-O, gE*uSg raf a
!
O)

-;
=' o
1$gilE3$g:EgH s g,d-(o
B,a"
.<(D(D O-

;g
E frH= E? i;s.;
o-
o'

r l;!ri qqiA
9
;; i-q; iC-."==gE o*:
ts6:t^
--
3 rgE[
6'
a
o
e.{ E,
tigia*fi+dB+ )a
a
X JE=
r9 H 3se.E;
o J-X * ro: ='=o o o:l
(,
H
I -9oor{
.as:
f,,
C)
1'
o

iiitii;li+g3 H
q
f
Edq3
d;-dg
o
-
o- f='o'
:orf.
- di
B gH
=3 *
Io,o6-
riEga**rg+-e
r30-
;a
-o,
+

lEii*g
=;-
,--lI Biltt$ii$if$ o o.

o o, =Or
'Tr tD
-'o,
c)
rtEgt$l*iil
3 =.:Y {o
=c
d3 3;
aE E
oQr{o =
J
o
J
I=OI -o-
3;38 nEEili[$rl
q ae€
]J=
oo -
-;
ie=-lidgE
<€
E-L
:,
trT *EEiq- EE r
oi ='
HE E
(oo :ro a+a;iEfrAA
Eadi6Bfia
EEgirqi$B J
E F; *=tg[E
o

gsrs
iEi$iiEEFEigliil
o
I
B a
@
i3eT
i
*
i;in*-llfgigaggl
I
J
o
3EH E#* o
o
aE i. I' (o
NJ o
o (o
ta N
o to o o, cr o
(o
s o
cl
srPf,ts=
-.9.6- { +i5 oI

89q ET < L i
o
ia!, -!! r)
g:
--
'<a?--' o,
llt
H o
o
Qab O3a
=! =
1'
<3
to ^r
-a < =(J o,

s-oB;
o
g
o
1I
:.
o
a
q
o
c.
)
c
f,,
-r.
o, ()
1'
o
r

NJ

Dq.

3€
GtD
r sil;+ilar+rFatr+e*rnsa+nn++ r
Eg)
:5
lo
rd
D=
ryiiiaffirgggir;
=I
gE
=->
I
o=.
I Cr=
=,o
' Eco
J
q
I
o
3
o

l
tn
{
o

iB ii [iiigi iif iiF$g $iEs HFe$iiigffg


i-,i [.d5g+g-'f
$ i f,
ii Ei ii F ri
a
(o

iig'flA
I

P E$,-;e35+'*:3 9a'5*u.,*13 g;agii.f


E o
o
(o
(,
J
@
6-l ! NJ 5
o
=:t
x-.cl o(, o
o
'o'g o
En E
(,)qJ a
oc) o

€ai o ;o o
o,
o
r=. o o
o
-:o o
5'
oor 3. 1'
o,
gr'( o,
o
eo, *o,
HE.
;E
io_
I
o
o
oO o
a
cDf
;9 ,
38 c)
T
o=' C)
YO I
t<
<q
o

iEesl$BIa i1;i[*i[a;E
<o
o
o='
*in; eiai *i
,iiu$[a'ffiliEEgE s
io
orO
=-a
ro
o
i
Ei*BB
o
o.

*B*lEiEE

iiilri5ligiiii Eg5ffiigiggii
; r$ "1d
i*ii,rli
igrigtifiigfiifii'$fllFi
'g;i

f
o
{
o
o
(o
I

o
o
(O
(,
@
o)

o
o

****qgEEElililli* E
*iaiiiiEii l i*i,
o
o
C)
o,
o
o
a
3'
-t

iiEigl,
o)

'*** o
g
o)
f.
r)
g)

I
=;iE[[1!
llfilflifigfiggggii6i;+i;E$[gl;g[}ii
Ea,P!
c)
1'
()
-
=73'a
?--}ii!EEEalia[*$[ii*1it1g3
liiiisEi
ifq1
I oE
B
I
*; (J
6' 6

A$$:saF*3l8il*€:{g3 ; ; sq ;rs
nirEig$iiiiicrlfi F *$i ;$[

ii$Eiii[$iiiiii;f$[i$gi$
$i;g$l8HFgi$;firy iE Ei
mgri3E$i;g[5iiiiS
{Hasa8gr*i=;g{t$
*$
S 6s
ra
3
tt
{
o

pg'$[E:ieaa=il'F
:[il:'xs='#giFFlgJ;- !.:=
a
(o
I

o
;d -aOi#i- 6-' ig
i- o
+ " o:g (o
!
,A
.\ ai *_=c)
._\
i r* .rf,
ba
{
" i
-'t-, { :- lrl
i-
1111
:a -ln -x
l1u"!
-r- I :i
r-J
**
{t Lrt
=:*tit
rlf{- :?: 1.-:,
Li=E:--
:,
Ir *'=: :
.--
"-.: i.a
=.-!
=a;:::=-
i--=
:g-+.==.

,+=

: ":!
....,
i =-1
a-=3

{-u I
I *:
!=r
\"- #t
1 .1 Z
F

l;.-.B'

Вам также может понравиться